Peds Final Cumulative Questions

Pataasin ang iyong marka sa homework at exams ngayon gamit ang Quizwiz!

14. The nurse assesses the patient on Adriamycin very carefully when the patient complains of: 1. nausea. 2. visual disturbances. 3. headache and dizziness. 4. rapid heart beat.

ANS: 4 Adriamycin is cardiotoxic and can cause heart failure.

Chronic adrenocortical insufficiency is also referred to as: a. Graves disease. b. Addison disease. c. Cushing syndrome. d. Hashimoto disease.

ANS: B Addison disease is chronic adrenocortical insufficiency.

The nurse is caring for an 11-year-old boy who has recently been diagnosed with diabetes. Which of the following should be included in the teaching plan for daily injections? a. The parents do not need to learn the procedure. b. He is old enough to give most of his own injections. c. Self-injections will be possible when he is closer to adolescence. d. He can learn about self-injections when he is able to reach all injection sites.

ANS: B School-age children are able to give their own injections.

Diabetes insipidus is a disorder of which of the following? a. Anterior pituitary b. Posterior pituitary c. Adrenal cortex d. Adrenal medulla

ANS: B The principal disorder of posterior pituitary hypofunction is diabetes insipidus.

To help the adolescent deal with diabetes, the nurse must consider which of the following characteristics of adolescence? a. Desire to be unique b. Preoccupation with the future c. Need to be perfect and similar to peers d. Need to make peers aware of the seriousness of hypoglycemic reactions

ANS: C Adolescence is a time when the individual wants to be perfect and similar to peers. Having diabetes makes adolescents different from their peers.

An 18-month-old child is seen in the clinic with AOM. Trimethoprim-sulfamethoxazole (Bactrim) is prescribed. Which statement made by the parent indicates a correct understanding of the instructions?

"I will administer all prescribed medications"

Parents have understood teaching about prevention of childhood otitis media if they make which statement?

"We will be sure to keep immunizations up to date."

45. Which of the following occurs in septic shock? 1. Hypothermia 2. Increased cardiac output 3. Vasoconstriction 4. Angioneurotic edema

ANS: 2 2. Increased cardiac output, which results in warm, flushed skin, is one of the manifestations of septic shock

Which is a useful skill that the nurse should expect a 5-year-old child to be able to master? a. Tie shoelaces. b. Use knife to cut meat. c. Hammer a nail. d. Make change out of a quarter.

ANS: A Tying shoelaces is a fine motor task of 5-year-olds. Using a knife to cut meat is a fine motor task of a 7-year-old. Hammering a nail and making change out of a quarter are fine motor and cognitive tasks of an 8- to 9-year-old

According to Piaget, the adolescent is in the fourth stage of cognitive development, or period of what? a. Formal operations b. Concrete operations c. Conventional thought d. Postconventional thought

ANS: A Feedback A Cognitive thinking culminates with capacity for abstract thinking. This stage, the period of formal operations, is Piaget's fourth and last stage. B Concrete operations usually develops between ages 7 and 11 years. C Conventional and postconventional thought refer to Kohlberg's stages of moral development. D Conventional and postconventional thought refer to Kohlberg's stages of moral development.

A nasal spray of desmopressin acetate (DDAVP) is used to treat: a. hypopituitarism. b. diabetes insipidus. c. acute adrenocortical insufficiency. d. syndrome of inappropriate ADH.

ANS: B The drug of choice for the treatment of diabetes insipidus is DDAVP, which is a synthetic analog of vasopressin.

16. Where in the health history should the nurse describe all details related to the chief complaint? a. Past history b. Chief complaint c. Present illness d. Review of systems

ANS: C

Which play is most typical of the preschool period? a. Solitary b. Parallel c. Associative d. Team

ANS: C Associative play is group play in similar or identical activities but without rigid organization or rules. Solitary play is that of infants. Parallel play is that of toddlers. School-age children play in teams

Hyperglycemia associated with diabetic ketoacidosis is defined as a blood glucose measurement equal to or greater than: a. 185 mg/dl. b. 220 mg/dl. c. 280 mg/dl. d. 330 mg/dl.

ANS: D Diabetic ketoacidosis is a state of relative insulin insufficiency and may include the presence of hyperglycemia, a blood glucose level greater than or equal to 330 mg/dl.

The nurse is caring for a child with suspected diabetes insipidus. Which of the following clinical manifestations would the nurse expect to observe? a. Oliguria b. Glycosuria c. Nausea and vomiting d. Polyuria and polydipsia

ANS: D Excessive urination accompanied by insatiable thirst is the primary clinical manifestation of diabetes. These symptoms may be so severe that the child does little other than drink and urinate.

A child will start treatment for precocious puberty. This involves injections of synthetic: a. thyrotropin. b. gonadotropins. c. somatotropic hormone. d. luteinizing hormone-releasing hormone.

ANS: D Precocious puberty of central origin is treated with monthly subcutaneous injections of luteinizing hormone-releasing hormone.

The nurse is discussing with a child and family the various sites used for insulin injections. Which of the following sites usually has the fastest rate of absorption? a. Arm b. Leg c. Buttock d. Abdomen

ANS: D The abdomen has the fastest rate of absorption but the shortest duration.

The school nurse has been asked to begin teaching sex education in the 5th grade. The nurse should recognize that a. Children in 5th grade are too young for sex education. b. Children should be discouraged from asking too many questions. c. Correct terminology should be reserved for children who are older. d. Sex can be presented as a normal part of growth and development.

ANS: D Feedback A Fifth graders are usually 10 to 11 years old. This age is not too young to speak about physiologic changes in their bodies. B They should be encouraged to ask questions. C Preadolescents need precise and concrete information. D When sexual information is presented to school-age children, sex should be treated as a normal part of growth and development.

Which following parameters correlate best with measurements of the body's total protein stores? a. Height b. Weight c. Skin-fold thickness d. Upper arm circumference

ANS: D Upper arm circumference is correlated with measurements of total muscle mass. Muscle serves as the body's major protein reserve and is considered an index of the body's protein stores. Height is reflective of past nutritional status. Weight is indicative of current nutritional status. Skin-fold thickness is a measurement of the body's fat content

____________________ is the leading cause of death in children of every age-group beyond 1 year of age.

ANS: Unintentional injury

It is generally recommended that a child with acute streptococcal pharyngitis can return to school:

After taking antibiotics for 24 hours

When caring for an infant with an upper respiratory tract infection and elevated temperature, an appropriate nursing intervention is to:

Give small amounts of favorite fluids frequently to prevent dehydration.

Which statement expresses accurately the genetic implications of cystic fibrosis (CF)?

If it is present in a child, both parents are carriers of this defective gene.

The earliest recognizable clinical manifestation of cystic fibrosis (CF) is:

Meconium ileus.

The nurse is caring for a child with acute respiratory distress syndrome (ARDS) associated with sepsis. Nursing actions should include:

Monitor pulse oximetry.

Skin testing for tuberculosis (the Mantoux test) is recommended:

Periodically for children who reside in high-prevalence regions.

The parent of an infant with nasopharyngitis should be instructed to notify the health care professional if the infant:

Shows signs of an earache.

nurse encourages mother of a toddler with acute laryngotracheobronchitis to

stay at the bedside as much as possible. rationale for this action is primarily mother's presence reduce anxiety ease the child's respiratory efforts

A child with cystic fibrosis is receiving recombinant human deoxyribonuclease (rhDNase). This drug: May cause

voice alterations.

19. Which of the following is a common sign of digoxin toxicity? 1. Seizures 2. Vomiting 3. Bradypnea 4. Tachycardia

ANS: 2 2. Vomiting is a common sign of digoxin toxicity

Which of the following drugs would be used to treat a child who has increased ICP resulting from cerebral edema? a. Mannitol b. Epinephrine hydrochloride c. Atropine sulfate d. Sodium bicarbonate

ANS: A For increased ICP, mannitol, an osmotic diuretic, administered intravenously, is the drug used most frequently for rapid reduction.

20. The family is concerned about the possibility of addiction due to the frequent doses of Roxanol. The nurse's best response is: 1. "At this stage, addiction is the least of our worries." 2. "This is a low-dose drug and controls pain without the problems of addiction." 3. "Addiction is rare, but we have drugs that can counteract the effect of this narcotic." 4. "You may want to share your concerns with the doctor. He can order another drug that is not addictive."

ANS: 2 Roxanol is a low-dose narcotic with very little potential for addiction.

Primary (deciduous) teeth are replaced by permanent teeth. By adulthood the child will have __________ permanent teeth.

ANS: 32 thirty-two

18. The nurse cautions that the most common site of cancer in adult men is the: 1. colon. 2. lung. 3. pancreas. 4. prostate.

ANS: 4 Gender defines some of the potential risks for cancer.

A goiter is an enlargement or hypertrophy of which gland? a. Thyroid b. Adrenal c. Anterior pituitary d. Posterior pituitary

ANS: A A goiter is an enlargement or hypertrophy of the thyroid gland.

The parents of a 14-year-old girl are concerned that their adolescent spends too much time looking in the mirror. Which statement is the most appropriate for the nurse to make? a. "Your teenager needs clearer and stricter limits about her behavior." b. "Your teenager needs more responsibility at home." c. "During adolescence this behavior is not unusual." d. "The behavior is abnormal and needs further investigation."

ANS: C Feedback A Stricter limits are not an appropriate response for a behavior that is part of normal development. B More responsibility at home is not an appropriate response for this situation. C Egocentric and narcissistic behavior is normal during this period of development. The teenager is seeking a personal identity. D The behavior is normal and needs no further investigation.

35. When palpating the child's cervical lymph nodes, the nurse notes that they are tender, enlarged, and warm. What is the best explanation for this? a. Some form of cancer b. Local scalp infection common in children c. Infection or inflammation distal to the site d. Infection or inflammation close to the site

ANS: D Small nontender nodes are normal. Tender, enlarged, and warm lymph nodes may indicate infection or inflammation close to their location. Tender lymph nodes are not usually indicative of cancer. A scalp infection would usually not cause inflamed lymph nodes. The lymph nodes close to the site of inflammation or infection would be inflamed

32. Pulses can be graded according to certain criteria. Which is a description of a normal pulse: a. 0 b. +1 c. +2 d. +3

ANS: D A normal pulse is described as +3. A pulse that is easy to palpate and not easily obliterated with pressure is considered normal. A pulse graded 0 is not palpable. A pulse graded +1 is difficult to palpate, thready, weak, and easily obliterated with pressure. A pulse graded +2 is difficult to palpate and may be easily obliterated with pressure.

By which age should the nurse expect that most children could obey prepositional phrases such as "under," "on top of," "beside," and "behind"? a. 18 months b. 24 months c. 3 years d. 4 years

ANS: D At 4 years, children can understand directional phrases. Children at 18 months, 24 months, and 3 years are too young.

Which characteristic best describes the language of a 3-year-old child? a. Asks meanings of words b. Follows directional commands c. Describes an object according to its composition d. Talks incessantly regardless of whether anyone is listening

ANS: D Because of the dramatic vocabulary increase at this age, 3-year-olds are known to talk incessantly regardless of whether anyone is listening. A 4- to 5-year-old asks lots of questions and can follow simple directional commands. A 6-year-old can describe an object according to its composition.

The nurse is caring for a child with severe head trauma after a car accident. Which of the following is an ominous sign that often precedes death? a. Papilledema b. Delirium c. Doll's head maneuver d. Periodic and irregular breathing

ANS: D Periodic or irregular breathing is an ominous sign of brainstem (especially medullary) dysfunction that often precedes complete apnea.

A child eats some sugar cubes after experiencing symptoms of hypoglycemia. This rapid-releasing sugar should be followed by which of the following? a. Fat b. Fruit juice c. Several glasses of water d. Complex carbohydrate and protein

ANS: D Symptoms of hypoglycemia are treated with a rapid-releasing sugar source followed by a complex carbohydrate and protein.

12. The nurse is having difficulty communicating with a hospitalized 6-year-old child. What technique might be most helpful? a. Suggest that the child keep a diary. b. Suggest that the parent read fairy tales to the child. c. Ask the parent if the child is always uncommunicative. d. Ask the child to draw a picture.

ANS: D Drawing is one of the most valuable forms of communication. Children's drawings tell a great deal about them because they are projections of the child's inner self. It would be difficult for a 6-year-old child who is most likely learning to read to keep a diary. Parents reading fairy tales to the child is a passive activity involving the parent and child. It would not facilitate communication with the nurse. The child is in a stressful situation and is probably uncomfortable with strangers

38. At what age should the nurse expect the anterior fontanel to close? a. 2 months b. 2 to 4 months c. 6 to 8 months d. 12 to 18 months

ANS: D The anterior fontanel normally closes between ages 12 and 18 months. Two to 8 months is too early. The expected closure of the anterior fontanel occurs between ages 12 and 18 months; if it closes between ages 2 and 8 months, the child should be referred for further evaluation

The nurse is caring for a 10-month-old infant with respiratory syncytial virus (RSV) bronchiolitis. Which interventions should be included in the child's care

Encourage infant to drink 8 ounces of formula every 4 hours. Institute cluster care to encourage adequate rest. Place on noninvasive oxygen monitoring.

Which type of croup is always considered a medical emergency?

Epiglottitis is always a medical emergency needing antibiotics and airway support for treatment

Which information should the nurse teach workers at a day care center about respiratory syncytial virus (RSV)?

Frequent hand washing can decrease the spread of the virus.

Pancreatic enzymes are administered to the child with cystic fibrosis. Nursing should include:

Pancreatic enzymes can be swallowed whole or sprinkled on a small amount of food taken at the beginning of a meal.

Why do infants and young children quickly have respiratory distress in acute and chronic alterations of the respiratory system?

Mucus and edema obstruct small airways.

43. An infant with pyloric stenosis experiences excessive vomiting that can result in: a. hyperchloremia. b. hypernatremia. c. metabolic acidosis. d. metabolic alkalosis.

NS: D Infants with excessive vomiting are prone to metabolic alkalosis from the loss of hydrogen ions. Chloride ions and sodium are lost with vomiting. Metabolic alkalosis, not acidosis, is likely.

Which statement best describes why children have fewer respiratory tract infections as they grow older?

Repeated exposure to organisms causes increased immunity.

Which information should the nurse teach families about reducing exposure to pollens and dust

Replace wall-to-wall carpeting with wood and tile floors. Use an air conditioner. Put dust-proof covers on pillows and mattresses.

A parent whose two school-age children have asthma asks the nurse in what sports, if any, they can participate. The nurse should recommend:

Swimming.

Which intervention for treating croup at home should be taught to parents?

Take the child outside.Taking the child into the cool, humid, night air may relieve mucosal swelling and improve symptoms

The nurse is conducting an assessment on a school-age child with urosepsis. Which assessment finding should the nurse expect? a. Fever with a positive blood culture b. Proteinuria and edema c. Oliguria and hypertension d. Anemia and thrombocytopenia

a. Fever with a positive blood culture Symptoms of urosepsis include a febrile UTI coexisting with systemic signs of bacterial illness; blood culture reveals presence of urinary pathogen. Proteinuria and edema are symptoms of minimal change nephrotic syndrome (MCNS). Oliguria and hypertension are symptoms of acute glomerulonephritis (AGN). Anemia and thrombocytopenia are symptoms of hemolytic uremic syndrome (HUS).

The nurse is planning care for an adolescent with AIDS. Which is the priority nursing goal? a. Preventing infection b. Preventing secondary cancers c. Restoring immunologic defenses d. Identifying source of infection

a. Preventing infection Because the child is immunocompromised in association with HIV infection, the prevention of infection is paramount. Although certain precautions are justified in limiting exposure to infection, these must be balanced with the concern for the child's normal developmental needs. Preventing secondary cancers is not currently possible. Current drug therapy is affecting the disease progression; although not a cure, these drugs can suppress viral replication, preventing further deterioration. Case finding is not a priority nursing goal.

Which term refers to those times in an individual's life when he or she is more susceptible to positive or negative influences? a. Sensitive period c. Terminal points b. Sequential period d. Differentiation points

a. Sensitive period

Which should the nurse recommend to prevent urinary tract infections in young girls? a. Wear cotton underpants. b. Limit bathing as much as possible. c. Increase fluids; decrease salt intake. d. Cleanse perineum with water after voiding.

a. Wear cotton underpants. Cotton underpants are preferable to nylon underpants. No evidence exists that limiting bathing, increasing fluids/decreasing salt intake, or cleansing the perineum with water after voiding decrease urinary tract infections in young girls.

A mother reports that her 6-year-old child is highly active and irritable and that she has irregular habits and adapts slowly to new routines, people, or situations. According to Chess and Thomas, which category of temperament best describes this child? a. Easy child c. Slow-to-warm-up child b. Difficult child d. Fast-to-warm-up child

b. Difficult child

Which statement is true about the basal metabolic rate (BMR) in children? a. It is reduced by fever. b. It is slightly higher in boys than in girls at all ages. c. It increases with the age of child. d. It decreases as proportion of surface area to body mass increases.

b. It is slightly higher in boys than in girls at all ages.

Parents of a school-age child with hemophilia ask the nurse, "Which sports are recommended for children with hemophilia?" Which sports should the nurse recommend? (Select all that apply.) a. Soccer b. Swimming c. Basketball d. Golf e. Bowling

b. Swimming d. Golf e. Bowling ANS: B, D, E Because almost all persons with hemophilia are boys, the physical limitations in regard to active sports may be a difficult adjustment, and activity restrictions must be tempered with sensitivity to the child's emotional and physical needs. Use of protective equipment, such as padding and helmets, is particularly important, and noncontact sports, especially swimming, walking, jogging, tennis, golf, fishing, and bowling, are encouraged. Contact sport such as soccer and basketball are not recommended.

Which "expected outcome" would be developmentally appropriate for a hospitalized 4-year-old child? a. The child will be dressed and fed by the parents. b. The child will independently ask for play materials or other personal needs. c. The child will be able to verbalize an understanding of the reason for the hospitalization. d. The child will have a parent stay in the room at all times.

b. The child will independently ask for play materials or other personal needs.

Which should the nurse teach about prevention of sickle cell crises to parents of a preschool child with sickle cell disease? (Select all that apply.) a. Limit fluids at bedtime. b. Notify the health care provider if a fever of 38.5° C (101.3° F) or greater occurs. c. Give penicillin as prescribed. d. Use ice packs to decrease the discomfort of vasoocclusive pain in the legs. e. Notify the health care provider if your child begins to develop symptoms of a cold.

b. Notify the health care provider if a fever of 38.5° C (101.3° F) or greater occurs. c. Give penicillin as prescribed. e. Notify the health care provider if your child begins to develop symptoms of a cold. ANS: B, C, E The most important issues to teach the family of a child with sickle cell anemia are to (1) seek early intervention for problems, such as a fever of 38.5° C (101.3° F) or greater; (2) give penicillin as ordered; (3) recognize signs and symptoms of splenic sequestration, as well as respiratory problems that can lead to hypoxia; and (4) treat the child normally. The nurse emphasizes the importance of adequate hydration to prevent sickling and to delay the adhesion-stasis-thrombosis-ischemia cycle. It is not sufficient to advise parents to "force fluids" or "encourage drinking." They need specific instructions on how many daily glasses or bottles of fluid are required. Many foods are also a source of fluid, particularly soups, flavored ice pops, ice cream, sherbet, gelatin, and puddings. Increased fluids combined with impaired kidney function result in the problem of enuresis. Parents who are unaware of this fact frequently use the usual measures to discourage bedwetting, such as limiting fluids at night. Enuresis is treated as a complication of the disease, such as joint pain or some other symptom, to alleviate parental pressure on the child. Ice should not be used during a vasoocclusive pain crisis because it vasoconstricts and impairs circulation even more.

Which best describes acute glomerulonephritis? a. Occurs after a urinary tract infection b. Occurs after a streptococcal infection c. Associated with renal vascular disorders d. Associated with structural anomalies of genitourinary tract

b. Occurs after a streptococcal infection Acute glomerulonephritis is an immune-complex disease that occurs after a streptococcal infection with certain strains of the group A â-hemolytic streptococcus. Acute glomerulonephritis usually follows streptococcal pharyngitis and is not associated with renal vascular disorders or genitourinary tract structural anomalies.

An adolescent will receive a bone marrow transplant (BMT). The nurse should explain that the bone marrow will be administered by which route? a. Bone grafting b. Bone marrow injection c. IV infusion d. Intra-abdominal infusion

c. IV infusion Bone marrow from a donor is infused intravenously, and the transfused stem cells will repopulate the marrow. Because the stem cells migrate to the recipient's marrow when given intravenously, this is the method of administration.

Which statement best describes -thalassemia major (Cooley anemia)? a. All formed elements of the blood are depressed. b. Inadequate numbers of red blood cells are present. c. Increased incidence occurs in families of Mediterranean extraction. d. Increased incidence occurs in persons of West African descent.

c. Increased incidence occurs in families of Mediterranean extraction. Individuals who live near the Mediterranean Sea and their descendants have the highest incidence of thalassemia. An overproduction of red cells occurs. Although numerous, the red cells are relatively unstable. Sickle cell disease is common in persons of West African descent.

The nurse is conducting a staff in-service on sickle cell anemia. Which describes the pathologic changes of sickle cell anemia? a. Sickle-shaped cells carry excess oxygen. b. Sickle-shaped cells decrease blood viscosity. c. Increased red blood cell destruction occurs. d. Decreased adhesion of sickle-shaped cells occurs.

c. Increased red blood cell destruction occurs. The clinical features of sickle cell anemia are primarily the result of increased red blood cell destruction and obstruction caused by the sickle-shaped red blood cells. Sickled red cells have decreased oxygen-carrying capacity and transform into the sickle shape in conditions of low oxygen tension. When the sickle cells change shape, they increase the viscosity in the area where they are involved in the microcirculation. Increased adhesion and entanglement of cells occurs.

Which should the nurse recommend for the diet of a child with chronic renal failure? a. High in protein b. Low in vitamin D c. Low in phosphorus d. Supplemented with vitamins A, E, and K

c. Low in phosphorus Dietary phosphorus is controlled by the reduction of protein and milk intake to prevent or control the calcium-phosphorus imbalance. Protein should be limited in chronic renal failure to decrease intake of phosphorus. Vitamin D therapy is administered in chronic renal failure to increase calcium absorption. Supplementation of vitamins A, E, and K is not part of dietary management in chronic renal disease.

According to Kohlberg, children develop moral reasoning as they mature. What is most characteristic of a preschooler's stage of moral development? a. Obeying the rules of correct behavior is important. b. Showing respect for authority is important behavior. c. Behavior that pleases others is considered good. d. Actions are determined as good or bad in terms of their consequences.

d. Actions are determined as good or bad in terms of their consequences.

A child's skeletal age is best determined by: a. Assessment of dentition. c. Facial bone development. b. Assessment of height over time. d. Radiographs of the hand and wrist.

d. Radiographs of the hand and wrist.

A toddler playing with sand and water would be participating in _____ play. a. Skill c. Social-affective b. Dramatic d. Sense-pleasure

d. Sense-pleasure

Which function of play is a major component of play at all ages? a. Creativity c. Intellectual development b. Socialization d. Sensorimotor activity

d. Sensorimotor activity

Which is the most common cause of acute renal failure in children? a. Pyelonephritis b. Tubular destruction c. Urinary tract obstruction d. Severe dehydration

d. Severe dehydration The most common cause of acute renal failure in children is dehydration or other causes of poor perfusion that may respond to restoration of fluid volume. Pyelonephritis and tubular destruction are not common causes of acute renal failure. Obstructive uropathy may cause acute renal failure, but it is not the most common cause.

A child has had cold symptoms for more than 2 weeks, a headache, nasal congestion with purulent nasal drainage, facial tenderness, and a cough that increases

during sleep.

41. When caring for the child with Kawasaki disease, the nurse should know which of the following? 1. Child's fever is usually responsive to antibiotics within 48 hours. 2. Principal area of involvement is the joints. 3. Aspirin is contraindicated. 4. Therapeutic management includes administration of gamma globulin and aspirin.

ANS: 4 4. High-dose IV gamma globulin and aspirin therapy is indicated to reduce the incidence of coronary artery abnormalities when given within the first 10 days of the illness

5. Nursing interventions for the child after a cardiac catheterization would include which of the following? 1. Allow ambulation as tolerated. 2. Monitor vital signs every 2 hours. 3. Assess the affected extremity for temperature and color. 4. Check pulses above the catheterization site for equality and symmetry.

ANS: 3 3. The extremity that was used for access for the cardiac catheterization must be checked for temperature and color. Coolness and blanching may indicate arterial occlusion.

Cystic fibrosis is suspected in a toddler. Which test is essential in establishing this diagnosis?

A sweat chloride test result greater than 60 mEq/L is diagnostic of CF

Asthma in infants is usually triggered by:

A viral infection Viral illnesses cause inflammation that causes increased airway reactivity in asthma.

What factors indicate that parents should seek genetic counseling for their child (Select all that apply)? a. Abnormal newborn screen b. Family history of a hereditary disease c. History of hypertension in the family d. Severe colic as an infant e. Metabolic disorder

A, B, E

Play serves many purposes. In teaching parents about appropriate activities, the nurse should inform them that play serves the following function (Select all that apply): a. Intellectual development b. Physical development c. Self-awareness d. Creativity e. Temperament development

A, C, D

A nurse is preparing to administer a Denver II. Which statement(s) about the Denver II test is (are) accurate (Select all that apply)? a. All items intersected by the age line should be administered. b. There is no correction for a child born prematurely. c. The tool is an intelligence test. d. Toddlers and preschoolers should be prepared by presenting the test as a game. e. Presentation of the toys from the kit should be done one at a time.

A, D, E

The treatment of brain tumors in children consists of which of the following therapies? (Select all that apply.) a. Surgery b. Bone marrow transplantation c. Chemotherapy d. Stem cell transplantation e. Radiation f. Myelography

ANS: A, C, E Treatment for brain tumors in children may consist of surgery, chemotherapy, and radiotherapy alone or in combination.

11. When the nurse interviews an adolescent, which is especially important? a. Focus the discussion on the peer group. b. Allow an opportunity to express feelings. c. Emphasize that confidentiality will always be maintained. d. Use the same type of language as the adolescent.

ANS: B

An infant's parents ask the nurse about preventing otitis media (OM). What should the nurse recommend?

Avoid tobacco smoke.

A child with cystic fibrosis (CF) receives aerosolized bronchodilator medication. When should this medication be administered?

Before chest physiotherapy (CPT) Bronchodilators should be given before CPT to open bronchi and make expectoration easier

A school-age child has had an upper respiratory tract infection for several days and then began having a persistent dry, hacking cough that was worse at night. The cough has become productive in the past 24 hours.This is most suggestive of:

Bronchitis.

A nurse is charting that a hospitalized child has labored breathing. Which describes labored breathing?

Dyspnea

The mother of a toddler yells to the nurse, "Help! He is choking to death on his food." The nurse determines that lifesaving measures are necessary based on:

Inability to speak.

The parent of a toddler calls the nurse, asking about croup. What is a distinguishing manifestation of spasmodic croup?

It is marked by a harsh, metallic, barky cough; sore throat; inspiratory stridor; and hoarseness.

Which statement is characteristic of acute otitis media (AOM)?

It is treated with a broad range of antibiotics.

Which vitamin supplements are necessary for children with cystic fibrosis?

Vitamins A, D, E, and K Fat-soluble vitamins are poorly absorbed because of deficient

The nurse is preparing to give oral care to a school-age child with mucositis secondary to chemotherapy administered to treat leukemia. Which preparations should the nurse use for oral care on this child? (Select all that apply.) a. Chlorhexidine gluconate (Peridex) b. Lemon glycerin swabs c. Antifungal troches (lozenges) d. Lip balm (Aquaphor) e. Hydrogen peroxide

a. Chlorhexidine gluconate (Peridex) c. Antifungal troches (lozenges) d. Lip balm (Aquaphor) ANS: A, C, D Preparations that may be used to prevent or treat mucositis include chlorhexidine gluconate (Peridex) because of its dual effectiveness against candidal and bacterial infections, antifungal troches (lozenges) or mouthwash, and lip balm (e.g., Aquaphor) to keep the lips moist. Agents that should not be used include lemon glycerin swabs (irritate eroded tissue and can decay teeth), hydrogen peroxide (delays healing by breaking down protein), and milk of magnesia (dries mucosa).

The theorist who viewed developmental progression as a lifelong series of conflicts that need resolution is: a. Erikson. c. Kohlberg. b. Freud. d. Piaget.

a. Erikson.

The nurse is teaching parents about the importance of iron in a toddler's diet. Which explains why iron deficiency anemia is common during toddler-hood? a. Milk is a poor source of iron. b. Iron cannot be stored during fetal development. c. Fetal iron stores are depleted by age 1 month. d. Dietary iron cannot be started until age 12 months.

a. Milk is a poor source of iron. Children between the ages of 12 and 36 months are at risk for anemia because cow's milk is a major component of their diet and it is a poor source of iron. Iron is stored during fetal development, but the amount stored depends on maternal iron stores. Fetal iron stores are usually depleted by age 5 to 6 months. Dietary iron can be introduced by breastfeeding, iron-fortified formula, and cereals during the first 12 months of life.

What is probably the single most important influence on growth at all stages of development? a. Nutrition c. Culture b. Heredity d. Environment

a. Nutrition

The nurse is conducting an admission assessment on a school-age child with acute renal failure. Which are the primary clinical manifestations the nurse expects to find with this condition? a. Oliguria and hypertension b. Hematuria and pallor c. Proteinuria and muscle cramps d. Bacteriuria and facial edema

a. Oliguria and hypertension The principal feature of acute renal failure is oliguria; hypertension is a nonspecific clinical manifestation. Hematuria and pallor, proteinuria and muscle cramps, and bacteriuria and facial edema are not principal features of acute renal failure.

The nurse is conducting a staff in-service on inherited childhood blood disorders. Which statement describes severe combined immunodeficiency syndrome (SCIDS)? a. There is a deficit in both the humoral and cellular immunity with this disease. b. Production of red blood cells is affected with this disease. c. Adult hemoglobin is replaced by abnormal hemoglobin in this disease. d. There is a deficiency of T and B lymphocyte production with this disease.

a. There is a deficit in both the humoral and cellular immunity with this disease. Severe combined immunodeficiency syndrome (SCIDS) is a genetic disorder that results in deficits of both humoral and cellular immunity. Wiskott-Aldrich is an X-linked recessive disorder with selected deficiencies of T and B lymphocytes. Fanconi syndrome is a hereditary disorder of red cell production. Sickle cell disease is characterized by the replacement of adult hemoglobin with an abnormal hemoglobin S.

The nurse is conducting teaching for an adolescent being discharged to home after a renal transplant. The adolescent needs further teaching if which statement is made? a. "I will report any fever to my primary health care provider." b. "I am glad I only have to take the immunosuppressant medication for two weeks." c. "I will observe my incision for any redness or swelling." d. "I won't miss doing kidney dialysis every week."

b. "I am glad I only have to take the immunosuppressant medication for two weeks." The immunosuppressant medications are taken indefinitely after a renal transplant, so they should not be discontinued after two weeks. Reporting a fever and observing an incision for redness and swelling are accurate statements. The adolescent is correct in indicating dialysis will not need to be done after the transplant.

Which statement is true about toy safety? a. Adults should be the only ones who select toys. b. Adults should be alert to notices of recalls by manufacturers. c. Government agencies inspect all toys on the market. d. Evaluation of toy safety is a joint effort between children and adults.

b. Adults should be alert to notices of recalls by manufacturers.

A young boy will receive a bone marrow transplant (BMT). This is possible because one of his older siblings is a histocompatible donor. Which is this type of BMT called? a. Syngeneic b. Allogeneic c. Monoclonal d. Autologous

b. Allogeneic Allogeneic transplants are from another individual. Because he and his sibling are histocompatible, the BMT can be done. Syngeneic marrow is from an identical twin. There is no such thing as a monoclonal BMT. Autologous refers to the individual's own marrow.

A boy with leukemia screams whenever he needs to be turned or moved. Which is the most probable cause of this pain? a. Edema b. Bone involvement c. Petechial hemorrhages d. Changes within the muscles

b. Bone involvement The invasion of the bone marrow with leukemic cells gradually causes a weakening of the bone and a tendency toward fractures. As leukemic cells invade the periosteum, increasing pressure causes severe pain. Edema, petechial hemorrhages, and changes within the muscles would not cause severe pain.

The nurse is conducting a staff in-service on newborn defects of the genitourinary system. Which describes the narrowing of the preputial opening of the foreskin? a. Chordee b. Phimosis c. Epispadias d. Hypospadias

b. Phimosis Phimosis is the narrowing or stenosis of the preputial opening of the foreskin. Chordee is the ventral curvature of the penis. Epispadias is the meatal opening on the dorsal surface of the penis. Hypospadias is a congenital condition in which the urethral opening is located anywhere along the ventral surface of the penis

Which is often administered to prevent or control hemorrhage in a child with cancer? a. Nitrosoureas b. Platelets c. Whole blood d. Corticosteroids

b. Platelets Most bleeding episodes can be prevented or controlled with the administration of platelet concentrate or platelet-rich plasma. Nitrosoureas, whole blood, and corticosteroids would not prevent or control hemorrhage.

The parents of a child hospitalized with sickle cell anemia tell the nurse that they are concerned about narcotic analgesics causing addiction. Which is appropriate for the nurse to explain about narcotic analgesics? a. Are often ordered but not usually needed b. Rarely cause addiction because they are medically indicated c. Are given as a last resort because of the threat of addiction d. Are used only if other measures, such as ice packs, are ineffective

b. Rarely cause addiction because they are medically indicated The pain of sickle cell anemia is best treated by a multidisciplinary approach. Mild to moderate pain can be controlled by ibuprofen and acetaminophen. When narcotics are indicated, they are titrated to effect and are given around the clock. Patient-controlled analgesia reinforces the patient's role and responsibility in managing the pain and provides flexibility in dealing with pain. Few, if any, patients who receive opioids for severe pain become behaviorally addicted to the drug. Narcotics are often used because of the severe nature of the pain of vasoocclusive crisis. Ice is contraindicated because of its vasoconstrictive effects.

The nurse is teaching parents about prevention of urinary tract infections in children. Which factor predisposes the urinary tract to infection? a. Increased fluid intake b. Short urethra in young girls c. Prostatic secretions in males d. Frequent emptying of the bladder

b. Short urethra in young girls The short urethra in females provides a ready pathway for invasion of organisms. Increased fluid intake and frequent emptying of the bladder offer protective measures against urinary tract infections. Prostatic secretions have antibacterial properties that inhibit bacteria.

Which is an objective of care for a 10-year-old child with minimal change nephrotic syndrome? a. Reduce blood pressure. b. Reduce excretion of urinary protein. c. Increase excretion of urinary protein. d. Increase ability of tissues to retain fluid.

b. Reduce excretion of urinary protein. The objectives of therapy for the child with minimal change nephrotic syndrome include reduction of the excretion of urinary protein, reduction of fluid retention, prevention of infection, and minimization of complications associated with therapy. Blood pressure is usually not elevated in minimal change nephrotic syndrome. Excretion of urinary protein and fluid retention are part of the disease process and must be reversed.

Which is included in the diet of a child with minimal change nephrotic syndrome? a. High protein b. Salt restriction c. Low fat d. High carbohydrate

b. Salt restriction Salt is usually restricted (but not eliminated) during the edema phase. The child has little appetite during the acute phase. Favorite foods are provided (with the exception of high-salt ones) in an attempt to provide nutritionally complete meals.

What is characteristic of the preoperational stage of cognitive development? a. Thinking is logical. c. Reasoning is inductive. b. Thinking is concrete. d. Generalizations can be made.

b. Thinking is concrete

Which immunization should be given with caution to children infected with human immunodeficiency virus (HIV)? a. Influenza b. Varicella c. Pneumococcal d. Inactivated poliovirus (IPV)

b. Varicella The children should be carefully evaluated before being given live viral vaccines such as varicella, measles, mumps, and rubella. The child must be immunocompetent and not have contact with other severely immunocompromised individuals. Influenza, pneumococcal, and inactivated poliovirus (IPV) are not live vaccines.

The nurse is teaching the parent about the diet of a child experiencing severe edema associated with acute glomerulonephritis. Which information should the nurse include in the teaching? a. "You will need to decrease the number of calories in your child's diet." b. "Your child's diet will need an increased amount of protein." c. "You will need to avoid adding salt to your child's food." d. "Your child's diet will consist of low-fat, low-carbohydrate foods."

c. "You will need to avoid adding salt to your child's food." For most children, a regular diet is allowed, but it should contain no added salt. The child should be offered a regular diet with favorite foods. Severe sodium restrictions are not indicated.

Which is caused by a virus that primarily infects a specific subset of T lymphocytes, the CD4+ T cells? a. Wiskott-Aldrich syndrome b. Idiopathic thrombocytopenic purpura c. Acquired immunodeficiency syndrome (AIDS) d. Severe combined immunodeficiency disease

c. Acquired immunodeficiency syndrome (AIDS) AIDS is caused by the human immunodeficiency virus (HIV), which primarily attacks the CD4+ T cells. Wiskott-Aldrich syndrome, idiopathic thrombocytopenic purpura, and severe combined immunodeficiency disease are not viral illnesses.

The intrauterine environment can have a profound and permanent effect on the developing fetus with or without chromosome or gene abnormalities. Most adverse intrauterine effects are the result of teratogens. The nurse is cognizant that this group of agents does not include: a. Accutane c. Amniotic bands b. Rubella d. Alcohol

c. Amniotic bands

The nurse is teaching parents of an infant about the causes of iron deficiency anemia. Which statement best describes iron deficiency anemia in infants? a. It is caused by depression of the hematopoietic system. b. It is easily diagnosed because of an infant's emaciated appearance. c. Clinical manifestations are similar regardless of the cause of the anemia. d. Clinical manifestations result from a decreased intake of milk and the premature addition of solid foods.

c. Clinical manifestations are similar regardless of the cause of the anemia. In iron deficiency anemia, the child's clinical appearance is a result of the anemia, not the underlying cause. Usually the hematopoietic system is not depressed in iron deficiency anemia. The bone marrow produces red cells that are smaller and contain less hemoglobin than normal red cells. Children who are iron deficient from drinking excessive quantities of milk are usually pale and overweight. They are receiving sufficient calories, but are deficient in essential nutrients. The clinical manifestations result from decreased intake of iron-fortified solid foods and an excessive intake of milk.

The nurse is admitting a school-age child in acute renal failure with reduced glomerular filtration rate. Which urine test is the most useful clinical indication of glomerular filtration rate? a. pH b. Osmolality c. Creatinine d. Protein level

c. Creatinine The most useful clinical indication of glomerular filtration is the clearance of creatinine. It is a substance that is freely filtered by the glomerulus and secreted by the renal tubule cells. The pH and osmolality are not estimates of glomerular filtration. Although protein in the urine demonstrates abnormal glomerular permeability, it is not a measure of filtration rate.

Frequent developmental assessments are important for which reason? a. Stable developmental periods during infancy provide an opportunity to identify any delays or deficits. b. Infants need stimulation specific to the stage of development. c. Critical periods of development occur during childhood. d. Child development is unpredictable and needs monitoring.

c. Critical periods of development occur during childhood.

Which is the most appropriate nursing diagnosis for the child with acute glomerulonephritis? a. Risk for Injury related to malignant process and treatment b. Fluid Volume Deficit related to excessive losses c. Fluid Volume Excess related to decreased plasma filtration d. Fluid Volume Excess related to fluid accumulation in tissues and third spaces

c. Fluid Volume Excess related to decreased plasma filtration Glomerulonephritis has a decreased filtration of plasma, which results in an excessive accumulation of water and sodium that expands plasma and interstitial fluid volumes, leading to circulatory congestion and edema. No malignant process is involved in acute glomerulonephritis. A fluid volume excess is found. The fluid accumulation is secondary to the decreased plasma filtration.

The nurse is planning activity for a 4-year-old child with anemia. Which activity should the nurse plan for this child? a. Game of "hide and seek" in the children's outdoor play area b. Participation in dance activities in the playroom c. Puppet play in the child's room d. A walk down to the hospital lobby

c. Puppet play in the child's room Because the basic pathologic process in anemia is a decrease in oxygen-carrying capacity, an important nursing responsibility is to assess the child's energy level and minimize excess demands. The child's level of tolerance for activities of daily living and play is assessed, and adjustments are made to allow as much self-care as possible without undue exertion. Puppet play in the child's room would not be overly tiring. Hide and seek, dancing, and walking to the lobby would not conserve the anemic child's energy.

Which child should the nurse document as being anemic? a. 7-year-old child with a hemoglobin of 11.5 g/dl b. 3-year-old child with a hemoglobin of 12 g/dl c. 14-year-old child with a hemoglobin of 10 g/dl d. 1-year-old child with a hemoglobin of 13 g/dl

d. 1-year-old child with a hemoglobin of 13 g/dl Anemia is a condition in which the number of red blood cells, or hemoglobin concentration, is reduced below the normal values for age. Anemia is defined as a hemoglobin level below 10 or 11 g/dl. The child with a hemoglobin of 10 g/dl would be considered anemic. The normal hemoglobin for a child after 2 years of age is 11.5 to 15.5 g/dl.

An infant who weighs 7 pounds at birth would be expected to weigh how many pounds at age 1 year? a. 14 c. 18 b. 16 d. 21

d. 21

The nurse is conducting a staff in-service on childhood blood disorders. Which describes the pathology of idiopathic thrombocytopenic purpura? a. Bone marrow failure in which all elements are suppressed b. Deficiency in the production rate of globin chains c. Diffuse fibrin deposition in the microvasculature d. An excessive destruction of platelets

d. An excessive destruction of platelets Idiopathic thrombocytopenic purpura is an acquired hemorrhagic disorder characterized by an excessive destruction of platelets, discolorations caused by petechiae beneath the skin, and a normal bone marrow. Aplastic anemia refers to a bone marrow-failure condition in which the formed elements of the blood are simultaneously depressed. Thalassemia major is a group of blood disorders characterized by deficiency in the production rate of specific hemoglobin globin chains. Disseminated intravascular coagulation is characterized by diffuse fibrin deposition in the microvasculature, consumption of coagulation factors, and endogenous generation of thrombin and plasma.

The predominant characteristic of the intellectual development of the child ages 2 to 7 years is egocentricity. What best describes this concept? a. Selfishness c. Preferring to play alone b. Self-centeredness d. Inability to put self in another's place

d. Inability to put self in another's place

Which is a common side effect of short-term corticosteroid therapy? a. Fever b. Hypertension c. Weight loss d. Increased appetite

d. Increased appetite Side effects of short-term corticosteroid therapy include an increased appetite. Fever is not a side effect of therapy. It may be an indication of infection. Hypertension is not usually associated with initial corticosteroid therapy. Weight gain, not weight loss, is associated with corticosteroid therapy.

The nurse is recommending how to prevent iron deficiency anemia in a healthy, term, breast-fed infant. Which should be suggested? a. Iron (ferrous sulfate) drops after age 1 month b. Iron-fortified commercial formula by age 4 to 6 months c. Iron-fortified infant cereal by age 2 months d. Iron-fortified infant cereal by age 4 to 6 months

d. Iron-fortified infant cereal by age 4 to 6 months Breast milk supplies inadequate iron for growth and development after age 5 months. Supplementation is necessary at this time. The mother can supplement the breastfeeding with iron-fortified infant cereal. Iron supplementation or the introduction of solid foods in a breast-fed baby is not indicated. Providing iron-fortified commercial formula by age 4 to 6 months should be done only if the mother is choosing to discontinue breastfeeding.

Which immunization should not be given to a child receiving chemotherapy for cancer? a. Tetanus vaccine b. Inactivated poliovirus vaccine c. Diphtheria, pertussis, tetanus (DPT) d. Measles, rubella, mumps

d. Measles, rubella, mumps The vaccine used for measles, mumps, and rubella is a live virus and can result in an overwhelming infection. Tetanus vaccine, inactivated poliovirus vaccine, and diphtheria, pertussis, tetanus (DPT) are not live virus vaccines.

Several complications can occur when a child receives a blood transfusion. Which is an immediate sign or symptom of an air embolus? a. Chills and shaking b. Nausea and vomiting c. Irregular heart rate d. Sudden difficulty in breathing

d. Sudden difficulty in breathing Signs of air embolism are sudden difficulty breathing, sharp pain in the chest, and apprehension. Air emboli should be avoided by carefully flushing all tubing of air before connecting to patient. Chills, shaking, nausea, and vomiting are associated with hemolytic reactions. Irregular heart rate is associated with electrolyte disturbances and hypothermia.

A preschool child is being admitted to the hospital with dehydration and a urinary tract infection (UTI). Which urinalysis result should the nurse expect with these conditions? a. WBC <1; specific gravity 1.008 b. WBC <2; specific gravity 1.025 c. WBC >2; specific gravity 1.016 d. WBC >2; specific gravity 1.030

d. WBC >2; specific gravity 1.030 WBC count in a routine urinalysis should be <1 or 2. Over that amount indicates a urinary tract inflammatory process. The urinalysis specific gravity for children with normal fluid intake is 1.016 to 1.022. When the specific gravity is high, dehydration is indicated. A low specific gravity is seen with excessive fluid intake, distal tubular dysfunction, or insufficient antidiuretic hormone secretion.

Cardiopulmonary resuscitation is begun on a toddler. Which pulse is usually palpated because it is the most central and accessible?

Carotid

The nurse is assessing a child with acute epiglottitis. Examining the child's throat by using a tongue depressor might precipitate which symptom or condition?

Complete obstruction

13. The nurse is meeting a 5-year-old child for the first time and would like the child to cooperate during a dressing change. The nurse decides to do a simple magic trick using gauze. This should be interpreted as: a. inappropriate, because of child's age . b. a way to establish rapport. c. too distracting, when cooperation is important. d. acceptable, if there is adequate time

. ANS: B A magic trick or other simple game may help alleviate anxiety for a 5-year-old. It is an excellent method to build rapport and facilitate cooperation during a procedure. Magic tricks appeal to the natural curiosity of young children. The nurse should establish rapport with the child. Failure to do so may cause the procedure to take longer and be more traumatic.

Abdominal thrusts (the Heimlich maneuver) are recommended for airway obstruction in children older than

1 year.

Chronic otitis media with effusion (OME) is differentiated from acute otitis media (AOM) because it is usually characterized by:

A feeling of fullness in the ear.

Children have a total of __________ primary (deciduous) teeth that they begin to lose when they are school age.

ANS: 20 twenty

In assessing adolescents using Tanner staging, sexual maturity is rated using _________ distinct stages. (Your answer should appear as a number.)

ANS: 5 Tanner stages of adolescent sexual development describe five distinct stages of sexual maturity rating. There are separate rating scales for males and females, but both use five stages.

A nurse is planning a class for school-age children on obesity. Which percentile does the body mass index (BMI) need to exceed for a child to be assessed as obese?

ANS: 95 95th When intake of food exceeds expenditure, the excess is stored as fat. Obesity is an excessive accumulation of fat in the body and is assessed in children as a BMI that exceeds the 95th percentile for age.

Adolescent sexuality refers to the thoughts, feelings, and behaviors related to the teen's sexual identity. The most recent research (2009) indicates that 46% of all adolescents have been involved in some kind of sexual activity. The only complete protection from pregnancy and sexually transmitted diseases (STDs) is ____________.

ANS: abstinence Adolescents should be encouraged that there is nothing wrong with abstaining from sexual activity. Adolescents who engage in sexual activity at a young age are more likely to participate in other high-risk behaviors such as alcohol and drug use. Adolescents who demonstrate high self-esteem are more likely to delay sexual intercourse.

Adolescents' eyes and ears are fully developed and, with the exception of minor infections, the sensory system remains quite healthy during this period of development. The mother of a 12-year-old complains to the nurse that she is concerned that her daughter frequently needs changes to her corrective lenses. This is a condition known as ___________.

ANS: myopia Myopia (nearsightedness) occurs in early adolescence, between the ages of 11 and 13 and is a normal part of adolescent development

A toddler with leukemia is on intravenous chemotherapy treatments. The toddler's lab results are WBC: 1000; neutrophils: 7%; nonsegmented neutrophils (bands): 7%. What is this child's absolute neutrophil count (ANC)? (Record your answer in a whole number.)

ANS: 140 To calculate an ANC for a WBC = 1000; neutrophils = 7%; and nonsegmented neutrophils (bands) = 7%, the steps are Step 1: 7% + 7% = 14%. Step 2: 0.14 1000 = 140 ANC.

2. Which of the following is a complication that may occur after a cardiac catheterization? 1. Cardiac arrhythmia 2. Hypostatic pneumonia 3. Congestive heart failure 4. Rapidly increasing blood pressure

ANS: 1 1. Because a catheter is introduced into the heart, a risk exists of catheter-induced dysrhythmias occurring during the procedure. These are usually transient.

16. Which of the following drugs is an angiotensin-converting enzyme (ACE) inhibitor? 1. Captopril (Capoten) 2. Furosemide (Lasix) 3. Spironolactone (Aldactone) 4. Chlorothiazide (Diuril)

ANS: 1 1. Capoten is a drug which is an ACE inhibitor.

15. A beneficial effect of administering digoxin (Lanoxin) is that it: 1. Decreases edema. 2. Decreases cardiac output. 3. Increases heart size. 4. Increases venous pressure.

ANS: 1 1. Digoxin has a rapid onset and is useful increasing cardiac output, decreasing venous pressure, and as a result, decreasing edema.

33. What painful, tender, pea-sized nodules may appear on the pads of the fingers or toes in bacterial endocarditis? 1. Osler nodes 2. Janeway lesions 3. Subcutaneous nodules 4. Aschoff nodes

ANS: 1 1. Osler nodes are red, painful, intradermal nodes found on pads of the phalanges in bacterial endocarditis.

36. Which of the following is a major clinical manifestation of rheumatic fever? 1. Polyarthritis 2. Osler nodes 3. Janeway spots 4. Splinter hemorrhages of distal third of nails

ANS: 1 1. Polyarthritis, which is swollen, hot, red, and painful joints. The affected joints will change every 1 to 2 days. Primarily the large joints are affected.

11. Which of the following structural defects constitute tetralogy of Fallot? 1. Pulmonary stenosis, ventricular septal defect, overriding aorta, right ventricular hypertrophy 2. Aortic stenosis, ventricular septal defect, overriding aorta, right ventricular hypertrophy 3. Aortic stenosis, atrial septal defect, overriding aorta, left ventricular hypertrophy 4. Pulmonary stenosis, ventricular septal defect, aortic hypertrophy, left ventricular hypertrophy

ANS: 1 1. Tetralogy of Fallot has these four characteristics: pulmonary stenosis, ventricular septal defect, overriding aorta, and right ventricular hypertrophy.

8. Which of the following should be included in the instructions to an active adolescent who is going home after a cardiac catheterization? 1. Avoid tub baths but may shower. 2. Maintain strict bed rest for 3 days. 3. Leave pressure dressing on for 7 days. 4. Stay home from school until Band-Aid is removed.

ANS: 1 1. The catheterization site should be kept relatively dry with a adhesive bandage. Showers are recommended.

26. Parents of a 3-year-old child with congenital heart disease are afraid to let their child play with other children because of possible overexertion. The nurse's reply should be based on which of the following? 1. Child needs opportunities to play with peers. 2. Child needs to understand that peers' activities are too strenuous. 3. Parents can meet all the child's needs. 4. Constant parental supervision is needed to avoid overexertion.

ANS: 1 1. The child needs opportunities for social development. Children usually limit their activities if allowed to set their own pace.

37. Therapeutic management of the child with rheumatic fever includes: 1. Administration of penicillin. 2. Avoid salicylates (aspirin). 3. Strict bed rest for 4 to 6 weeks. 4. Administration of corticosteroids if chorea develops.

ANS: 1 1. The goal of medical management is the eradication of the hemolytic streptococci. Penicillin is the drug of choice

21. The infant with congestive heart failure has a need for: 1. Increased calories. 2. Increased fluids. 3. Decreased protein. 4. Decreased fat.

ANS: 1 1. The metabolic rate of infants with congestive heart failure is greater because of poor cardiac function and increased heart and respiratory rates. Their caloric needs are greater than those of the average infants, yet their ability to take in the calories is diminished by their fatigue.

25. The patient is scheduled for a colonoscopy at 10 AM today. The effectiveness of the nurse's teaching for this procedure is verified by the following statement by the patient: 1. "I know that the lighted tube he will use will help the doctor look at my tumor and he might take a small piece of tissue to look at in the lab." 2. "I know that light on the tube will help to cure my cancer for me." 3. "I know colonoscopy is very painful and embarrassing and I hope no one sees me in that position." 4. "My daughter is coming in to see me today. I am glad to be looking forward to something pleasant."

ANS: 1 Effective teaching may be evaluated by patient statements referring to correct concepts.

3. Identification of cancer risks is part of every nurse's assessment skills. Some of the most common of these signs and symptoms include: 1. appearance of recent skin area changes that look markedly unlike surrounding tissues. 2. exposure to street repairs by smoothing newly laid concrete. 3. coughs and colds that respond quickly to ampicillin and tea with honey. 4. frequently forgetting monthly breast self-examination.

ANS: 1 Health promotion through self-knowledge and teaching of the public may lead to application measures of early detection and treatment.

Which statement reflects useful information to include in a teaching plan for a cancer patient? 1. Cancer is a group of diseases. The cancer cells are different from the cells in the tissue of origin in growth and the spreading of abnormal cells. 2. Cancer is the third leading cause of death in the United States. Many hospitals have the highest number of patients with this diagnosis. 3. Americans who have a diagnosis of cancer die within a year or less. 4. When a person is genetically predisposed to a type of cancer, there is nothing that can be done to prevent its occurrence.

ANS: 1 Information about the disease and disease process is helpful to allay anxiety as well as to instruct about the pathophysiology.

16. The nurse counsels that the most common site of cancer in adult women is the: 1. breast. 2. lung. 3. kidney. 4. uterus.

ANS: 1 The gender of the person determines the risk for some cancers and the need for early detection

1. The nurse cautions a group of middle-aged persons that conditions that promote the formation of malignant cells are (select all that apply): 1. general emotional health. 2. increasing age. 3. hormonal changes. 4. chronic irritation of tissue. 5. diet.

ANS: 1, 2, 3, 4, 5 All options mentioned are possible sources of increased incidence of cancer in middle-aged persons

3. The nurse explains that adjuvant therapy is given to cancer patients who are free of signs of the disease to (select all that apply): 1. ensure eradication of undetected cells. 2. stabilize normal cells. 3. diminish recurrence of breast cancer. 4. reduce the extent of the tumor before surgery or radiation. 5. change the pH of the system to inhibit cell growth.

ANS: 1, 3 Adjuvant therapy is given to symptom-free cancer victims to eradicate undetected cells and to diminish the recurrence of breast cancer. Administration prior to surgery or radiation to reduce tumor bulk is call neoadjuvant therapy. The therapy does not stabilize normal cells or alter the pH.

2. The nurse reminds a visitor to a patient who has an internal radiation implant to (select all that apply): 1. avoid visitation if you are pregnant. 2. take off all metals, such as your watch and belt. 3. limit visitation time. 4. wear a protective lead apron. 5. stay at least 6 feet away from bedside.

ANS: 1, 3, 5 Visitors are important to reduce the isolation of the radiation patient, but pregnant women should not visit. The visits of all persons should be limited to a few minutes and they should be at least 6 feet from the bedside. Removing metal objects and wearing protective devices are not necessary.

42. One of the most frequent causes of hypovolemic shock in children is which of the following? 1. Sepsis 2. Blood loss 3. Anaphylaxis 4. Congenital heart disease

ANS: 2 2. Blood loss is the most frequent cause of hypovolemic shock in children

22. As part of the treatment for congestive heart failure, the child takes the diuretic furosemide. As part of teaching home care, the nurse encourages the family to give the child foods such as bananas, oranges, and leafy vegetables. These foods are recommended for this child because they are high in which of the following? 1. Chlorides 2. Potassium 3. Sodium 4. Vitamins

ANS: 2 2. Diuretics that work on the proximal and distal renal tubules contribute to increased losses of potassium. The child's diet should be supplemented with this electrolyte

17. An 8-year-old child is receiving digoxin (Lanoxin). The nurse should notify the practitioner and withhold the medication if the apical pulse is less than which of the following? 1. 60 2. 70 3. 90 4. 100

ANS: 2 2. If a 1-minute apical pulse is less than 70 for an older child, the digoxin is withheld. 1. This is the cut-off for holding the digoxin dose in an adult.

25. The nurse is caring for a child with persistent hypoxia secondary to a cardiac defect. The nurse recognizes that a risk exists of cerebrovascular accidents (strokes). Which of the following is an important objective to decrease this risk? 1. Minimize seizures 2. Prevent dehydration 3. Promote cardiac output 4. Reduce energy expenditure

ANS: 2 2. In children with persistent hypoxia, polycythemia develops. Dehydration must be prevented in hypoxemic children because it potentiates the risk of strokes

12. Which of the following defects results in decreased pulmonary blood flow? 1. Atrial septal defect 2. Tetralogy of Fallot 3. Ventricular septal defect 4. Patent ductus arteriosus

ANS: 2 2. Tetralogy of Fallot results in decreased blood flow to the lungs. The pulmonic stenosis increases the pressure in the right ventricle, causing the blood to go from right to left across the ventricular septal defect

27. Which of the following should the nurse consider when preparing a school-age child and the family for heart surgery? 1. Unfamiliar equipment should not be shown. 2. Let child hear the sounds of an ECG monitor. 3. Avoid mentioning postoperative discomfort and interventions. 4. Explain that an endotracheal tube will not be needed if the surgery goes well.

ANS: 2 2. The child and family should be exposed to the sights and sounds of the intensive care unit (ICU). All positive, nonfrightening aspects of the environment are emphasized

24. An 8-month-old infant has a hypercyanotic spell while blood is being drawn. The nurse's first action should be which of the following? 1. Assess for neurologic defects. 2. Place the child in the knee-chest position 3. Begin cardiopulmonary resuscitation. 4. Prepare family for imminent death.

ANS: 2 2. The first action is to place the infant in the knee-chest position. Blow-by oxygen may be indicated.

46. The nurse suspects shock in a child who is 1-day after surgery. The initial nursing action should be which of the following? 1. Obtain blood gases. 2. Administer oxygen. 3. Place on cardiac monitor. 4. Place in Trendelenburg position.

ANS: 2 2. The initial nursing action in shock is to establish ventilatory support. This would be done by the administration of oxygen.

34. The primary nursing intervention to prevent bacterial endocarditis is which of the following? 1. Institute measures to prevent dental procedures. 2. Counsel parents of high-risk children about prophylactic antibiotics. 3. Observe children for complications, such as embolism and heart failure. 4. Encourage restricted mobility in susceptible children.

ANS: 2 2. The objective of nursing care is to counsel the parents of high-risk children about both the need for prophylactic antibiotics for dental procedures and the necessity of maintaining excellent oral health. The child's dentist should be aware of the child's cardiac condition.

40. Which of the following is the leading cause of death after heart transplantation? 1. Infection 2. Rejection 3. Cardiomyopathy 4. Congestive heart failure

ANS: 2 2. The posttransplant course is complex. The leading cause of death after cardiac transplant is rejection.

17. An appropriate cancer warning sign to teach the public would be: 1. intense pain in an area such as a hip or groin after carrying several gallons of paint up a ladder and painting the garage. 2. a mole on the calf of the leg that enlarges over a month and lifts up after 2 weeks of being in the sun at the beach. 3. diarrhea that lasts 2 days after an all-day picnic at the beach. 4. a painful lump under the umbilicus that recedes when pushed, but comes out again with a sneeze or hard cough.

ANS: 2 An obvious change in a wart or mole is an American Cancer Society (ACS) published risk for a cancerous sign.

8. A patient is scheduled for a chemotherapy treatment in about 30 minutes. Breakfast trays have arrived and are being served on the unit. The nurse's best intervention would be to: 1. encourage the patient to eat all his breakfast to keep up his strength to fight the cancer. Remind the patient that breakfast is about one third of daily intake. 2. listen attentively to any concerns that the patient voices regarding the treatment. Offer to hold his tray until after the treatment. 3. offer to call the family to come and be present after the treatment. Encourage the patient to drink at least all of orange juice and coffee. 4. suggest that the patient request a dose of strong analgesic instead of eating, because this treatment is very painful.

ANS: 2 Chemotherapy causes nausea and vomiting. Holding the tray until later provides for better intake and for holding the food in the stomach for digestion.

4. A patient is receiving another course of chemotherapy as his cancer treatment after some previous radiation treatments. He asks about several cancer words, which he has heard referring to him. Correct interpretation of these words for this patient are that: 1. alopecia refers to the darkening of the skin over his cancer area. 2. carcinogen refers to some of the materials in his environment such as cigarettes, asbestos, and mercury. 3. biotherapy refers to special diet foods and specific vitamins that he will need to take on a regular basis now. 4. antineoplastic refers to drugs that increase the spread of his cancer.

ANS: 2 Correct definitions are important when the patient overhears someone using words about himself and his disease.

19. The nurse assesses beginning acceptance of the diagnosis of cancer when the patient: 1. begins to act in a cheerful manner. 2. inquires about support groups. 3. cries over loss of health. 4. actively interacts with his or her family.

ANS: 2 Directed planning for support for the diagnosis is indicative of acceptance. Crying and a cheerful manner are not necessarily positive. Interaction with the family is not indicative of acceptance.

15. An addition that should be made in the nursing care plan when a diagnosis of breast cancer is first made at stage T1 N0 M0 is: 1. "risk for disturbed body image related to threats of anticipated changes." 2. "risk of anxiety related to outcome of treatments." 3. "risk for infection related to decreased white blood cell count." 4. "risk for ineffective coping related to husband's expectations regarding anticipated treatments."

ANS: 2 Early stages of cancer create anxiety about the outcome of treatments for the patient.

2. Helpful sources of information that the nurse may use in beginning to develop a teaching plan for patients having cancer diagnostic testing include: 1. other patients, visitors, or hospital workers who have been associated with someone who has had cancer. 2. analysis of what the patient and family already know about prevention, detection, treatment, and outcomes. 3. American Cancer Society (ACS), National Cancer Institute, Oncology Nursing Society, VFW, or DAV. 4. supermarket tabloids or public books from the local library.

ANS: 2 Effective teaching starts with the level of knowledge of the learner. That content is then analyzed for specific information that needs to be corrected or added.

10. During a gentle bathing, several new bruises are noted on the cancer patient's upper arms and thighs. An understanding of the possible causes and correct nurse's actions for these findings are that: 1. the patient must have fallen last night walking to the bathroom. Teach the patient to use the call bell when assistance is needed. 2. the patient may have disseminated intravascular coagulation. Size, shape, location, color, and tenderness must be reported and recorded fully. 3. an intravascular fluid overload is occurring because of the chemotherapy. Place the patient on strict I/O and limit fluids. 4. the patient must have had a drug-induced seizure, which caused arm and leg thrashing and the bruises. Chart findings and pad the side rails.

ANS: 2 Nurses are the health care providers most often physically closer to the patient. They are frequently better able to assess the skin totally and to monitor the patient's responses to treatments and potential risks. Serious signs need to be reported in a timely manner and fully described

23. The best menu choice for a patient who is undergoing radiation treatments every other day is: 1. a bowl of vegetable soup, chopped egg and pickle sandwich on wheat bread, apple, 8 oz. orange juice. 2. pinto beans, strained, with rice; 1-oz. slice of plain American cheese;1/2 cup spinach; 1/2 ripe banana; 8 oz. milk. 3. Spanish rice, 1/2 cup mixed green salad, cup canned peaches, 8 oz. Coke. 4. spaghetti with tomato sauce, cheddar cheese toast strips, six celery sticks with peanut butter, 8 oz. whole milk.

ANS: 2 The relationship between concepts of proper nutrition, (normal versus during cancer treatments), the special needs of the patient, and adequate nutrition are differentiated to reduce digestive side effects of certain food groups.

24. A patient is close to death with terminal liver cancer. He has widespread metastases. The nurse decides that the patient's frequent call bell summons need to be analyzed and an appropriate action plan implemented. In selecting the best choice, nursing action(s) should be based on which of the following judgments? 1. Encourage and insist that the family requests a transfer to hospice care, because the general hospital does not have enough staff to keep responding to the patient's end-stage frequent calling and requests for minor help. 2. Use fixed interval and cocktail medication administration. Frequently evaluate for breakthrough pain and anxieties. Answer the call bell quickly on the intercom or in person. 3. Tell the family that as of this afternoon, all the patient's questions, comments, and expressed fears of dying and financial worries will be referred to the social worker, physician, or clergy. Otherwise, one of them can come in and sit beside the bed. 4. Plan to limit the time spent with the patient strictly, because there is not much that the nurse can do that could be beneficial at this point.

ANS: 2 Therapeutic touch, the nurse's presence, and sufficient pain medication to make the patient comfortable are appropriate nursing actions.

11. The nurse closely monitors for an oncologic emergency consisting of a blood calcium concentration that is too high. This may be manifested by: 1. hypertension and bradycardia. 2. fatigue, confusion, and weakness. 3. laboratory test results of potassium 2.5 mEq/L, sodium 143 mEq/L 4. urine output less than 30 mL/hr.

ANS: 2 This is the application of previous learned information in a new situation.

1. A chest radiograph film is ordered for a child with suspected cardiac problems. The child's parent asks the nurse, "What will the radiograph show about the heart?" The nurse's response should be based on knowledge that the x-ray film will do which of the following? 1. Show bones of chest but not the heart 2. Measure electrical potential generated from heart muscle 3. Provide permanent record of heart size and configuration 4. Provide computerized image of heart vessels and tissues

ANS: 3 3. A chest radiograph will provide information on the heart size and pulmonary blood-flow patterns. It will be provide a baseline for future comparisons

47. A child is brought to the emergency department experiencing an anaphylactic reaction to a bee sting. While an airway is being established, the nurse should prepare which of the following medications for immediate administration? 1. Diphenhydramine (Benadryl) 2. Dopamine 3. Epinephrine 4. Calcium chloride

ANS: 3 3. After the first priority of establishing an airway, administration of epinephrine is the drug of choice.

30. The nurse is caring for a child after heart surgery. Which of the following should the nurse do if evidence is found of cardiac tamponade? 1. Increase analgesia. 2. Apply warming blankets. 3. Immediately report this to physician. 4. Encourage child to cough, turn, and breathe deeply.

ANS: 3 3. If evidence is noted of cardiac tamponade, blood or fluid in the pericardial space constricting the heart, the physician is notified immediately of this life-threatening complication

29. Which of the following is an important nursing consideration when suctioning a young child who has had heart surgery? 1. Perform suctioning at least every hour. 2. Suction for no longer than 30 seconds at a time. 3. Administer supplemental oxygen before and after suctioning. 4. Expect symptoms of respiratory distress when suctioning.

ANS: 3 3. If suctioning is indicated, supplemental oxygen is administered with a manual resuscitation bag before and after the procedure to prevent hypoxia

18. A 6-month-old infant is receiving digoxin (Lanoxin). The nurse should notify the practitioner and withhold the medication if the apical pulse is less than which of the following? 1. 60 2. 70 3. 90 to 110 4. 110 to 120

ANS: 3 3. If the 1-minute apical is below 90 to 110, the digoxin should not be given to a 6-month-old.

31. Which of the following is an important nursing consideration when chest tubes will be removed from a child? 1. Explain that it is not painful. 2. Explain that only a Band-Aid will be needed 3. Administer analgesics before procedure. 4. Expect bright red drainage for several hours after removal.

ANS: 3 3. It is appropriate to prepare the child for the removal of chest tubes with analgesics. Short-acting medications can be used that are administered through an existing IV line

38. Which of the following actions by the school nurse is important in the prevention of rheumatic fever? 1. Encourage routine cholesterol screenings. 2. Conduct routine blood pressure screenings. 3. Refer children with sore throats for throat cultures. 4. Recommend salicylates instead of acetaminophen for minor discomforts.

ANS: 3 3. Nurses have a role in prevention; primarily in screening school-age children for sore throats caused by group A streptococci. This can be by actively participating in throat culture screening or by referring children with possible streptococcal sore throats for testing.

48. Clinical manifestations of toxic shock syndrome include which of the following? 1. Severe hypertension 2. Subnormal temperature 3. Erythematous macular rash 4. Papular rash over extremities

ANS: 3 3. One of the diagnostic criteria for toxic shock syndrome is a diffuse macular erythroderma.

14. Which of the following is a clinical manifestation of the systemic venous congestion that can occur with congestive heart failure? 1. Tachypnea 2. Tachycardia 3. Peripheral edema 4. Pale, cool extremities

ANS: 3 3. Peripheral edema, especially periorbital edema, is a clinical manifestation of systemic venous congestion

10. Which of the following defects results in increased pulmonary blood flow? 1. Pulmonic stenosis 2. Tricuspid atresia 3. Atrial septal defect 4. Transposition of the great arteries

ANS: 3 3. The atrial septal defect results in increased pulmonary blood flow. Blood flows from the left atrium (higher pressure) into the right atrium (lower pressure) and then to the lungs via the pulmonary artery.

13. Which of the following is best described as the inability of the heart to pump an adequate amount of blood to the systemic circulation at normal filling pressures? 1. Pulmonary congestion 2. Congenital heart defect 3. Congestive heart failure 4. Systemic venous congestion

ANS: 3 3. The definition of congestive heart failure is the inability of the heart to pump an adequate amount of blood to the systemic circulation at normal filling pressures to meet the metabolic demands of the body.

12. A senior female college student has had a melanoma of the forehead surgically removed and given a course of chemotherapy. Which of the following comments that she has made demonstrate her appropriate understanding of the treatments and prognosis? 1. "Why did you bring me this shampoo? You guys took all my hair, so I don't have anything to wash or fix." 2. "Why don't my friends from school come to visit? Did you tell them to stay away?" 3. "My spring dance is only 3 weeks away. Do you think I could find a wig to cover my head where the hair fell out from the chemo?" 4. "Well, this looks like the end of the problem for me, thank goodness! I won't have to bother that doctor again until I graduate in a couple of years because all my shots must be up to date now."

ANS: 3 Acceptance of the diagnosis, treatments, side effects, and prognosis by the patient are important so that the nurse can judge their understanding and acceptance by the patient.

13. A patient whose cancer has been staged at T4 N2 M2 has been assigned for care. What is the best interpretation of this staging information in planning care for this patient? 1. The primary tumor has shrunken, although some lymph nodes remain involved. Teach the patient that this is good news. 2. The primary tumor has now responded to a combination of chemotherapy and radiation. The patient should now receive much less analgesic medication. 3. The primary tumor is quite large and has extended to lymph glands and distant areas. Gentle touch and therapeutic listening will be especially helpful. 4. After the series of radiation treatments, the distant metastases are still present. Prepare the patient to accept only the cure of the primary tumor.

ANS: 3 Correct interpretation and differentiating components of staging data are used to plan effective nursing care.

22. Assisting the health care team to prepare a terminal cancer nursing care plan includes which of the following? 1. Assessment of hyponatremia, hypotension, or cough, all of which must be alleviated by appropriate medications before treatments can begin 2. Preplanning, which includes acceptance by the nurse that it is the patient who is dying (nurse's personal feelings about death are not considered) 3. Acceptance that late-stage cancer needs management of severe continual pain, and acceptance of anxieties related to patient's physical deterioration and to personal grieving 4. Helping the patient or family to contact the social worker (no use at this point)

ANS: 3 Pain and anxiety management are within the expected scope of underlying principles and concepts as nursing implementations for terminal cancer patients.

21. The nurse explains that the most common cytologic test, usually performed in outpatient settings, and that suggests the probability of a need for further testing for cancer cells, is a: 1. chest x-ray. 2. Koch test. 3. Pap test. 4. tine test.

ANS: 3 Preventive testing (Pap test) and screening reduce risks and increase chances of early treatment

7. A patient has a cancer that has been staged as T3 N2 M3. He has a PRN order of morphine, 4 mg, IM q3-4hr. He requests another pain shot about hours after the last one. An appropriate nursing action would be to: 1. inform the patient that this narcotic may be given only every 4 hours to prevent addiction. 2. ignore the call bell for 20 minutes, and then take at least 10 minutes to prepare and administer the injection. 3. give the morphine; evaluate the results of pain relief. Arrange for the physician to evaluate for breakthrough pain. 4. ask the family to assist in helping the patient accept waiting longer to receive an addicting medication such as morphine.

ANS: 3 Terminal care does not include concerns about morphine addiction. Medication may be given 15 minutes before or after an allotted time. The occurrence of breakthrough pain is a real concern for this patient.

9. The nurse explains that drugs such as cannabinoids, Benadryl, and Vistaril are frequently ordered for cancer patients to help: 1. promote amnesia to dampen the fears of dying and loss of financial income. 2. maintain fluid retention to prevent dehydration. 3. control nausea, vomiting, and taste disorders caused by therapy. 4. control bouts of diarrhea or uncomfortable constipation.

ANS: 3 This question requires the demonstration of concept application for specific drugs to patients and expected results.

6. The nurse explains that tumors that originate from tissues in the skin are: 1. carcinomas. 2. lymphomas. 3. melanomas. 4. sarcomas.

ANS: 3 Tumors are classified by anatomic site and tissue of origin. Carcinomas include the skin.

6. After the child returns from cardiac catheterization, the nurse monitors the child's vital signs. The heart rate should be counted for how many seconds? 1. 10 2. 15 3. 30 4. 60

ANS: 4 4. After cardiac catheterization, the heart rate should be counted for a full minute to detect evidence of dysrhythmias or bradycardia.

43. Which of the following types of shock is characterized by a hypersensitivity reaction causing massive vasodilation and capillary leaks, which may occur with drug or latex allergy? 1. Neurogenic shock 2. Cardiogenic shock 3. Hypovolemic shock 4. Anaphylactic shock

ANS: 4 4. Anaphylactic shock results from extreme allergy or hypersensitivity to a foreign substance

35. Which of the following is a common, serious complication of rheumatic fever? 1. Seizures 2. Cardiac arrhythmias 3. Pulmonary hypertension 4. Cardiac valve damage

ANS: 4 4. Cardiac valve damage is the most significant complication of rheumatic fever.

44. Which of the following clinical manifestations would the nurse expect to see as shock progresses in a child and becomes uncompensated shock? 1. Thirst 2. Irritability 3. Apprehension 4. Confusion and somnolence

ANS: 4 4. Confusion and somnolence are beginning signs of uncompensated shock

20. The parents of a young child with congestive heart failure tell the nurse that they are "nervous" about giving digoxin. The nurse's response should be based on which of the following? 1. It is a very safe, frequently used drug. 2. It is difficult to either overmedicate or undermedicate with digoxin. 3. Parents lack the expertise necessary to administer digoxin. 4. Parents must learn specific, important guidelines for administration of digoxin.

ANS: 4 4. Digoxin has a narrow therapeutic range. The margin of safety between therapeutic, toxic, and lethal doses is very small. Specific guidelines are available for parents to learn how to administer the drug safely and to monitor for side effects

39. When discussing hyperlipidemia with a group of adolescents, the nurse should explain that high levels of what are thought to protect against cardiovascular disease? 1. Cholesterol 2. Triglycerides 3. Low-density lipoproteins (LDLs) 4. High-density lipoproteins (HDLs)

ANS: 4 4. HDLs contain very low concentrations of triglycerides, relatively little cholesterol, and high levels of proteins. It is thought that HDLs protect against cardiovascular disease.

7. The nurse is caring for a school-age girl who has had a cardiac catheterization. The child tells the nurse that her bandage is "too wet." The nurse finds the bandage and bed soaked with blood. The most appropriate initial nursing action is which of the following? 1. Notify physician. 2. Apply new bandage with more pressure. 3. Place in Trendelenburg position. 4. Apply direct pressure above catheterization site.

ANS: 4 4. If bleeding occurs, direct continuous pressure is applied 2.5 cm (1 inch) above the percutaneous skin site to localize pressure over the vessel puncture.

28. Seventy-two hours after cardiac surgery, a young child has a temperature of 101° F. The nurse should do which of the following? 1. Keep child warm with blankets. 2. Apply a hypothermia blanket. 3. Record temperature on nurses' notes. 4. Report findings to physician.

ANS: 4 4. In the first 24 to 48 hours after surgery, the body temperature may increase to 37.7° C or 100° F as part of the inflammatory response to tissue trauma. If the temperature is higher or continues after this period, it is most likely a sign of an infection and immediate investigation is indicated

23. Which of the following is a sign of hypokalemia? 1. Apnea 2. Oliguria 3. Twitching 4. Muscle weakness

ANS: 4 4. Muscle weakness is a characteristic clinical manifestation of hypokalemia.

3. José is a 4-year-old child scheduled for a cardiac catheterization. Preoperative teaching should be: 1. Directed at his parents, because he is too young to understand. 2. Detailed in regard to the actual procedures so he will know what to expect. 3. Done several days before the procedure so that he will be prepared. 4. Adapted to his level of development so that he can understand.

ANS: 4 4. Preoperative teaching should always be directed at the child's stage of development. The caregivers also benefit from the same explanations.

32. What is the most common causative agent of bacterial endocarditis? 1. Staphylococcus albus 2. Streptococcus hemolyticus 3. Staphylococcus albicans 4. Streptococcus viridans

ANS: 4 4. Staphylococcus viridans is the most common causative agent in bacterial (infective) endocarditis

4. Which of the following explanations regarding cardiac catheterization is appropriate for a preschool child? 1. Postural drainage will be performed every 4 to 6 hours after the test. 2. It is necessary to be completely "asleep" during the test. 3. The test is very short, usually taking less than 1 hour. 4. When the procedure is done, you will have to keep your leg straight for at least 4 hours.

ANS: 4 4. The child's leg will have to be maintained in a straight position for approximately 4 hours. Younger children can be held in the parent's lap with the leg maintained in the correct position

9. Surgical closure of the ductus arteriosus would do which of the following? 1. Stop the loss of unoxygenated blood to the systemic circulation 2. Decrease the edema in legs and feet 3. Increase the oxygenation of blood 4. Prevent the return of oxygenated blood to the lungs

ANS: 4 4. The ductus arteriosus allows blood to flow from the higher-pressure aorta to the lower-pressure pulmonary artery, causing a right-to-left shunt. If this is surgically closed, no additional oxygenated blood (from the aorta) will return to the lungs through the pulmonary artery

5. The nurse includes in the teaching plan that malignant tumors are similar to benign tumors because both: 1. contain cells that closely resemble those in the tissue of origin. 2. travel quickly to invade and destroy other tissues and organs. 3. always grow and multiply very rapidly, competing for space and nutrients and causing severe pain. 4. may press on nearby surrounding tissues, such as nerves and blood vessels, causing pain.

ANS: 4 This question requires interpretation of primary concepts of differences and similarities of benign and malignant tumor cells.

4. When the nurse sees that an IV vesicant cancer drug has extravasated, the nurse should (select all that apply): 1. place a warm compress on the area. 2. chill the area with an ice pack. 3. raise the patient's arm above the level of the heart. 4. stop the infusion. 5. notify the charge nurse.

ANS: 4, 5 The LPN should stop the infusion and notify the charge nurse so that specially trained personnel can intervene.

The recommendation for calcium for children 1 to 3 years of age is _____ milligrams. (Record your answer in a whole number.)

ANS: 500 While limiting fat consumption, it is important to ensure diets contain adequate nutrients such as calcium. The recommendation for daily calcium intake for children 1 to 3 years of age is 500 mg, and the recommendation for children 4 to 8 years of age is 800 mg.

8. The nurse's approach when introducing hospital equipment to a preschooler should be based on which principle? a. The child may think the equipment is alive. b. The child is too young to understand what the equipment does. c. Explaining the equipment will only increase the child's fear. d. One brief explanation will be enough to reduce the child's fear.

ANS: A

The nurse is preparing to perform a physical assessment on a 10-year-old girl. The nurse gives her the option of her mother either staying in the room or leaving. This action should be considered: a. appropriate because of child's age. b. appropriate because mother would be uncomfortable making decisions for child c. inappropriate because of child's age. d. inappropriate because child is same sex as mother.

ANS: A

17. The nurse is interviewing the mother of an infant. She reports, "I had a difficult delivery, and my baby was born prematurely." This information should be recorded under which of the following headings? a. Past history b. Present illness c. Chief complaint d. Review of systems

ANS: A The past history refers to information that relates to previous aspects of the child's health, not to the current problem. The mother's difficult delivery and prematurity are important parts of the past history of an infant. The history of the present illness is a narrative of the chief complaint from its earliest onset through its progression to the present. Unless the chief complaint is directly related to the prematurity, this information is not included in the history of present illness. The chief complaint is the specific reason for the child's visit to the clinic, office, or hospital. It would not include the birth information. The review of systems is a specific review of each body system. It does not include the premature birth. Sequelae such as pulmonary dysfunction would be included

During a funduscopic examination of a school-age child, the nurse notes a brilliant, uniform red reflex in both eyes. The nurse should recognize that this is a(n): a. normal finding. b. abnormal finding, so child needs referral to ophthalmologist. c. sign of possible visual defect, so child needs vision screening. d. sign of small hemorrhages, which will usually resolve spontaneously.

ANS: A A brilliant, uniform red reflex is an important normal finding. It rules out many serious defects of the cornea, aqueous chamber, lens, and vitreous chamber.

37. The nurse has just started assessing a young child who is febrile and appears very ill. There is hyperextension of the child's head (opisthotonos) with pain on flexion. Which is the most appropriate action? a. Refer for immediate medical evaluation b. Continue assessment to determine cause of neck pain. c. Ask parent when neck was injured. d. Record "head lag" on assessment record, and continue assessment of child.

ANS: A Hyperextension of the child's head with pain on flexion is indicative of meningeal irritation and needs immediate evaluation; it is not descriptive of head lag. The pain is indicative of meningeal irritation. No indication of injury is present

48. A nurse is receiving report on a newborn admitted yesterday after a gastroschisis repair. In the report, the nurse is told the newborn has a physician's prescription for an NG tube to low intermittent suction. The reporting nurse confirms that the NG tube is to low intermittent suction and draining light green stomach contents. Upon initial assessment, the nurse notes that the newborn has pulled the NG tube out. Which is the priority action the nurse should take? a. Replace the NG tube and continue the low intermittent suction. b. Leave the NG tube out and notify the physician at the end of the shift. c. Leave the NG tube out and monitor for bowel sounds. d. Replace the NG tube, but leave to gravity drainage instead of low wall suction.

ANS: A A newborn with a gastroschisis performed the day before will require bowel decompression with an NG tube to low wall intermittent suction. The nurse's priority action is to replace the NG tube and continue with the low wall intermittent suctioning. The NG tube cannot be left out this soon after surgery. The physician's prescription was to have the NG tube to low wall intermittent suction so the tube cannot be placed to gravity drainage.

Which of the following types of seizures may be difficult to detect? a. Absence b. Generalized c. Simple partial d. Complex partial

ANS: A Absence seizures may go unrecognized because little change occurs in the child's behavior during the seizure.

When caring for the child with Reye syndrome, the priority nursing intervention would be which of the following? a. Monitor intake and output. b. Prevent skin breakdown. c. Observe for petechiae. d. Do range-of-motion exercises.

ANS: A Accurate and frequent monitoring of intake and output is essential for adjusting fluid volumes to prevent both dehydration and cerebral edema.

Preschoolers' fears can best be dealt with by which intervention? a. Actively involving them in finding practical methods to deal with the frightening experience b. Forcing them to confront the frightening object or experience in the presence of their parents c. Using logical persuasion to explain away their fears and help them recognize how unrealistic the fears are d. Ridiculing their fears so that they understand that there is no need to be afraid

ANS: A Actively involving them in finding practical methods to deal with the frightening experience is the best way to deal with fears. Forcing a child to confront fears may make the child more afraid. Preconceptual thought prevents logical understanding. Ridiculing fears does not make them go away.

The nurse is caring for an adolescent with osteosarcoma being admitted to undergo chemotherapy. The adolescent had a right above-the-knee amputation 2 months ago and has been experiencing "phantom limb pain." Which prescribed medication is appropriate to administer to relieve phantom limb pain? a. Amitriptyline (Elavil) b. Hydrocodone (Vicodin) c. Oxycodone (OxyContin) d. Alprazolam (Xanax)

ANS: A Amitriptyline (Elavil) has been used successfully to decrease phantom limb pain. Opioids such as Vicodin or OxyContin would not be prescribed for this pain. A benzodiazepine, Xanax, would not be prescribed for this type of pain.

Steven, 16 months old, falls down a few stairs. He gets up and "scolds" the stairs as if they caused him to fall. This is an example of which of the following? a. Animism b. Ritualism c. Irreversibility d. Delayed cognitive development

ANS: A Animism is the attribution of lifelike qualities to inanimate objects. By scolding the stairs, the toddler is attributing human characteristics to them. Ritualism is the need to maintain the sameness and reliability. It provides a sense of comfort to the toddler. Irreversibility is the inability to reverse or undo actions initiated physically. Steven is acting in an age-appropriate manner.

13. A mother calls the clinic nurse about her 4-year-old son who has acute diarrhea. She has been giving him the antidiarrheal drug loperamide (Imodium A-D). The nurse's response should be based on knowledge that this drug is: a. not indicated. b. indicated because it slows intestinal motility. c. indicated because it decreases diarrhea. d. indicated because it decreases fluid and electrolyte losses.

ANS: A Antidiarrheal medications are not recommended for the treatment of acute infectious diarrhea. These medications have adverse effects and toxicity, such as worsening of the diarrhea because of slowing of motility and ileus, or a decrease in diarrhea with continuing fluid losses and dehydration. Antidiarrheal medications are not recommended in infants and small children.

Which can result from the bone demineralization associated with immobility? a. Osteoporosis b. Urinary retention c. Pooling of blood d. Susceptibility to infection

ANS: A Bone demineralization leads to a negative calcium balance, osteoporosis, pathologic fractures, extraosseous bone formation, and renal calculi. Urinary retention is secondary to the effect of immobilization on the urinary tract. Pooling of blood is a result of the cardiovascular effects of immobilization. Susceptibility to infection can result from the effects of immobilization on the respiratory and renal systems.

What clinical manifestations would suggest hydrocephalus in a neonate? a. Bulging fontanel and dilated scalp veins b. Closed fontanel and high-pitched cry c. Constant low-pitched cry and restlessness d. Depressed fontanel and decreased blood pressure

ANS: A Bulging fontanels, dilated scalp veins, and separated sutures are clinical manifestations of hydrocephalus in neonates.

Which accurately describes the speech of the preschool child? a. Dysfluency in speech patterns is normal. b. Sentence structure and grammatic usage are limited. c. By age 5 years, child can be expected to have a vocabulary of about 1000 words. d. Rate of vocabulary acquisition keeps pace with the degree of comprehension of speech.

ANS: A Dysfluency includes stuttering and stammering, a normal characteristic of language development. Children speak in sentences of three or four words at age 3 to 4 years and eight words by age 5 years. At 5 years, children have a vocabulary of 2100 words. Children often gain vocabulary beyond degree of comprehension

1. Which condition in a child should alert a nurse for increased fluid requirements? a. Fever b. Mechanical ventilation c. Congestive heart failure d. Increased intracranial pressure (ICP)

ANS: A Fever leads to great insensible fluid loss in young children because of increased body surface area relative to fluid volume. Respiratory rate influences insensible fluid loss and should be monitored in the mechanically ventilated child. Congestive heart failure is a case of fluid overload in children. Increased ICP does not lead to increased fluid requirements in children.

The most fatal type of burn in the toddler age group is: a. flame burn from playing with matches. b. scald burn from high-temperature tap water. c. hot object burn from cigarettes or irons. d. electric burn from electric outlets.

ANS: A Flame burns from matches and lighters represent one of the most fatal types of burns in the toddler age group. High-temperature tap water, hot objects, and electrical outlets are all significant causes of burn injury. The child should be protected from these causes by reducing the temperature on the hot water in the home, keeping objects such as cigarettes and irons away from children, and placing protective guards over electric outlets when not in use.

An adolescent with osteosarcoma is scheduled for a leg amputation in 2 days. The nurse's approach should include which action? a. Answering questions with straightforward honesty b. Avoiding discussing the seriousness of the condition c. Explaining that, although the amputation is difficult, it will cure the cancer d. Assisting the adolescent in accepting the amputation as better than a long course of chemotherapy

ANS: A Honesty is essential to gain the child's cooperation and trust. The diagnosis of cancer should not be disguised with falsehoods. The adolescent should be prepared for the surgery so he or she has time to reflect on the diagnosis and subsequent treatment. This allows questions to be answered. To accept the need for radical surgery, the child must be aware of the lack of alternatives for treatment. Amputation is necessary, but it will not guarantee a cure. Chemotherapy is an integral part of the therapy with surgery. The child should be informed of the need for chemotherapy and its side effects before surgery.

Which term is used to describe a type of fracture that does not produce a break in the skin? a. Simple b. Compound c. Complicated d. Comminuted

ANS: A If a fracture does not produce a break in the skin, it is called a simple, or closed, fracture. A compound, or open, fracture is one with an open wound through which the bone protrudes. A complicated fracture is one in which the bone fragments damage other organs or tissues. A comminuted fracture occurs when small fragments of bone are broken from the fractured shaft and lie in the surrounding tissue. These are rare in children.

12. A young child is brought to the emergency department with severe dehydration secondary to acute diarrhea and vomiting. Therapeutic management of this child should begin with: a. intravenous (IV) fluids. b. ORS. c. clear liquids, 1 to 2 ounces at a time. d. administration of antidiarrheal medication.

ANS: A In children with severe dehydration, IV fluids are initiated. ORS is acceptable therapy if the dehydration is not severe. Diarrhea is not managed by using clear liquids by mouth. These fluids have a high carbohydrate content, low electrolyte content, and high osmolality. Antidiarrheal medications are not recommended for the treatment of acute infectious diarrhea.

A child with GH deficiency is receiving GH therapy. When is the best time for the GH to be administered? a. At bedtime b. After meals c. Before meals d. On arising in the morning

ANS: A Injections are best given at bedtime to more closely approximate the physiologic release of GH.

Which is the leading cause of death during the toddler period? a. Injuries b. Infectious diseases c. Congenital disorders d. Childhood diseases

ANS: A Injuries are the single most common cause of death in children ages 1 through 4 years. This represents the highest rate of death from injuries of any childhood age group except adolescence. Infectious diseases and childhood diseases are less common causes of deaths in this age group. Congenital disorders are the second leading cause of death in this age group.

3. Which type of dehydration is defined as "dehydration that occurs in conditions in which electrolyte and water deficits are present in approximately balanced proportion"? a. Isotonic dehydration b. Hypotonic dehydration c. Hypertonic dehydration d. All types of dehydration in infants and small children

ANS: A Isotonic dehydration is the correct term for this definition and is the most frequent form of dehydration in children. Hypotonic dehydration occurs when the electrolyte deficit exceeds the water deficit, leaving the serum hypotonic. Hypertonic dehydration results from water loss in excess of electrolyte loss and is usually caused by a proportionately larger loss of water or a larger intake of electrolytes. This definition is specific to isotonic dehydration.

32. A nurse is admitting an infant with biliary atresia. Which is the earliest clinical manifestation of biliary atresia the nurse should expect to assess? a. Jaundice b. Vomiting c. Hepatomegaly d. Absence of stooling

ANS: A Jaundice is the earliest and most striking manifestation of biliary atresia. It is first observed in the sclera, may be present at birth, but is usually not apparent until age 2 to 3 weeks. Vomiting is not associated with biliary atresia. Hepatomegaly and abdominal distention are common but occur later. Stools are large and lighter in color than expected because of the lack of bile.

The nurse is preparing an adolescent with scoliosis for a Luque-rod segmental spinal instrumentation procedure. Which consideration should the nurse include? a. Nasogastric intubation and urinary catheter may be required. b. Ambulation will not be allowed for up to 3 months. c. Surgery eliminates the need for casting and bracing. d. Discomfort can be controlled with nonpharmacologic methods.

ANS: A Luque-rod segmental spinal instrumentation is a surgical procedure. Nasogastric intubation and urinary catheterization may be required. Ambulation is allowed as soon as possible. Depending on the instrumentation used, most patients walk by the second or third postoperative day. Casting and bracing are required postoperatively. The child usually has considerable pain for the first few days after surgery. Intravenous opioids should be administered on a regular basis.

28. The nurse is using calipers to measure skin-fold thickness over the triceps muscle in a school-age child. What is the purpose of doing this? a. To measure body fat b. To measure muscle mass c. To determine arm circumference d. To determine accuracy of weight measurement

ANS: A Measurement of skin-fold thickness is an indicator of body fat. Arm circumference is an indirect measure of muscle mass. The accuracy of weight measurement should be verified with a properly balanced scale. Body fat is just one indicator of weight.

Parents tell the nurse that their toddler daughter eats little at mealtime, only sits at the table with the family briefly, and wants snacks "all the time." Which intervention should the nurse recommend? a. Give her nutritious snacks. b. Offer rewards for eating at mealtimes. c. Avoid snacks so she is hungry at mealtimes. d. Explain to her in a firm manner what is expected of her.

ANS: A Most toddlers exhibit a physiologic anorexia in response to the decreased nutritional requirement associated with the slower growth rate. Parents should help the child develop healthy eating habits. The toddler is often unable to sit through a meal. Frequent nutritious snacks are a good way to ensure proper nutrition. To help with developing healthy eating habits, food should be not be used as positive or negative reinforcement for behavior. The child may develop habits of overeating or eat nonnutritious foods in response.

Which of the following best describes a neuroblastoma? a. Diagnosis is usually made after metastasis occurs. b. Early diagnosis is usually possible because of the obvious clinical manifestations. c. It is the most common brain tumor in young children. d. It is the most common benign tumor in young children.

ANS: A Neuroblastoma is a silent tumor with few symptoms. In more than 70% of cases, diagnosis is made after metastasis occurs, with the first signs caused by involvement in the nonprimary site.

A nurse is conducting a staff in-service on childhood cancers. Which is the primary site of osteosarcoma? a. Femur b. Humerus c. Pelvis d. Tibia

ANS: A Osteosarcoma is the most frequently encountered malignant bone cancer in children. The peak incidence is between ages 10 and 25 years. More than half occur in the femur. After the femur, most of the remaining sites are the humerus, tibia, pelvis, jaw, and phalanges.

45. The nurse is caring for a boy with probable intussusception. He had diarrhea before admission but, while waiting for administration of air pressure to reduce the intussusception, he passes a normal brown stool. Which nursing action is the most appropriate? a. Notify practitioner. b. Measure abdominal girth. c. Auscultate for bowel sounds. d. Take vital signs, including blood pressure.

ANS: A Passage of a normal brown stool indicates that the intussusception has reduced itself. This is immediately reported to the practitioner, who may choose to alter the diagnostic-therapeutic care plan. The first action would be to report the normal stool to the practitioner.

15. Which is a high-fiber food that the nurse should recommend for a child with chronic constipation? a. Popcorn b. Pancakes c. Muffins d. Ripe bananas

ANS: A Popcorn is a high-fiber food. Pancakes and muffins do not have significant fiber unless made with fruit or bran. Raw fruits, especially those with skins and seeds, other than ripe bananas, have high fiber.

Which of the following is the priority nursing intervention when a child is unconscious after a fall? a. Establish adequate airway. b. Perform neurologic assessment. c. Monitor intracranial pressure. d. Determine whether a neck injury is present.

ANS: A Respiratory effectiveness is the primary concern in the care of the unconscious child. Establishment of an adequate airway is always the first priority.

An adolescent boy is brought to the emergency department after a motorcycle accident. His respirations are deep, periodic, and gasping. There are extreme fluctuations in blood pressure. Pupils are dilated and fixed. The nurse should suspect what type of head injury? a. Brainstem b. Skull fracture c. Subdural hemorrhage d. Epidural hemorrhage

ANS: A Signs of brainstem injury include deep, rapid, periodic or intermittent, and gasping respirations. Wide fluctuations or noticeable slowing of the pulse, widening pulse pressure, or extreme fluctuations in blood pressure are consistent with a brainstem injury.

A child is upset because, when the cast is removed from her leg, the skin surface is caked with desquamated skin and sebaceous secretions. Which should the nurse suggest to remove this material? a. Soak in a bathtub. b. Vigorously scrub leg. c. Apply powder to absorb material. d. Carefully pick material off leg.

ANS: A Simple soaking in the bathtub is usually sufficient for the removal of the desquamated skin and sebaceous secretions. It may take several days to eliminate the accumulation completely. The parents and child should be advised not to scrub the leg vigorously or forcibly remove this material because it may cause excoriation and bleeding. Oil or lotion, but not powder, may provide comfort for the child.

A young girl has just injured her ankle at school. In addition to calling the child's parents, the most appropriate, immediate action by the school nurse is to: a. apply ice. b. observe for edema and discoloration. c. encourage child to assume a position of comfort. d. obtain parental permission for administration of acetaminophen or aspirin.

ANS: A Soft-tissue injuries should be iced immediately. In addition to ice, the extremity should be rested, be elevated, and have compression applied. Observing for edema and discoloration, encouraging the child to assume a position of comfort, and obtaining parental permission for administration of acetaminophen or aspirin are not immediate priorities. The application of ice can reduce the severity of the injury.

Parents are concerned that their child is showing aggressive behaviors. Which suggestion should the nurse make to the parents? a. Supervise television viewing. b. Ignore the behavior. c. Punish the child for the behavior. d. Accept the behavior if the child is male.

ANS: A Television is also a significant source for modeling at this impressionable age. Research indicates there is a direct correlation between media exposure, both violent and educational media, and preschoolers exhibiting physical and relational aggression (Ostrov, Gentile, and Crick, 2006). Therefore, parents should be encouraged to supervise television viewing. The behavior should not be ignored because it can escalate to hyperaggression. The child should not be punished because it may reinforce the behavior if the child is seeking attention. For example, children who are ignored by a parent until they hit a sibling or the parent learn that this act garners attention. The behavior should not be accepted from a male child, this is using a "double standard" and aggression should not be equated with masculinity.

A 10-year-old child, without a history of previous seizures, experiences a tonic-clonic seizure at school. Breathing is not impaired, but some postictal confusion occurs. The most appropriate initial action by the school nurse is which of the following? a. Stay with child and have someone call emergency medical service (EMS). b. Notify parent and regular practitioner. c. Notify parent that child should go home. d. Stay with child, offering calm reassurance.

ANS: A The EMS should be called to transport the child because this is the child's first seizure.

Four-year-old David is placed in Buck extension traction for Legg-Calvé-Perthes disease. He is crying with pain as the nurse assesses that the skin of his right foot is pale with an absence of pulse. Which action should the nurse take first? a. Notify the practitioner of the changes noted. b. Give the child medication to relieve the pain. c. Reposition the child and notify physician. d. Chart the observations and check the extremity again in 15 minutes.

ANS: A The absence of a pulse and change in color of the foot must be reported immediately for evaluation by the practitioner. Pain medication should be given after the practitioner is notified. Legg-Calvé-Perthes disease is an emergency condition; immediate reporting is indicated. The findings should be documented with ongoing assessment.

The parents of a 2-year-old tell the nurse that they are concerned because the toddler has started to use "baby talk" since the arrival of their new baby. The nurse should recommend which intervention? a. Ignore the "baby talk." b. Explain to the toddler that "baby talk" is for babies. c. Tell the toddler frequently, "You are a big kid now." d. Encourage the toddler to practice more advanced patterns of speech.

ANS: A The baby talk is a sign of regression in the toddler. It should be ignored, while praising the child for developmentally appropriate behaviors. Regression is children's way of expressing stress. The parents should not introduce new expectations and allow the child to master the developmental tasks without criticism.

Which technique is best for dealing with the negativism of the toddler? a. Offer the child choices. b. Remain serious and intent. c. Provide few or no choices for child. d. Quietly and calmly ask the child to comply.

ANS: A The child should have few opportunities to respond in a negative manner. Questions and requests should provide choices. This allows the child to be in control and reduces opportunities for negativism. The child will continue trying to assert control. The toddler is too young for verbal explanations. The negativism is the child testing limits. These should be clearly defined by structured choices.

A toddler fell out of a second-story window. She had brief loss of consciousness and vomited four times. Since admission, she has been alert and oriented. Her mother asks why a CT scan is required when she "seems fine." The nurse should explain that she: a. may have a brain injury. b. needs this because of her age. c. may start having seizures. d. probably has a skull fracture.

ANS: A The child's history of the fall, brief loss of consciousness, and vomiting four times necessitates evaluation of a potential brain injury. The severity of a head injury may not be apparent on clinical examination but will be detectable on a CT scan.

26. A nurse is admitting a child with Crohn disease. Parents ask the nurse, "How is this disease different from ulcerative colitis?" Which statement should the nurse make when answering this question? a. "With Crohn's the inflammatory process involves the whole GI tract." b. "There is no difference between the two diseases." c. "The inflammation with Crohn's is limited to the colon and rectum." d. "Ulcerative colitis is characterized by skip lesions."

ANS: A The chronic inflammatory process of Crohn disease involves any part of the GI tract from the mouth to the anus but most often affects the terminal ileum. Crohn disease involves all layers of the bowel wall in a discontinuous fashion, meaning that between areas of intact mucosa, there are areas of affected mucosa (skip lesions). The inflammation found with ulcerative colitis is limited to the colon and rectum, with the distal colon and rectum the most severely affected. Inflammation affects the mucosa and submucosa and involves continuous segments along the length of the bowel with varying degrees of ulceration, bleeding, and edema.

28. Bismuth subsalicylate, clarithromycin, and metronidazole are prescribed for a child with a peptic ulcer to: a. eradicate Helicobacter pylori. b. coat gastric mucosa. c. treat epigastric pain. d. reduce gastric acid production.

ANS: A The drug therapy combination of bismuth subsalicylate, clarithromycin, and metronidazole is effective in the treatment of H. pylori and is prescribed to eradicate it.

46. Which is an important nursing consideration in the care of a child with celiac disease? a. Refer to a nutritionist for detailed dietary instructions and education. b. Help child and family understand that diet restrictions are usually only temporary. c. Teach proper hand washing and standard precautions to prevent disease transmission. d. Suggest ways to cope more effectively with stress to minimize symptoms.

ANS: A The main consideration is helping the child adhere to dietary management. Considerable time is spent explaining to the child and parents about the disease process, the specific role of gluten in aggravating the condition, and foods that must be restricted. Referral to a nutritionist would help in this process. The most severe symptoms usually occur in early childhood and adult life. Dietary avoidance of gluten should be lifelong. Celiac disease is not transmissible or stress related.

The parent of a 16-month-old toddler asks, "What is the best way to keep our son from getting into our medicines at home?" The nurse's best advice is: a. "All medicines should be locked securely away." b. "The medicines should be placed in high cabinets." c. "The child just needs to be taught not to touch medicines." d. "Medicines should not be kept in the homes of small children."

ANS: A The major reason for poisoning in the home is improper storage. Toddlers can climb, unlatch cabinets, and obtain access to high-security places. For medications, only a locked cabinet is safe. Toddlers can climb by using furniture. High places are not a deterrent to an exploring toddler. Toddlers are not able to generalize as dangerous all the different forms of medications that may be available in the home. It is not feasible to not keep medicines in the homes of small children. Many parents require medications for chronic illnesses. Parents must be taught safe storage for their home and when they visit other homes.

A nurse places some x-ray contrast the toddler is to drink in a small cup instead of a large cup. Which concept of a toddler's preoperational thinking is the nurse using? a. Inability to conserve b. Magical thinking c. Centration d. Irreversibility

ANS: A The nurse is using the toddler's inability to conserve. This is when the toddler is unable to understand the idea that a mass can be changed in size, shape, volume, or length without losing or adding to the original mass. Instead, toddlers judge what they see by the immediate perceptual clues given to them. A small glass means less amount of contrast. Magical thinking is believing that thoughts are all-powerful and can cause events. Centration is focusing on one aspect rather than considering all possible alternatives. Irreversibility is the inability to undo or reverse the actions initiated, such as being unable to stop doing an action when told.

Which of the following tests is never performed on an awake child? a. Oculovestibular response b. Doll's head maneuver c. Funduscopic examination for papilledema d. Assessment of pyramidal tract lesions

ANS: A The oculovestibular response (caloric test) involves the instillation of ice water into the ear of a comatose child. The caloric test is painful and is never performed on an awake child or one who has a ruptured tympanic membrane.

31. The best chance of survival for a child with cirrhosis is: a. liver transplantation. b. treatment with corticosteroids. c. treatment with immune globulin. d. provision of nutritional support.

ANS: A The only successful treatment for end-stage liver disease and liver failure may be liver transplantation, which has improved the prognosis for many children with cirrhosis. Liver transplantation has revolutionized the approach to cirrhosis. Liver failure and cirrhosis are indications for transplantation. Liver transplantation reflects the failure of other medical and surgical measures to prevent or treat cirrhosis.

The nurse is taking care of an adolescent with osteosarcoma. The parents ask the nurse about treatment. The nurse should make which accurate response about treatment for osteosarcoma? a. Treatment usually consists of surgery and chemotherapy. b. Amputation of affected extremity is rarely necessary. c. Intensive irradiation is the primary treatment. d. Bone marrow transplantation offers the best chance of long-term survival.

ANS: A The optimal therapy for osteosarcoma is a combination of surgery and chemotherapy. Intensive irradiation and bone marrow transplantation are usually not part of the therapeutic management.

The nurse should monitor for which effect on the cardiovascular system when a child is immobilized? a. Venous stasis b. Increased vasopressor mechanism c. Normal distribution of blood volume d. Increased efficiency of orthostatic neurovascular reflexes

ANS: A The physiologic effects of immobilization, as a result of decreased muscle contraction, include venous stasis. This can lead to pulmonary emboli or thrombi. A decreased vasopressor mechanism results in orthostatic hypotension, syncope, hypotension, decreased cerebral blood flow, and tachycardia. An altered distribution of blood volume is found with decreased cardiac workload and exercise tolerance. Immobilization causes a decreased efficiency of orthostatic neurovascular reflexes with an inability to adapt readily to the upright position and with pooling of blood in the extremities in the upright position.

A parent of an 18-month-old boy tells the nurse that he says "no" to everything and has rapid mood swings. If he is scolded, he shows anger and then immediately wants to be held. The nurse's best interpretation of this behavior is included in which statement? a. This is normal behavior for his age. b. This is unusual behavior for his age. c. He is not effectively coping with stress. d. He is showing he needs more attention.

ANS: A Toddlers use distinct behaviors in the quest for autonomy. They express their will with continued negativity and the use of the word "no." Children at this age also have rapid mood swings. The nurse should reassure the parents that their child is engaged in expected behavior for an 18-month-old. Having a rapid mood swing is an expected behavior for a toddler.

50. A child has recurrent abdominal pain (RAP) and a dairy-free diet has been prescribed for 2 weeks. Which explanation is the reason for prescribing a dairy-free diet? a. To rule out lactose intolerance b. To rule out celiac disease c. To rule out sensitivity to high sugar content d. To rule out peptic ulcer disease

ANS: A Treatment for RAP involves providing reassurance and reducing or eliminating symptoms. Dietary modifications may include removal of dairy products to rule out lactose intolerance. Fructose is eliminated to rule out sensitivity to high sugar content and gluten is removed to rule out celiac disease. A dairy-free diet would not rule out peptic ulcer disease.

An important nursing intervention when caring for a child who is experiencing a seizure would be which of the following? a. Describe and record the seizure activity observed. b. Restrain the child when seizure occurs to prevent bodily harm. c. Place a tongue blade between the teeth if they become clenched. d. Suction the child during a seizure to prevent aspiration.

ANS: A When a child is having a seizure, the priority nursing care is observation of the child and seizure. The nurse then describes and records the seizure activity.

38. The nurse is caring for a neonate with a suspected tracheoesophageal fistula. Nursing care should include: a. elevating the head but give nothing by mouth. b. elevating the head for feedings. c. feeding glucose water only. d. avoiding suctioning unless infant is cyanotic.

ANS: A When a newborn is suspected of having a tracheoesophageal fistula, the most desirable position is supine with the head elevated on an inclined plane of at least 30 degrees. It is imperative that any source of aspiration be removed at once; oral feedings are withheld. Feedings should not be given to infants suspected of having tracheoesophageal fistulas. The oral pharynx should be kept clear of secretion by oral suctioning. This is to prevent the cyanosis that is usually the result of laryngospasm caused by overflow of saliva into the larynx.

Which activity is most appropriate for developing fine motor skills in the school-age child? a. Drawing b. Singing c. Soccer d. Swimming

ANS: A Feedback A Activities such as drawing, building models, and playing a musical instrument increase the school-age child's fine motor skills. B Singing is an appropriate activity for the school-age child, but it does not increase fine motor skills. C The school-age child needs to participate in group activities to increase both gross motor skills and social skills, but group activities do not increase fine motor skills. D Swimming is an activity that also increases gross motor skills.

The ability to mentally understand that 1 + 3 = 4 and 4 - 1 = 3 occurs in which stage of cognitive development? a. Concrete operations b. Formal operations c. Intuitive thought d. Preoperations

ANS: A Feedback A By 7 to 8 years of age, the child is able to retrace a process (reversibility) and has the skills necessary for solving mathematical problems. This stage is called concrete operations. B The formal operations stage deals with abstract reasoning and does not occur until adolescence. C Thinking in the intuitive stage is based on immediate perceptions. A child in this stage often solves problems by random guessing. D In preoperational thinking, the child is usually able to add 1 + 3 = 4 but is unable to retrace the process.

An 8-year-old girl tells the nurse that she has cancer because God is punishing her for "being bad." She shares her concern that if she dies, she will go to hell. The nurse should interpret this as being a. A belief common at this age b. A belief that forms the basis for most religions c. Suggestive of excessive family pressure d. Suggestive of a failure to develop a conscience

ANS: A Feedback A Children at this age may view illness or injury as a punishment for a real or imagined mystique. B The belief in divine punishment is common at this age. C The belief in divine punishment is common at this age. D The belief in divine punishment is common at this age.

A school nurse is teaching a health class for 5th grade children. The nurse plans to include which statement to best describe growth in the early school-age period? a. Boys grow faster than girls. b. Puberty occurs earlier in boys than in girls. c. Puberty occurs at the same age for all races and ethnicities. d. It is a period of rapid physical growth.

ANS: A Feedback A During the school-age developmental period, boys are approximately 1 inch taller and 2 pounds heavier than girls. B Puberty occurs 1 1/2 to 2 years later in boys, which is developmentally later than puberty in girls (not unusual in 9- or 10-year-old girls). C Puberty occurs approximately 1 year earlier in African-American girls than in white girls. D Physical growth is slow and steady during the school-age years.

A 14-year-old male seems to be always eating, although his weight is appropriate for his height. The best explanation for this is that a. This is normal because of increase in body mass. b. This is abnormal and suggestive of future obesity. c. His caloric intake would have to be excessive. d. He is substituting food for unfilled needs

ANS: A Feedback A In adolescence, nutritional needs are closely related to the increase in body mass. The peak requirements occur in the years of maximal growth. The caloric and protein requirements are higher than at almost any other time of life. B This describes the expected eating pattern for young adolescents; as long as weight and height are appropriate, obesity is not a concern. C This describes the expected eating pattern for young adolescents; as long as weight and height are appropriate, obesity is not a concern. D This describes the expected eating pattern for young adolescents; as long as weight and height are appropriate, obesity is not a concern.

A 9-year-old girl often comes to the school nurse complaining of stomach pains. Her teacher says she is completing her schoolwork satisfactorily, but lately she has been somewhat aggressive and stubborn in the classroom. The school nurse should recognize this as a. Signs of stress b. Developmental delay c. Physical problem causing emotional stress d. Lack of adjustment to school environment

ANS: A Feedback A Signs of stress include stomach pains or headache, sleep problems, bedwetting, changes in eating habits, aggressive or stubborn behavior, reluctance to participate, or regression to early behaviors. B This child is exhibiting signs of stress. C This child is exhibiting signs of stress. D This child is exhibiting signs of stress.

A nurse is teaching adolescent boys about pubertal changes. The first sign of pubertal change seen with boys is a. Testicular enlargement b. Facial hair c. Scrotal enlargement d. Voice deepens

ANS: A Feedback A The first sign of pubertal changes in boys is testicular enlargement in response to testosterone secretion, which usually occurs in Tanner stage 2. Slight pubic hair is present and the smooth skin texture of the scrotum is somewhat altered. B During Tanner stages 4 and 5, facial hair appears at the corners of the upper lip and chin. C As testosterone secretion increases, the penis, testes, and scrotum enlarge. D During Tanner stages 4 and 5, rising levels of testosterone cause the voice to deepen.

7. A nurse is preparing to assess a 3-year-old child. What communication technique should the nurse use for this child? a. Focus communication on child. b. Explain experiences of others to child. c. Use easy analogies when possible. d. Assure child that communication is private.

ANS: A Because children of this age are able to see things only in terms of themselves, the best approach is to focus communication directly on them. Children should be provided with information about what they can do and how they will feel. With children who are egocentric, experiences of others, analogies, and assurances that the communication is private will not be effective because the child is not capable of understanding

5. Which approach would be best to use to ensure a positive response from a toddler? a. Assume an eye-level position and talk quietly. b. Call the toddler's name while picking him or her up. c. Call the toddler's name and say, "I'm your nurse." d. Stand by the toddler, addressing him or her by name.

ANS: A It is important that the nurse assume a position at the child's level when communicating with the child. By speaking quietly and focusing on the child, the nurse should be able to obtain a positive response. The nurse should engage the child and inform the toddler what is going to occur. If the nurse picks up the child without explanation, the child is most likely going to become upset. The toddler may not understand the meaning of the phrase, "I'm your nurse." If a positive response is desired, the nurse should assume the child's level when speaking if possible.

14. The nurse must assess 10-month-old infant. The infant is sitting on the father's lap and appears to be afraid of the nurse and of what might happen next. Which initial action by the nurse would be most appropriate? a. Initiate a game of peek-a-boo. b. Ask father to place the infant on the examination table. c. Undress the infant while he is still sitting on his father's lap. d. Talk softly to the infant while taking him from his father.

ANS: A Peek-a-boo is an excellent means of initiating communication with infants while maintaining a safe, nonthreatening distance. The child will most likely become upset if separated from his father. As much of the assessment as possible should be done on the father's lap. The nurse should have the father undress the child as needed for the examination

30. Which would be best for the nurse to use when determining the temperature of a preterm infant under a radiant heater? a. Axillary sensor b. Tympanic membrane sensor c. Rectal mercury glass thermometer d. Rectal electronic thermometer

ANS: A The axillary sensor measures the infrared heat energy radiating from the axilla. It can be used on wet skin, in incubators, or under radiant warmers. Ear thermometry does not show sufficient correlation with established methods of measurement. It should not be used when body temperature must be assessed with precision. Mercury thermometers should never be used. The release of mercury, should the thermometer be broken, can cause harmful vapors. Rectal temperatures should be avoided unless no other suitable way exists for the temperature to be measured

The nurse is seeing an adolescent boy and his parents in the clinic for the first time. What should the nurse do first? a. Introduce self. b. Make family comfortable. c. Explain purpose of interview. d. Give assurance of privacy.

ANS: A The first thing that nurses should do is to introduce themselves to the patient and family. Parents and other adults should be addressed with appropriate titles unless they specify a preferred name. During the initial part of the interview, the nurse should include general conversation to help make the family feel at ease. Clarification of the purpose of the interview and the nurse's role is the next thing that should be done. The interview should take place in an environment as free of distraction as possible. In addition, the nurse should clarify which information will be shared with other members of the health care team and any limits to the confidentiality

The nurse is caring for an infant with developmental dysplasia of the hip. Which clinical manifestations should the nurse expect to observe? (Select all that apply.) a. Positive Ortolani click b. Unequal gluteal folds c. Negative Babinski sign d. Trendelenburg sign e. Telescoping of the affected limb f. Lordosis

ANS: A, B A positive Ortolani test and unequal gluteal folds are clinical manifestations of developmental dysplasia of the hip seen from birth to 2 to 3 months. Unequal gluteal folds, negative Babinski sign, and Trendelenburg sign are signs that appear in older infants and children. Telescoping of the affected limb and lordosis are not clinical manifestations of developmental dysplasia of the hip.

An adolescent with juvenile idiopathic arthritis (JIA) is prescribed abatacept (Orencia). Which should the nurse teach the adolescent regarding this medication? (Select all that apply.) a. Avoid receiving live immunizations while taking the medication. b. Before beginning this medication, a tuberculin screening test will be done. c. You will be getting a twice-a-day dose of this medication. d. This medication is taken orally.

ANS: A, B Abatacept reduces inflammation by inhibiting T cells and is given intravenously every 4 weeks. Possible side effects of biologics include an increased infection risk. Because of the infection risk, children should be evaluated for tuberculosis exposure before starting these medications. Live vaccines should be avoided while taking these agents.

A school-age child is diagnosed with systemic lupus erythematosus (SLE). The nurse should plan to implement which interventions for this child? (Select all that apply.) a. Instructions to avoid exposure to sunlight b. Teaching about body changes associated with SLE c. Preparation for home schooling d. Restricted activity

ANS: A, B Key issues for a child with SLE include therapy compliance; body-image problems associated with rash, hair loss, and steroid therapy; school attendance; vocational activities; social relationships; sexual activity; and pregnancy. Specific instructions for avoiding exposure to the sun and UVB light, such as using sunscreens, wearing sun-resistant clothing, and altering outdoor activities, must be provided with great sensitivity to ensure compliance while minimizing the associated feeling of being different from peers. The child should continue school attendance in order to gain interaction with peers and activity should not be restricted, but promoted.

You are working as the triage nurse in a pediatric emergency room. You receive a telephone call from the mother of an adolescent whose front tooth was completely knocked out of his mouth while he was playing soccer. The mother is seeking advice. Which is the appropriate response? Select all that apply. a. Rinse the tooth in lukewarm tap water. b. Place the tooth in saline, milk, or water. c. Scrub the tooth with a disinfectant, such as mouth wash. d. Bring the child to the emergency room within the next hour for the best prognosis.

ANS: A, B Feedback Correct Rinse the tooth in lukewarm tap water—this is a correct response. Place the tooth in saline, milk, or water—this is a correct response. Incorrect The tooth should not be scrubbed, and cleaning agents and disinfectants should be avoided. The prognosis is best if the injury is treated within 30 minutes.

Peer victimization is becoming a significant problem for school-age children and adolescents in the United States. Parents should be educated regarding signs that a child is being bullied. These might include (select all that apply) a. The child spends an inordinate amount of time in the nurse's office. b. Belongings frequently go missing or are damaged. c. The child wants to be driven to school. d. School performance improves. e. The child freely talks about his day.

ANS: A, B, C Feedback Correct Signs that may indicate a child is being bullied are similar to signs of other types of stress and include nonspecific illness or complaints, withdrawal, depression, school refusal, and decreased school performance. Children express fear of going to school or riding the school bus. Very often, children will not talk about what is happening to them. Incorrect These are not indications of bullying.

Which demonstrates the school-age child's developing logic in the stage of concrete operations? Select all that apply. a. The school-age child is able to recognize that 1 lb of feathers is equal to 1 lb of metal. b. The school-age child is able to recognize that he can be a son, brother, or nephew at the same time. c. The school-age child understands the principles of adding, subtracting, and reversibility. d. The school-age child has thinking that is characterized by egocentrism, animism, and centration

ANS: A, B, C Feedback Correct The school-age child understands that the properties of objects do not change when their order, form, or appearance does. Conservation occurs in the concrete operations stage. Comprehension of class inclusion occurs as the school-age child's logic increases. The child begins to understand that a person can be in more than one class at the same time. This is characteristic of concrete thinking and logical reasoning. The school-age child is able to understand principles of adding and subtracting, as well as the process of reversibility, which occurs in the stage of concrete operations. Incorrect This type of thinking occurs in the intuitive thought stage, not the concrete operations stage of development.

Which toys should a nurse provide to promote imaginative play for a 3-year-old hospitalized child? (Select all that apply.) a. Plastic telephone b. Hand puppets c. Jigsaw puzzle (100 pieces) d. Farm animals and equipment e. Jump rope

ANS: A, B, D To promote imaginative play for a 3-year-old child, the nurse should provide: dress-up clothes, dolls, housekeeping toys, dollhouses, play-store toys, telephones, farm animals and equipment, village sets, trains, trucks, cars, planes, hand puppets, or medical kits. A 100-piece jigsaw puzzle and a jump rope would be appropriate for a young, school-age child but not a 3-year-old child.

Which gross motor milestones should the nurse assess in an 18-month-old child? (Select all that apply.) a. Jumps in place with both feet b. Takes a few steps on tiptoe c. Throws ball overhand without falling d. Pulls and pushes toys e. Stands on one foot momentarily

ANS: A, C, D An 18-month-old child can jump in place with both feet, throw a ball overhand without falling, and pull and push toys. Taking a few steps on tiptoe and standing on one foot momentarily is not acquired until 30 months of age.

Which interventions are appropriate for preventing childhood obesity? Select all that apply. a. Establish consistent times for meals and snacks. b. Eliminate all snacks. Eat three nutritious meals a day. c. Teach the family and child how to select foods and prepare foods. d. Encourage schools to provide snack machines with popcorn, cookies, and diet soda. e. Limit computer and television time.

ANS: A, C, E Feedback Correct Preventing obesity includes encouraging families to establish consistent times for meals and snacks and discouraging between-meal eating. Parents and children also need to be taught how to select and prepare healthful foods. Because snacks are an important aspect in childhood nutrition, nutritious snacks should be identified. School-age children usually require a healthful snack after school and in the evening. A child who spends time with social media has less interest in physical activity and going outdoors. Incorrect Snacks are an important aspect in childhood nutrition. Nutritious snacks should be identified, not eliminated. Healthy snack options include fruit, popcorn, nuts, and yogurt, not cookies and diet soda. In schools with snack machines, children may use their lunch money to purchase high-calorie snacks versus a nutritious lunch.

The nurse is caring for an infant with a suspected urinary tract infection. Which clinical manifestations should be expected? (Select all that apply.) a. Vomiting b. Jaundice c. Failure to gain weight d. Swelling of the face e. Back pain f. Persistent diaper rash

ANS: A, C, F Vomiting, failure to gain weight, and persistent diaper rash are clinical manifestations observed in an infant with a UTI.

A school-age child is admitted to the hospital with acute glomerulonephritis and oliguria. Which dietary menu items should be allowed for this child? (Select all that apply.) a. Apples b. Bananas c. Cheese d. Carrot sticks e. Strawberries

ANS: A, D, E Moderate sodium restriction and even fluid restriction may be instituted for children with acute glomerulonephritis. Foods with substantial amounts of potassium are generally restricted during the period of oliguria. Apples, carrot sticks, and strawberries would be items low in sodium and allowed. Bananas are high in potassium and cheese is high in sodium. Those items would be restricted.

3. A nurse is planning preoperative care for a newborn with tracheoesophageal fistula (TEF). Which interventions should the nurse plan to implement? (Select all that apply.) a. Positioning with head elevated on a 30-degree plane b. Feedings through a gastrostomy tube c. Nasogastric tube to continuous low wall suction d. Suctioning with a Replogle tube passed orally to the end of the pouch e. Gastrostomy tube to gravity drainage

ANS: A, D, E The most desirable position for a newborn who has TEF is supine (or sometimes prone) with the head elevated on an inclined plane of at least 30 degrees. This positioning minimizes the reflux of gastric secretions at the distal esophagus into the trachea and bronchi, especially when intraabdominal pressure is elevated. It is imperative to immediately remove any secretions that can be aspirated. Until surgery, the blind pouch is kept empty by intermittent or continuous suction through an indwelling double-lumen or Replogle catheter passed orally or nasally to the end of the pouch. In some cases, a percutaneous gastrostomy tube is inserted and left open so that any air entering the stomach through the fistula can escape, thus minimizing the danger of gastric contents being regurgitated into the trachea. The gastrostomy tube is emptied by gravity drainage. Feedings through the gastrostomy tube and irrigations with fluid are contraindicated before surgery in an infant with a distal TEF. A nasogastric tube to low intermittent suctioning could not be accomplished because the esophagus ends in a blind pouch in TEF.

1. A child who has just had definitive repair of a high rectal malformation is to be discharged. Which should the nurse address in the discharge preparation of this family? (Select all that apply.) a. Perineal and wound care b. Necessity of firm stools to keep suture line clean c. Bowel training beginning as soon as child returns home d. Reporting any changes in stooling patterns to practitioner e. Use of diet modification to prevent constipation

ANS: A, D, E Wound care instruction is necessary in a child who is being discharged after surgery. The parents are taught to notify the practitioner if any signs of an anal stricture or other complications develop. Constipation is avoided, since a firm stool will place strain on the suture line. Fiber and stool softeners are often given to keep stools soft and avoid tension on the suture line. The child needs to recover from the surgical procedure. Then bowel training may begin, depending on the child's developmental and physiologic readiness.

Injuries claim many lives during adolescence. Which factors contribute to early adolescents engaging in risk-taking behaviors? Select all that apply. a. Peer pressure b. A desire to master their environment c. Engagement in the process of separation from their parents d. A belief that they are invulnerable e. Impulsivity

ANS: A, D, E Feedback Correct Peer pressure (including impressing peers) is a factor contributing to adolescent injuries. During early to middle adolescence, children feel that they are exempt from the consequences of risk-taking behaviors; they believe that negative consequences only happen to others. Feelings of invulnerability ("It can't happen to me") are evident in adolescence. Impulsivity places adolescents in unsafe situations. Incorrect Mastering the environment is the task of young school-age children. Emancipation is a major issue for the older adolescent. The process is accomplished as the teenager gains an education or vocational training

An infant with hydrocephalus is hospitalized for surgical placement of a ventriculoperitoneal shunt. Which of the following interventions should be included in the child's postoperative care? (Select all that apply.) a. Observe closely for signs of infection. b. Pump the shunt reservoir to maintain patency. c. Administer sedation to decrease irritability. d. Maintain Trendelenburg position to decrease pressure on the shunt. e. Maintain an accurate record of intake and output. f. Monitor for abdominal distention.

ANS: A, E, F Infection is a major complication of ventriculoperitoneal shunts. Observation for signs of infection is a priority nursing intervention. Intake and output should be measured carefully. Abdominal distention could be a sign of peritonitis or a postoperative ileus.

The nurse is taking a sexual history on an adolescent girl. Which is the best way to determine whether she is sexually active? a. Ask her, "Are you sexually active?" b. Ask her, "Are you having sex with anyone?" c. Ask her, "Are you having sex with a boyfriend?" d. Ask both the girl and her parent whether she is sexually active.

ANS: B Asking the adolescent girl whether she is having sex with anyone is a direct question that is well understood. The phrase sexually active is broadly defined and may not provide specific information to the nurse to provide necessary care. The word anyone is preferred to using gender-specific terms such as boyfriend or girlfriend. Because homosexual experimentation may occur, it is preferable to use gender-neutral terms. Questioning about sexual activity should occur when the adolescent is alone

40. Parents of a newborn are concerned because the infant's eyes often "look crossed" when the infant is looking at an object. The nurse's response is that this is normal based on the knowledge that binocularity is normally present by what age? a. 1 month b. 3 to 4 months c. 6 to 8 months d. 12 months

ANS: B Binocularity is usually achieved by ages 3 to 4 months. 1 month is too young. If binocularity is not achieved by ages 6 to 12 months, the child must be observed for strabismus

3. Which communication technique should the nurse avoid when interviewing children and their families? a. Using silence b. Using clichés c. Directing the focus d. Defining the problem

ANS: B Using stereotyped comments or clichés can block effective communication, and this technique should be avoided. After use of such trite phrases, parents will often not respond. Silence can be an effective interviewing tool. Silence permits the interviewee to sort out thoughts and feelings and search for responses to questions. To be effective, the nurse must be able to direct the focus of the interview while allowing maximal freedom of expression. By using open-ended questions, along with guiding questions, the nurse can obtain the necessary information and maintain the relationship with the family. The nurse and parent must collaborate and define the problem that will be the focus of the nursing intervention

In providing nourishment for a child with cystic fibrosis (CF), which factor should the nurse keep in mind?

Diet should be high in carbohydrates and protein.

The nurse is caring for a hospitalized 4-year-old boy. His parents tell the nurse that they will be back to visit at 6 PM. When the child asks the nurse when his parents are coming, the nurse's best response is a. "They will be here soon." b. "They will come after dinner." c. "Let me show you on the clock when 6 PM is." d. "I will tell you every time I see you how much longer it will be."

ANS: B A 4-year-old child understands time in relation to events such as meals. Children perceive "soon" as a very short time. The nurse may lose the child's trust if his parents do not return in the time he perceives as "soon." Children cannot read or use a clock for practical purposes until age 7 years. I will tell you every time I see you how much longer it will be assumes the child understands the concepts of hours and minutes, which are not developed until age 5 or 6 years.

A school-age child has sustained a head injury and multiple fractures after being thrown from a horse. The child's level of consciousness is variable. The parents tell the nurse that they think their child is in pain because of periodic crying and restlessness. The most appropriate nursing action is which of the following? a. Discuss with parents the child's previous experiences with pain. b. Discuss with practitioner what analgesia can be safely administered. c. Explain that analgesia is contraindicated with a head injury. d. Explain that analgesia is unnecessary when child is not fully awake and alert.

ANS: B A key nursing role is to provide sedation and analgesia for the child. Consultation with the appropriate practitioner is necessary to avoid conflict between the necessity to monitor the child's neurologic status and the promotion of comfort and relief of anxiety.

The parent of a 4-year-old boy tells the nurse that the child believes that monsters and boogeymen are in his bedroom at night. The nurse's best suggestion for coping with this problem is to: a. let the child sleep with his parents. b. keep a night-light on in the child's bedroom. c. help the child understand that these fears are illogical. d. tell the child frequently that monsters and boogeymen do not exist.

ANS: B A night-light shows a child that imaginary creatures do not lurk in the darkness. Letting the child sleep with parents will not get rid of the fears. A 4-year-old child is in the preconceptual age and cannot understand logical thought.

Which of the following statements best describes a subdural hematoma? a. Bleeding occurs between the dura and the skull. b. Bleeding occurs between the dura and the cerebrum. c. Bleeding is generally arterial, and brain compression occurs rapidly. d. The hematoma commonly occurs in the parietotemporal region.

ANS: B A subdural hematoma is bleeding that occurs between the dura and the cerebrum as a result of a rupture of cortical veins that bridge the subdural space.

The nurse is conducting a staff in-service on casts. Which is an advantage to using a fiberglass cast instead of a plaster of Paris cast? a. Cheaper b. Dries rapidly c. Molds closely to body parts d. Smooth exterior

ANS: B A synthetic casting material dries in 5 to 30 minutes as compared with a plaster cast, which takes 10 to 72 hours to dry. Synthetic casts are more expensive and have a rough exterior, which may scratch surfaces. Plaster casts mold closer to body parts.

6. Parents call the clinic and report that their toddler has had acute diarrhea for 24 hours. The nurse should further ask the parents if the toddler has which associated factor that is causing the acute diarrhea? a. Celiac disease b. Antibiotic therapy c. Immunodeficiency d. Protein malnutrition

ANS: B Acute diarrhea is a sudden increase in frequency and change in consistency of stools and may be associated with antibiotic therapy. Celiac disease is a problem with gluten intolerance and may cause chronic diarrhea if not identified and managed appropriately. Immunodeficiency would occur with chronic diarrhea. Protein malnutrition or kwashiorkor causes chronic diarrhea from lowered resistance to infection.

49. Parents of a child undergoing an endoscopy to rule out peptic ulcer disease (PUD) from H. pylori ask the nurse, "If H. pylori is found will my child need another endoscopy to know that it is gone?" Which is the nurse's best response? a. "Yes, the only way to know the H. pylori has been eradicated is with another endoscopy." b. "We can collect a stool sample and confirm that the H. pylori has been eradicated." c. "A blood test can be done to determine that the H. pylori is no longer present." d. "Your child will always test positive for H. pylori because after treatment it goes into remission, but can't be completely eradicated."

ANS: B An upper endoscopy is the procedure initially performed to diagnose PUD. A biopsy can determine the presence of H. pylori. Polyclonal and monoclonal stool antigen tests are an accurate, noninvasive method to confirm H. pylori has been eradicated after treatment. A blood test can identify the presence of the antigen to this organism, but because H. pylori was already present it would not be as accurate as a stool sample to determine whether it has been eradicated. H. pylori can be treated and, once the treatment is complete, the stool sample can determine that it was eradicated.

Which is descriptive of a toddler's cognitive development at age 20 months? a. Searches for an object only if he or she sees it being hidden b. Realizes that "out of sight" is not out of reach c. Puts objects into a container but cannot take them out d. Understands the passage of time, such as "just a minute" and "in an hour"

ANS: B At this age, the child is in the final sensorimotor stage. Children will now search for an object in several potential places, even though they saw only the original hiding place. Children have a more developed sense of objective permanence. They will search for objects even if they have not seen them hidden. When a child puts objects into a container but cannot take them out, this is indicative of tertiary circular reactions. An embryonic sense of time exists, although the children may behave appropriately to time-oriented phrases; their sense of timing is exaggerated.

A neonate born with ambiguous genitalia is diagnosed with congenital adrenogenital hyperplasia. Therapeutic management includes administration of: a. vitamin D. b. cortisone. c. stool softeners. d. calcium carbonate.

ANS: B Cortisone is administered to suppress the abnormally high secretions of adrenocorticotropic hormone (ACTH). This in turn inhibits the secretion of adrenocorticosteroid, which stems the progressive virilization.

Kristin, age 10 years, sustained a fracture in the epiphyseal plate of her right fibula when she fell off of a tree. When discussing this injury with her parents, the nurse should consider which statement? a. Healing is usually delayed in this type of fracture. b. Growth can be affected by this type of fracture. c. This is an unusual fracture site in young children. d. This type of fracture is inconsistent with a fall.

ANS: B Detection of epiphyseal injuries is sometimes difficult, but fractures involving the epiphysis or epiphyseal plate present special problems in determining whether bone growth will be affected. Healing of epiphyseal injuries is usually prompt. The epiphysis is the weakest point of the long bones. This is a frequent site of damage during trauma.

47. An infant with short bowel syndrome will be discharged home on total parenteral nutrition (TPN) and gastrostomy feedings. Nursing care should include: a. preparing family for impending death. b. teaching family signs of central venous catheter infection. c. teaching family how to calculate caloric needs. d. securing TPN and gastrostomy tubing under diaper to lessen risk of dislodgment.

ANS: B During TPN therapy, care must be taken to minimize the risk of complications related to the central venous access device, such as catheter infections, occlusions, or accidental removal. This is an important part of family teaching. The prognosis for patients with short bowel syndrome depends in part on the length of residual small intestine. It has improved with advances in TPN. Although parents need to be taught about nutritional needs, the caloric needs and prescribed TPN and rate are the responsibility of the health care team. The tubes should not be placed under the diaper due to risk of infection.

Which should the nurse expect for a toddler's language development at age 18 months? a. Vocabulary of 25 words b. Increasing level of comprehension c. Use of holophrases d. Approximately one third of speech understandable

ANS: B During the second year of life, level of comprehension and understanding of speech increases and is far greater than the child's vocabulary. This is also true for bilingual children, who are able to achieve this linguistic milestone in both languages. The 18-month-old child has a vocabulary of 10 or more words. At this age, the child does not use the one-word sentences that are characteristic of the 1-year-old child. The child has a limited vocabulary of single words that are comprehensible.

Which of the following is a condition that can result if hypersecretion of GH occurs after epiphyseal closure? a. Dwarfism b. Acromegaly c. Gigantism d. Cretinism

ANS: B Excess GH after closure of the epiphyseal plates results in acromegaly.

Exophthalmos (protruding eyeballs) may occur in children with which of the following conditions? a. Hypothyroidism b. Hyperthyroidism c. Hypoparathyroidism d. Hyperparathyroidism

ANS: B Exophthalmos is a clinical manifestation of hyperthyroidism.

The nurse is preparing a school-age child for computed tomography (CT scan) to assess cerebral function. The nurse should include which of the following statements in preparing the child? a. "Pain medication will be given." b. "The scan will not hurt." c. "You will be able to move once the equipment is in place." d. "Unfortunately no one can remain in the room with you during the test."

ANS: B For CT scans, the child must be immobilized. It is important to emphasize to the child that at no time is the procedure painful.

33. A newborn was admitted to the nursery with a complete bilateral cleft lip and palate. The physician explained the plan of therapy and its expected good results. However, the mother refuses to see or hold her baby. Initial therapeutic approach to the mother should be: a. restating what the physician has told her about plastic surgery. b. encouraging her to express her feelings. c. emphasizing the normalcy of her baby and the baby's need for mothering. d. recognizing that negative feelings toward the child continue throughout childhood.

ANS: B For parents, cleft lip and cleft palate deformities are particularly disturbing. The nurse must place emphasize not only the infant's physical needs but also the parents' emotional needs. The mother needs to be able to express her feelings before she can accept her child. Although the nurse will restate what the physician has told the mother about plastic surgery, it is not part of the initial therapeutic approach. As the mother expresses her feelings, the nurse's actions should convey to the parents that the infant is a precious human being. The nurse emphasizes the child's normalcy and helps the mother recognize the child's uniqueness. Maternal-infant attachment was not negatively affected at age 1 year.

The most effective way to clean a toddler's teeth is for the: a. child to brush regularly with a toothpaste of his or her choice. b. parent to stabilize the chin with one hand and brush with the other. c. parent to brush the mandibular occlusive surfaces, leaving the rest for the child. d. parent to brush the front labial surfaces, leaving the rest for the child.

ANS: B For young children, the most effective cleaning of teeth is by the parents. Different positions can be used if the child's back is to the adult. The adult should use one hand to stabilize the chin and the other to brush the child's teeth. The child can participate in brushing, but for a thorough cleaning, adult intervention is necessary.

40. Pyloric stenosis can best be described as: a. dilation of the pylorus. b. hypertrophy of the pyloric muscle. c. hypotonicity of the pyloric muscle. d. reduction of tone in the pyloric muscle.

ANS: B Hypertrophic pyloric stenosis occurs when the circumferential muscle of the pyloric sphincter becomes thickened, resulting in elongation and narrowing of the pyloric channel. Dilation of the pylorus, hypotonicity of the pyloric muscle, and reduction of tone in the pyloric muscle are not the definition of pyloric stenosis.

The nurse is caring for a preschool child immobilized by a spica cast. Which effect on metabolism should the nurse monitor on this child related to the immobilized status? a. Hypocalcemia b. Decreased metabolic rate c. Positive nitrogen balance d. Increased production of stress hormones

ANS: B Immobilization causes a decreased metabolic rate with slowing of all systems and a decreased food intake. Immobilization leads to hypercalcemia and causes a negative nitrogen balance secondary to muscle atrophy. A decreased production of stress hormones occurs with decreased physical and emotional coping capacity.

A nurse, instructing parents of a hospitalized preschool child, explains that which is descriptive of the preschooler's understanding of time? a. Has no understanding of time b. Associates time with events c. Can tell time on a clock d. Uses terms like "yesterday" appropriately

ANS: B In a preschooler's understanding, time has a relation with events such as "We'll go outside after lunch." Preschoolers develop an abstract sense of time at age 3 years. Children can tell time on a clock at age 7 years. Children do not fully understand use of time-oriented words until age 6 years.

17. Enemas are ordered to empty the bowel preoperatively for a child with Hirschsprung disease. The enema solution should be: a. tap water. b. normal saline. c. oil retention. d. phosphate preparation.

ANS: B Isotonic solutions should be used in children. Saline is the solution of choice. Plain water is not used. This is a hypotonic solution and can cause rapid fluid shift, resulting in fluid overload. Oil-retention enemas will not achieve the "until clear" result. Phosphate enemas are not advised for children because of the harsh action of the ingredients. The osmotic effects of the phosphate enema can result in diarrhea, which can lead to metabolic acidosis.

Kimberly's parents have been using a rearward-facing, convertible car seat since she was born. Most car seats can be safely switched to the forward-facing position when the child reaches which age? a. 1 b. 2 c. 3 d. 4

ANS: B It is now recommended that all infants and toddlers ride in rear-facing car safety seats until they reach the age of 2 years or height recommended by the car seat manufacturer. Children 2 years old and older who have outgrown the rear-facing height or weight limit for their car safety seat should use a forward-facing car safety seat with a harness up to the maximum height or weight recommended by the manufacturer. One year is too young to switch to a forward-facing position.

Five-year-old José is being prepared for surgery to remove a brain tumor. Nursing actions should be based on which of the following? a. Removal of tumor will stop the various symptoms. b. Usually the postoperative dressing covers the entire scalp. c. He is not old enough to be concerned about his head being shaved. d. He is not old enough to understand the significance of the brain.

ANS: B José should be told what he will look and feel like after surgery. This includes the size of the dressing. The nurse can demonstrate on a doll the expected size and shape of the dressing.

When taking the history of a child hospitalized with Reye syndrome, the nurse should not be surprised that a week ago the child had recovered from which of the following? a. Measles b. Varicella c. Meningitis d. Hepatitis

ANS: B Most cases of Reye syndrome follow a common viral illness such as varicella or influenza.

Two toddlers are playing in a sandbox when one child suddenly grabs a toy from the other child. Which is the best interpretation of this behavior? a. This is typical behavior because toddlers are aggressive. b. This is typical behavior because toddlers are egocentric. c. Toddlers should know that sharing toys is expected of them. d. Toddlers should have the cognitive ability to know right from wrong.

ANS: B Play develops from the solitary play of infancy to the parallel play of toddlers. The toddler plays alongside other children, not with them. This typical behavior of the toddler is not intentionally aggressive. Shared play is not within their cognitive development. Toddlers do not conceptualize shared play. Because the toddler cannot view the situation from the perspective of the other child, it is okay to take the toy. Therefore, no right or wrong is associated with taking a toy.

Which play item should the nurse bring from the playroom to a hospitalized toddler in isolation? a. Small plastic Lego b. Set of large plastic building blocks c. Brightly colored balloon d. Coloring book and crayons

ANS: B Play objects for toddlers must still be chosen with an awareness of danger from small parts. Large, sturdy toys without sharp edges or removable parts are safest. Large plastic blocks are appropriate for a toddler in isolation. Small plastic toys such as Lego can cause choking or can be aspirated. Balloons can cause significant harm if swallowed or aspirated. Coloring book and crayons would be too advanced for a toddler.

A 4-year-old child is hospitalized with a serious bacterial infection. The child tells the nurse that he is sick because he was "bad." Which is the nurse's best interpretation of this comment? a. Sign of stress b. Common at this age c. Suggestive of maladaptation d. Suggestive of excessive discipline at home

ANS: B Preschoolers cannot understand the cause and effect of illness. Their egocentrism makes them think they are directly responsible for events, making them feel guilty for things outside their control. Children of this age show stress by regressing developmentally or acting out. Maladaptation is unlikely. Telling the nurse that he is sick because he was "bad" does not imply excessive discipline at home.

Which is an appropriate recommendation for preventing tooth decay in young children? a. Substitute raisins for candy. b. Substitute sugarless gum for regular gum. c. Use honey or molasses instead of refined sugar. d. When sweets are to be eaten, select a time not during meals.

ANS: B Regular gum has high sugar content. When the child chews gum, the sugar is in prolonged contact with the teeth. Sugarless gum is less cariogenic than regular gum. Raisins, honey, and molasses are highly cariogenic and should be avoided. Sweets should be consumed with meals so that the teeth can be cleaned afterward. This decreases the amount of time that the sugar is in contact with the teeth.

20. A 4-month-old infant has gastroesophageal reflux (GER) but is thriving without other complications. Which should the nurse suggest to minimize reflux? a. Place in Trendelenburg position after eating. b. Thicken formula with rice cereal. c. Give continuous nasogastric tube feedings. d. Give larger, less frequent feedings.

ANS: B Small, frequent feedings of formula combined with 1 teaspoon to 1 tablespoon of rice cereal per ounce of formula has been recommended. Milk-thickening agents have been shown to decrease the number of episodes of vomiting and to increase the caloric density of the formula. This may benefit infants who are underweight as a result of GER disease. Placing the child in a Trendelenburg position would increase the reflux. Continuous nasogastric feedings are reserved for infants with severe reflux and failure to thrive.

The parents of a neonate with adrenogenital hyperplasia tell the nurse that they will be afraid to have any more children. The nurse should explain that: a. it is not hereditary. b. genetic counseling is indicated. c. it can be prevented during pregnancy. d. all future children will have the disorder.

ANS: B Some forms of adrenogenital hyperplasia are hereditary and should be referred for genetic counseling. Affected offspring should also be referred for genetic counseling.

A nurse is teaching parents about language development for preschool children. Which dysfunctional speech pattern is a normal characteristic the parents might expect? a. Lisp b. Stammering c. Echolalia d. Repetition without meaning

ANS: B Stammering and stuttering are normal dysfluency patterns in preschool-age children. Lisps are not a normal characteristic of language development. Echolalia and repetition are traits of toddlers' language

Which of the following terms is used when a patient remains in a deep sleep, responsive only to vigorous and repeated stimulation? a. Coma b. Stupor c. Obtundation d. Persistent vegetative state

ANS: B Stupor exists when the child remains in a deep sleep, responsive only to vigorous and repeated stimulation.

The Glasgow Coma Scale consists of an assessment of: a. pupil reactivity and motor response. b. eye opening and verbal and motor responses. c. level of consciousness and verbal response. d. ICP and level of consciousness.

ANS: B The Glasgow Coma Scale assesses eye opening and verbal and motor responses.

24. The nurse is caring for a child admitted with acute abdominal pain and possible appendicitis. Which is appropriate to relieve the abdominal discomfort? a. Place in Trendelenburg position. b. Allow to assume position of comfort. c. Apply moist heat to the abdomen. d. Administer a saline enema to cleanse bowel.

ANS: B The child should be allowed to take a position of comfort, usually with the legs flexed. The Trendelenburg position will not help with the discomfort. In any instance in which appendicitis is a possibility, there is a danger in administering a laxative or enemas or applying heat to the area. Such measures stimulate bowel motility and increase the risk of perforation.

35. A mother who intended to breastfeed has given birth to an infant with a cleft palate. Nursing interventions should include: a. giving medication to suppress lactation. b. encouraging and helping mother to breastfeed. c. teaching mother to feed breast milk by gavage. d. recommending use of a breast pump to maintain lactation until infant can suck.

ANS: B The mother who wishes to breastfeed may need encouragement and support because the defect does present some logistical issues. The nipple must be positioned and stabilized well back in the infant's oral cavity so that the tongue action facilitates milk expression. Because breastfeeding is an option, if the mother wishes to breastfeed, medications should not be given to suppress lactation. Because breastfeeding can usually be accomplished, gavage feedings are not indicated. The suction required to stimulate milk, absent initially, may be useful before nursing to stimulate the let-down reflex.

18. Which is most important to document about immunizations in the child's health history? a. Dosage of immunizations received b. Occurrence of any reaction after an immunization c. The exact date the immunizations were received d. Practitioner who administered the immunizations

ANS: B The occurrence of any reaction after an immunization was given is the most important to document in a history because of possible future reactions, especially allergic reactions. Exact dosage of the immunization received may not be recorded on the immunization record. Exact dates are important to obtain but not as important as a history of reaction to an immunization. The practitioner who administered the immunization does not need to be recorded in the health history. A potentially severe physiologic response is the most threatening and most important information to document for safety reasons

The nurse is closely monitoring a child who is unconscious after a fall and notices that the child suddenly has a fixed and dilated pupil. The nurse should interpret this as which of the following? a. Eye trauma b. Neurosurgical emergency c. Severe brainstem damage d. Indication of brain death

ANS: B The sudden appearance of a fixed and dilated pupil(s) is a neurosurgical emergency. The nurse should immediately report this finding.

36. The nurse is caring for an infant whose cleft lip was repaired. Important aspects of this infant's postoperative care include: a. arm restraints, postural drainage, mouth irrigations. b. cleansing the suture line, supine and side-lying positions, arm restraints. c. mouth irrigations, prone position, cleansing suture line. d. supine and side-lying positions, postural drainage, arm restraints.

ANS: B The suture line should be cleansed gently after feeding. The child should be positioned on the back, on the side, or in an infant seat. Elbows are restrained to prevent the child from accessing the operative site. Postural drainage is not indicated. This would increase the pressure on the operative site when the child is placed in different positions. There is no reason to perform mouth irrigations, and the child should not be placed in the prone position where injury to the suture site can occur.

Parents tell the nurse that they found their 3-year-old daughter and a male cousin of the same age inspecting each other closely as they used the bathroom. Which is the most appropriate recommendation the nurse should make? a. Punish children so this behavior stops. b. Neither condone nor condemn the curiosity. c. Allow children unrestricted permission to satisfy this curiosity. d. Get counseling for this unusual and dangerous behavior.

ANS: B Three-year-olds become aware of anatomic differences and are concerned about how the other "works." Such exploration should not be condoned or condemned. Children should not be punished for this normal exploration. Encouraging the children to ask questions of the parents and redirecting their activity are more appropriate than giving permission. Exploration is age-appropriate and not dangerous behavior

Which is an appropriate nursing intervention when caring for a child in traction? a. Remove adhesive traction straps daily to prevent skin breakdown. b. Assess for tightness, weakness, or contractures in uninvolved joints and muscles. c. Provide active range-of-motion exercises to affected extremity three times a day. d. Keep the child in one position to maintain good alignment.

ANS: B Traction places stress on the affected bone, joint, and muscles. The nurse must assess for tightness, weakness, or contractures developing in the uninvolved joints and muscles. The adhesive straps should be released or replaced only when absolutely necessary. Active, passive, or active with resistance exercises should be carried out for the unaffected extremity only. Movement is expected with children. Each time the child moves, the nurse should check to ensure that proper alignment is maintained.

30. Which vaccine is now recommended for the immunization of all newborns? a. Hepatitis A vaccine b. Hepatitis B vaccine c. Hepatitis C vaccine d. Hepatitis A, B, and C vaccines

ANS: B Universal vaccination for hepatitis B is now recommended for all newborns. A vaccine is available for hepatitis A, but it is not yet universally recommended. No vaccine is currently available for hepatitis C. Only hepatitis B vaccine is recommended for newborns.

The vector reservoir for agents causing viral encephalitis in the United States is which of the following? a. Tarantula spiders b. Mosquitoes and ticks c. Carnivorous wild animals d. Domestic and wild animals

ANS: B Viral encephalitis, not attributable to a childhood viral disease, is usually transmitted by mosquitoes and ticks. The vector reservoir for most agents pathogenic for humans and detected in the United States are mosquitoes and ticks; therefore most cases of encephalitis appear during the hot summer months.

Which statement is correct about toilet training? a. Bladder training is usually accomplished before bowel training? b. Wanting to please the parent helps motivate the child to use the toilet. c. Watching older siblings use the toilet confuses the child. d. Children must be forced to sit on the toilet when first learning

ANS: B Voluntary control of the anal and urethral sphincters is achieved sometime after the child is walking. The child must be able to recognize the urge to let go and to hold on. The child must want to please parent by holding on rather than pleasing self by letting go. Bowel training precedes bladder training. Watching older siblings provides role modeling and facilitates imitation for the toddler. The child should be introduced to the potty chair or toilet in a nonthreatening manner.

An adolescent with a fractured femur is in Russell's traction. Surgical intervention to correct the fracture is scheduled for the morning. Nursing actions should include which action? a. Maintaining continuous traction until 1 hour before the scheduled surgery b. Maintaining continuous traction and checking position of traction frequently c. Releasing traction every hour to perform skin care d. Releasing traction once every 8 hours to check circulation

ANS: B When the muscles are stretched, muscle spasm ceases and permits realignment of the bone ends. The continued maintenance of traction is important during this phase because releasing the traction allows the muscle's normal contracting ability to again cause malpositioning of the bone ends. Continuous traction must be maintained to keep the bone ends in satisfactory realignment. Releasing at any time, either 1 hour before surgery, once every hour for skin care, or once every 8 hours would not keep the fracture in satisfactory alignment.

An important consideration for the school nurse who is planning a class on bicycle safety is a. Most bicycle injuries involve collision with an automobile. b. Head injuries are the major causes of bicycle-related fatalities. c. Children should wear bicycle helmets if they ride on paved streets. d. Children should not ride double unless the bicycle has an extra-large seat.

ANS: B Feedback A Although motor vehicle collisions do cause injuries to bicyclists, most injuries result from falls. B The most important aspect of bicycle safety is to encourage the rider to use a protective helmet. Head injuries are the major cause of bicycle-related fatalities. C The child should always wear a properly fitted helmet approved by the U.S. Consumer Product Safety Commission. D Children should not ride double.

Which behavior by parents or teachers will best assist the child in negotiating the developmental task of industry? a. Identifying failures immediately and asking the child's peers for feedback b. Structuring the environment so that the child can master tasks c. Completing homework for children who are having difficulty in completing assignments d. Decreasing expectations to eliminate potential failures

ANS: B Feedback A Asking peers for feedback reinforces the child's feelings of failure. B The task of the caring teacher or parent is to identify areas in which a child is competent and to build on successful experiences to foster feelings of mastery and success. Structuring the environment to enhance self-confidence and to provide the opportunity to solve increasingly more complex problems will promote a sense of mastery. C When teachers or parents complete children's homework for them, it sends the message that you do not trust them to do a good job. Providing assistance and suggestions and praising their best efforts are more appropriate. D Decreasing expectations to eliminate failures will not promote a sense of achievement or mastery.

A 17-year-old tells the nurse that he is not having sex because it would make his parents very angry. This response indicates that the adolescent has a developmental lag in which area? a. Cognitive development b. Moral development c. Psychosocial development d. Psychosexual development

ANS: B Feedback A Cognitive development is related to moral development, but it is not the pivotal point in determining right and wrong behaviors. B The appropriate moral development for a 17-year-old would include evidence that the teenager has internalized a value system and does not depend on parents to determine right and wrong behaviors. Adolescents who remain concrete thinkers may never advance beyond conformity to please others and avoid punishment. C Identity formation is the psychosocial development task. Energy is focused within the adolescent, who exhibits behavior that is self-absorbed and egocentric. D Although a task during adolescence is the development of a sexual identity, the teenager's dependence on the parents' sanctioning of right or wrong behavior is more appropriately related to moral development.

Which is assessed with Tanner staging? a. Hormone levels b. Secondary sex characteristics c. Response to growth hormone secretion tests d. Hyperthyroidism

ANS: B Feedback A Hormone levels are assessed by their concentration in the blood. B Tanner stages are used to assess staging of secondary sex characteristics at puberty. C Growth hormone secretion tests are not associated with Tanner staging. D Tanner stages are not associated with hyperthyroidism.

According to Erikson, the psychosocial task of adolescence is to develop a. Intimacy b. Identity c. Initiative d. Independence

ANS: B Feedback A Intimacy is the developmental stage for early adulthood. B Traditional psychosocial theory holds that the developmental crises of adolescence lead to the formation of a sense of identity. C Initiative is the developmental stage for early childhood. D Independence is not one of Erikson's developmental stages.

What predisposes the adolescent to feel an increased need for sleep? a. An inadequate diet b. Rapid physical growth c. Decreased activity that contributes to a feeling of fatigue d. The lack of ambition typical of this age-group

ANS: B Feedback A Rapid physical growth, the tendency toward overexertion, and the overall increased activity of this age contributes to fatigue. B During growth spurts, the need for sleep is increased. C Rapid physical growth, the tendency toward overexertion, and the overall increased activity of this age contributes to fatigue. D Rapid physical growth, the tendency toward overexertion, and the overall increased activity of this age contributes to fatigue.

Which statement is the most appropriate advice to give parents of a 16-year-old who is rebellious? a. "You need to be stricter so that your teen stops trying to test the limits." b. "You need to collaborate with your daughter and set limits that are perceived as being reasonable." c. "Increasing your teen's involvement with her peers will improve her self-esteem." d. "Allow your teenager to choose the type of discipline that is used in your home."

ANS: B Feedback A Setting stricter limits typically does not decrease rebelliousness or decrease testing of parental limits. B Allowing teenagers to choose between realistic options and offering consistent and structured discipline typically enhances cooperation and decreases rebelliousness. Structure helps adolescents to feel more secure and assists them in the decision-making process. C Increasing peer involvement does not typically increase self-esteem. D Allowing teenagers to choose the method of discipline is not realistic and typically does not reduce rebelliousness.

Many adolescents decide to follow a vegetarian diet during their teen years. The nurse can advise the adolescent and his or her parents that a. This diet will not meet the nutritional requirements of growing teens. b. A vegetarian diet is healthy for this population. c. An adolescent on a vegetarian diet is less likely to eat high-fat or low-nutrient foods. d. A vegetarian diet requires little extra meal planning.

ANS: B Feedback A Several dietary organizations have suggested that a vegetarian diet, if correctly followed, is healthy for this population. B A vegetarian diet is healthy for this population, and the low-fat aspect of the diet can prevent future cardiovascular problems. C As with any adolescent, nurses need to advise teens who follow a vegetarian eating plan to avoid low-nutrient, high-fat foods. D The nurse can assist with planning food choices that will provide sufficient calories and necessary nutrients. The focus is on obtaining enough calories for growth and energy from a variety of fruits and vegetables, whole grains, nuts, and soymilk.

A nurse is assessing an older school-age child recently admitted to the hospital. Which assessment indicates that the child is in an appropriate stage of cognitive development? a. The child's addition and subtraction ability b. The child's ability to classify c. The child's vocabulary d. The child's play activity

ANS: B Feedback A Subtraction and addition are appropriate cognitive activities for the young school-age child. B The ability to classify things from simple to complex and the ability to identify differences and similarities are cognitive skills of the older school-age child; this demonstrates use of classification and logical thought processes. C Vocabulary is not as valid an assessment of cognitive ability as is the child's ability to classify. D Play activity is not as valid an assessment of cognitive function as is the child's ability to classify.

Which behavior suggests appropriate psychosocial development in the adolescent? a. The adolescent seeks validation for socially acceptable behavior from older adults. b. The adolescent is self-absorbed and self-centered and has sudden mood swings. c. Adolescents move from peers and enjoy spending time with family members. d. Conformity with the peer group increases in late adolescence.

ANS: B Feedback A The peer group validates acceptable behavior during adolescence. B During adolescence, energy is focused within. Adolescents concentrate on themselves in an effort to determine who they are or who they will be. Adolescents are likely to be impulsive and impatient. Parents often describe their teenager as being "self-centered or lazy." C Adolescents move from family and enjoy spending time with peers. Adolescents also spend time alone; they need this time to think and concentrate on themselves. D Conformity becomes less important in late adolescence.

Which is an appropriate disciplinary intervention for the school-age child? a. Using time-out periods b. Using a consequence that is consistent with the inappropriate behavior c. Using physical punishment d. Using lengthy dialog about inappropriate behavior

ANS: B Feedback A Time-out periods are more appropriate for younger children. B A consequence that is related to the inappropriate behavior is the recommended discipline. C Physical intervention is an inappropriate form of discipline. It does not connect the discipline with the child's inappropriate behavior. D Lengthy discussions typically are not helpful.

27. Which tool measures body fat most accurately? a. Stadiometer b. Calipers c. Cloth tape measure d. Paper or metal tape measure

ANS: B Calipers are used to measure skin-fold thickness, which is an indicator of body fat content. Stadiometers are used to measure height. Cloth tape measures should not be used because they can stretch. Paper or metal tape measures can be used for recumbent lengths and other body measurements that must be made

31. What is the earliest age at which a satisfactory radial pulse can be taken in children? a. 1 year b. 2 years c. 3 years d. 6 years

ANS: B Satisfactory radial pulses can be used in children older than 2 years. In infants and young children, the apical pulse is more reliable. The apical pulse can be used for assessment at these ages

15. The nurse is taking a health history on an adolescent. Which best describes how the chief complaint should be determined? a. Ask for detailed listing of symptoms. b. Ask adolescent, "Why did you come here today?" c. Use what adolescent says to determine, in correct medical terminology, what the problem is. d. Interview parent away from adolescent to determine chief complaint.

ANS: B The chief complaint is the specific reason for the child's visit to the clinic, office, or hospital. Because the adolescent is the focus of the history, this is an appropriate way to determine the chief complaint. A detailed listing of symptoms will make it difficult to determine the chief complaint. The adolescent should be prompted to tell which symptom caused him to seek help at this time. The chief complaint is usually written in the words that the parent or adolescent uses to describe the reason for seeking help. The parent and adolescent may be interviewed separately, but the nurse should determine the reason the adolescent is seeking attention at this time.

6. What is an important consideration for the nurse who is communicating with a very young child? a. Speak loudly, clearly, and directly. b. Use transition objects, such as a doll. c. Disguise own feelings, attitudes, and anxiety. d. Initiate contact with child when parent is not present.

ANS: B Using a transition object allows the young child an opportunity to evaluate an unfamiliar person (the nurse). This will facilitate communication with a child this age. Speaking in this manner will tend to increase anxiety in very young children. The nurse must be honest with the child. Attempts at deception will lead to a lack of trust. Whenever possible, the parent should be present for interactions with young children

Which are characteristic of physical development of a 30-month-old child? (Select all that apply.) a. Birth weight has doubled. b. Primary dentition is complete. c. Sphincter control is achieved. d. Anterior fontanel is open. e. Length from birth is doubled. f. Left or right handedness is established.

ANS: B, C Usually by age 30 months, the primary dentition of 20 teeth is completed, and the child has sphincter control in preparation for bowel and bladder control. Birth weight doubles at approximately ages 5 to 6 months. The anterior fontanel closes at age 12 to 18 months. Birth length is doubled around age 4. Left or right handedness is not established until about age 5.

Tattoos have become increasingly popular among mainstream adolescents. Like clothing and hairstyles, tattoos serve to define one's identity. It is important for nurses to caution adolescents on the health risks of obtaining a tattoo. These include (select all that apply) a. Amateur tattoos are difficult to remove. b. Tattoos pose a risk for bloodborne and skin infections. c. Health care professionals must be notified of the existence of a tattoo before a magnetic resonance imaging (MRI) scan. d. Tattoo dyes may cause allergic reactions. e. Tattoo parlors are well regulated.

ANS: B, C, D Feedback Correct Tattoos carry the risk for contracting bloodborne diseases such as hepatitis B and HIV. Infection, allergic reaction to the dye, scarring, or keloid formation can occur. Should an MRI ever be required, it is important to notify the health care professionals, because the dyes can contain iron and other metals. Incorrect Amateur tattoos are easily removed; however, studio tattoos made with red and green dye are extremely difficult to remove. Very little regulation exists in the tattoo industry; therefore, the cleanliness of each tattoo parlor varies. Teens should be counseled to avoid making an impulsive decision to get a tattoo.

2. Which is true concerning hepatitis B? (Select all that apply.) a. Hepatitis B cannot exist in carrier state. b. Hepatitis B can be prevented by HBV vaccine. c. Hepatitis B can be transferred to an infant of a breastfeeding mother. d. Onset of hepatitis B is insidious. e. Principal mode of transmission for hepatitis B is fecal-oral route. f. Immunity to hepatitis B occurs after one attack.

ANS: B, C, D, F The vaccine elicits the formation of an antibody to the hepatitis B surface antigen, which is protective against hepatitis B. Hepatitis B can be transferred to an infant of a breastfeeding mother, especially if the mother's nipples are cracked. The onset of hepatitis B is insidious. Immunity develops after one exposure to hepatitis B. Hepatitis B has a carrier state. The fecal-oral route is the principal mode of transmission for hepatitis A. Hepatitis B is transmitted through the parenteral route.

A 6-year-old child is scheduled for an IV urography (IVP) in the morning. Which preparatory interventions should the nurse plan to implement? (Select all that apply.) a. Clear liquids in the morning before the procedure b. Cathartic in the evening before the procedure c. Soapsuds enema the morning of the procedure d. Insertion of a Foley catheter before the procedure e. Teaching with regard to insertion of an intravenous catheter before the procedure

ANS: B, C, E The IV urography is a test done to provide information about the integrity of the kidneys, ureters, and bladder. It requires an IV injection of a contrast medium with X-ray films made 5, 10, and 15 minutes after injection. Delayed films (30, 60 minutes, and so on) are also obtained. The preparation for children ages 2 to 14 years includes cathartic on the evening before examination, nothing orally after midnight, and an enema (soapsuds) on the morning of examination. Teaching about the insertion of an intravenous catheter should be part of the preoperative preparation. Insertion of a Foley catheter is not part of the preparation for an IVP.

A clinic nurse is conducting a staff in-service for other clinic nurses about signs and symptoms of a rhabdomyosarcoma tumor. Which should be included in the teaching session? (Select all that apply.) a. Bone fractures b. Abdominal mass c. Sore throat and ear pain d. Headache e. Ecchymosis of conjunctiva

ANS: B, C, E The initial signs and symptoms of rhabdomyosarcoma tumors are related to the site of the tumor and compression of adjacent organs. Some tumor locations, such as the orbit, manifest early in the course of the illness. Other tumors, such as those of the retroperitoneal area, only produce symptoms when they are relatively large and compress adjacent organs. Unfortunately, many of the signs and symptoms attributable to rhabdomyosarcoma are vague and frequently suggest a common childhood illness, such as "earache" or "runny nose." An abdominal mass, sore throat and ear pain, and ecchymosis of conjunctiva are signs of a rhabdomyosarcoma tumor. Bone fractures would be seen in osteosarcoma and a headache is a sign of a brain tumor.

Which should the nurse teach to parents of toddlers about accidental poison prevention? (Select all that apply.) a. Keep toxic substances in the garage. b. Discard empty poison containers. c. Know the number of the nearest poison control center. d. Remove colorful labels from containers of toxic substances. e. Caution child against eating nonedible items, such as plants.

ANS: B, C, E To prevent accidental poisoning, parents should be taught to promptly discard empty poison containers, know the number of the nearest poison control center and to caution the child against eating nonedible items, such as plants. Parents should place all potentially toxic agents, including cosmetics, personal care items, cleaning products, pesticides, and medications in a locked cabinet, not in the garage. Parents should be taught to never remove labels from containers of toxic substances.

Which strategies can a nurse teach to parents of a child experiencing uncomplicated school refusal? Select all that apply. a. The child should be allowed to stay home until the anxiety about going to school is resolved. b. Parents should be empathetic yet firm in their insistence that the child attends school. c. A modified school attendance may be necessary. d. Parents need to pick the child up at school whenever the child wants to come home. e. Parents need to communicate with the teachers about the situation.

ANS: B, C, E Feedback Correct In uncomplicated cases of school refusal, the parent needs to return the child to school as soon as possible. If symptoms are severe, a limited period of part-time or modified school attendance may be necessary. For example, part of the day may be spent in the counselor's or school nurse's office, with assignments obtained from the teacher. Parents should be empathetic yet firm and consistent in their insistence that the child attend school. Incorrect Parents should not pick the child up at school once the child is there. The principal and teacher should be told about the situation so that they can cooperate with the treatment plan.

A school-age child has been admitted to the hospital with an exacerbation of nephrotic syndrome. Which clinical manifestations should the nurse expect to assess? (Select all that apply.) a. Weight loss b. Facial edema c. Cloudy smoky brown-colored urine d. Fatigue e. Frothy-appearing urine

ANS: B, D, E A child with nephrotic syndrome will present with facial edema, fatigue, and frothy-appearing urine (proteinuria). Weight gain, not loss, is expected because of the fluid retention. Cloudy smoky brown-colored urine is seen with acute glomerulonephritis but not with nephrotic syndrome because there is no gross hematuria associated with nephrotic syndrome.

4. The nurse is preparing to care for an infant returning from pyloromyotomy surgery. Which prescribed orders should the nurse anticipate implementing? (Select all that apply.) a. NPO for 24 hours b. Administration of analgesics for pain c. Ice bag to the incisional area d. IV fluids continued until tolerating PO e. Clear liquids as the first feeding

ANS: B, D, E Feedings are usually instituted soon after a pyloromyotomy surgery, beginning with clear liquids advancing to formula or breast milk as tolerated. IV fluids are administered until the infant is taking and retaining adequate amounts by mouth. Appropriate analgesics should be given around the clock because pain is continuous. Ice should not be applied to the incisional area as it vasoconstricts and would reduce circulation to the incisional area and impair healing.

Parents of a teenager ask the nurse what signs they should look for if their child is in a gang. The nurse should include which signs when answering? Select all that apply. a. Plans to try out for the debate team at school b. Skipping classes to go to the mall c. Hanging out with friends they have had since childhood d. Unexplained source of money e. Fear of the police

ANS: B, D, E Feedback Correct Signs of gang involvement include skipping classes, unexplained sources of money, and fear of the police. Associating with new friends while ignoring old friends is also a sign. A change in attitude toward participating in activities is another sign of gang involvement. Incorrect Plans to try out for the debate team at school are not a sign of gang involvement. Hanging out with friends he or she has had since childhood is not a sign of gang involvement.

In terms of language and cognitive development, a 4-year-old child would be expected to have which traits? (Select all that apply.) a. Think in abstract terms. b. Follow directional commands. c. Understand conservation of matter. d. Use sentences of eight words. e. Tell exaggerated stories. f. Comprehend another person's perspective.

ANS: B, E Children ages 3 to 4 years can give and follow simple commands and tell exaggerated stories. Children cannot think abstractly at age 4 years. Conservation of matter is a developmental task of the school-age child. Five-year-old children use sentences with eight words with all parts of speech. A 4-year-old child cannot comprehend another's perspective

Which is most likely to encourage parents to talk about their feelings related to their child's illness? a. Be sympathetic. b. Use direct questions. c. Use open-ended questions. d. Avoid periods of silence.

ANS: C Closed-ended questions should be avoided when attempting to elicit parents' feelings. Open-ended questions require the parent to respond with more than a brief answer. Sympathy is having feelings or emotions in common with another person rather than understanding those feelings (empathy). Sympathy is not therapeutic in helping the relationship. Direct questions may obtain limited information. In addition, the parent may consider them threatening. Silence can be an effective interviewing tool. It allows sharing of feelings in which two or more people absorb the emotion in depth. Silence permits the interviewee to sort out thoughts and feelings and search for responses to questions

34. The nurse observes yellow staining in the sclera of eyes, soles of feet, and palms of hands. This should be interpreted as: a. normal. b. erythema c. jaundice. d. ecchymosis.

ANS: C Jaundice is defined as the yellow staining of the skin, usually by bile pigments. Yellow staining is not a normal appearance of the skin. Erythema is redness that results from increased blood flow to the area. Ecchymosis is large, diffuse areas, usually black and blue, caused by hemorrhage of blood into the skin.

B-Adrenergic agonists and methylxanthines are often prescribed for a child with an asthma attack. What is their action?

Dilate the bronchioles

Which of the following neurologic diagnostic tests gives a visualized horizontal and vertical cross section of the brain at any axis? a. Nuclear brain scan b. Echoencephalography c. CT scan d. Magnetic resonance imaging (MRI)

ANS: C A CT scan provides a visualization of the horizontal and vertical cross sections of the brain at any axis.

Which of the following is descriptive of a concussion? a. Petechial hemorrhages cause amnesia. b. Visible bruising and tearing of cerebral tissue occur. c. It is a transient and reversible neuronal dysfunction. d. A slight lesion develops remote from the site of trauma.

ANS: C A concussion is a transient, reversible neuronal dysfunction with instantaneous loss of awareness and responsiveness resulting from trauma to the head.

A child has been seizure free for 2 years. A father asks the nurse how much longer the child will need to take the antiseizure medications. The nurse includes which of the following in the response? a. Medications can be discontinued at this time. b. The child will need to take the drugs for 5 years after the last seizure. c. A step-wise approach will be used to reduce the dosage gradually. d. Seizure disorders are a life-long problem. Medications cannot be discontinued.

ANS: C A predesigned protocol is used to wean a child gradually off antiseizure medications, usually when the child is seizure free for 2 years and has a normal electroencephalogram (EEG).

A child with hypopituitarism is being started on growth hormone (GH) therapy. Nursing considerations should be based on knowledge of which of the following? a. Treatment is most successful if it is started during adolescence. b. Treatment is considered successful if children attain full stature by adulthood. c. Replacement therapy requires daily subcutaneous injections. d. Replacement therapy will be required throughout child's lifetime.

ANS: C Additional support is required for children who require hormone replacement therapy, such as preparation for daily subcutaneous injections and education for self-management during the school-age years.

A 10-year-old boy on a bicycle has been hit by a car in front of the school. The school nurse immediately assesses airway, breathing, and circulation. The next nursing action should be which of the following? a. Place on side. b. Take blood pressure. c. Stabilize neck and spine. d. Check scalp and back for bleeding.

ANS: C After determining that the child is breathing and has adequate circulation, the next action is to stabilize the neck and spine to prevent any additional trauma.

A 3-year-old child is hospitalized after a near-drowning accident. The child's mother complains to the nurse, "This seems unnecessary when he is perfectly fine." The nurse's best reply would be which of the following? a. "He still needs a little extra oxygen." b. "I'm sure he is fine, but the doctor wants to make sure." c. "The reason for this is that complications could still occur." d. "It is important to observe for possible central nervous system problems."

ANS: C All children who have a near-drowning experience should be admitted to the hospital for observation. Although many children do not appear to have suffered adverse effects from the event, complications such as respiratory compromise and cerebral edema may occur 24 hours after the incident.

The child of 15 to 30 months is likely to be struggling with which developmental task? a. Trust b. Initiative c. Autonomy d. Intimacy

ANS: C Autonomy vs shame and doubt is the developmental task of toddlers. Trust vs mistrust is the developmental stage of infancy. Initiative vs guilt is the developmental stage of early childhood. Intimacy and solidarity vs isolation is the developmental stage of early adulthood.

A nurse is assessing a preschool-age child and notes the child exhibits magical thinking. According to Piaget, which describes magical thinking? a. Events have cause and effect. b. God is like an imaginary friend. c. Thoughts are all-powerful. d. If the skin is broken, the child's insides will come out.

ANS: C Because of their egocentrism and transductive reasoning, preschoolers believe that thoughts are all-powerful. Cause-and-effect implies logical thought, not magical thinking. Thinking God is like an imaginary friend is an example of concrete thinking in a preschooler's spiritual development. Thinking that if the skin is broken, the child's insides will come out is an example of concrete thinking in development of body image.

A parent asks the nurse why self-monitoring of blood glucose is being recommended for her child with diabetes. The nurse should base the explanation on which of the following? a. It is a less expensive method of testing. b. It is not as accurate as laboratory testing. c. Children are better able to manage the diabetes. d. The parents are better able to manage the disease.

ANS: C Blood glucose self-management has improved diabetes management and can be used successfully by children from the time of diagnosis. Insulin dosages can be adjusted based on blood glucose results.

25. A nurse is conducting an in-service on childhood gastrointestinal disorders. Which statement is most descriptive of Meckel diverticulum? a. It is more common in females than in males. b. It is acquired during childhood. c. Intestinal bleeding may be mild or profuse. d. Medical interventions are usually sufficient to treat the problem.

ANS: C Bloody stools are often a presenting sign of Meckel diverticulum. It is associated with mild to profuse intestinal bleeding. It is twice as common in males as in females, and complications are more frequent in males. Meckel diverticulum is the most common congenital malformation of the GI tract and is present in 1% to 4% of the general population. The standard therapy is surgical removal of the diverticulum.

In the clinic waiting room, a nurse observes a parent showing an 18-month-old child how to make a tower out of blocks. The nurse should recognize in this situation that: a. blocks at this age are used primarily for throwing. b. toddlers are too young to imitate the behavior of others. c. toddlers are capable of building a tower of blocks. d. toddlers are too young to build a tower of blocks.

ANS: C Building with blocks is a good parent-child interaction. The 18-month-old child is capable of building a tower of three or four blocks. The ability to build towers of blocks usually begins at age 15 months. With ongoing development, the child is able to build taller towers. The 18-month-old child imitates others around him or her.

Developmentally, most children at age 12 months: a. use a spoon adeptly. b. relinquish the bottle voluntarily. c. eat the same food as the rest of the family. d. reject all solid food in preference to the bottle.

ANS: C By age 12 months, most children are eating the same food that is prepared for the rest of the family. Using a spoon usually is not mastered until age 18 months. The parents should be engaged in weaning a child from a bottle if that is the source of liquid. Toddlers should be encouraged to drink from a cup at the first birthday and be weaned from the bottle totally by 14 months. The child should be weaned from a milk- or formula-based diet to a balanced diet that includes iron-rich sources of food.

The nurse is teaching the parents of a child who is receiving propylthiouracil for the treatment of hyperthyroidism (Graves disease). Which of the following statements made by the parent indicates a correct understanding of the teaching? a. "I would expect my child to gain weight while taking this medication." b. "I would expect my child to experience episodes of ear pain while taking this medication." c. "If my child develops a sore throat and fever, I should contact the physician immediately." d. "If my child develops the stomach flu, my child will need to be hospitalized."

ANS: C Children being treated with propylthiouracil must be carefully monitored for the side effects of the drug. Parents must be alerted that sore throat and fever accompany the grave complication of leukopenia. These symptoms should be immediately reported.

5. An infant is brought to the emergency department with dehydration. Which physical assessment finding does the nurse expect? a. Weight gain b. Bradycardia c. Poor skin turgor d. Brisk capillary refill

ANS: C Clinical manifestations of dehydration include poor skin turgor, weight loss, lethargy, and tachycardia. The infant would have prolonged capillary refill, not brisk

Which of the following types of seizures involves both hemispheres of the brain? a. Focal b. Partial c. Generalized d. Acquired

ANS: C Clinical observations of generalized seizures indicate that the initial involvement is from both hemispheres.

14. Constipation has recently become a problem for a school-age girl. She is healthy except for seasonal allergies that are being treated with antihistamines. The nurse should suspect that the constipation is most likely caused by: a. diet. b. allergies. c. antihistamines. d. emotional factors.

ANS: C Constipation may be associated with drugs such as antihistamines, antacids, diuretics, opioids, antiepileptics, and iron. Because this is the only known change in her habits, the addition of antihistamines is most likely the cause of the diarrhea. With a change in bowel habits, the role of any recently prescribed medications should be assessed.

27. Which is used to treat moderate to severe inflammatory bowel disease? a. Antacids b. Antibiotics c. Corticosteroids d. Antidiarrheal medications

ANS: C Corticosteroids, such as prednisone and prednisolone, are used in short bursts to suppress the inflammatory response in inflammatory bowel disease. Antacids and antidiarrheal medications are not drugs of choice in the treatment of inflammatory bowel disease. Antibiotics may be used as an adjunctive therapy to treat complications.

A young child's parents call the nurse after their child was bitten by a raccoon in the woods. The nurse's recommendation should be based on which of the following? a. Child should be hospitalized for close observation. b. No treatment is necessary if thorough wound cleaning is done. c. Antirabies prophylaxis must be initiated. d. Antirabies prophylaxis must be initiated if clinical manifestations appear.

ANS: C Current therapy for a rabid animal bite consists of a thorough cleansing of the wound and passive immunization with human rabies immune globulin (HRIG) as soon as possible.

Which of the following types of fractures describes traumatic separation of cranial sutures? a. Basilar b. Compound c. Diastatic d. Depressed

ANS: C Diastatic skull fractures are traumatic separations of the cranial sutures.

Which of the following is a common clinical manifestation of juvenile hypothyroidism? a. Insomnia b. Diarrhea c. Dry skin d. Accelerated growth

ANS: C Dry skin, mental decline, and myxedematous skin changes are associated with juvenile hypothyroidism.

Although a 14-month-old girl received a shock from an electric outlet recently, her parent finds her about to place a paper clip in another outlet. Which is the best interpretation of this behavior? a. Her cognitive development is delayed. b. This is typical behavior because toddlers are not very developed. c. This is typical behavior because of the inability to transfer knowledge to new situations. d. This is not typical behavior because toddlers should know better than to repeat an act that caused pain.

ANS: C During the tertiary circular reactions stage, children have only a rudimentary sense of the classification of objects. The appearance of an object denotes its function for these children. The slot of an outlet is for putting things into. Her cognitive development is appropriate for her age. Trying to put things into an outlet is typical behavior for a toddler. Only some awareness exists of a causal relation between events.

During the preschool period, injury prevention efforts should emphasize: a. constant vigilance and protection. b. punishment for unsafe behaviors. c. education for safety and potential hazards. d. limitation of physical activities.

ANS: C Education for safety and potential hazards is appropriate for preschoolers because they can begin to understand dangers. Constant vigilance and protection is not practical at this age because preschoolers are becoming more independent. Punishment may make children scared of trying new things. Limitation of physical activities is not appropriate

The parents of a child who has just been diagnosed with type 1 diabetes ask about exercise. The nurse should explain that: a. exercise will increase blood glucose. b. exercise should be restricted. c. extra snacks are needed before exercise. d. extra insulin is required during exercise.

ANS: C Exercise lowers blood glucose levels, which can be compensated for by extra snacks.

8. Which is a parasite that causes acute diarrhea? a. Shigella organisms b. Salmonella organisms c. Giardia lamblia d. Escherichia coli

ANS: C G. lamblia is a parasite that represents 10% of non-dysenteric illness in the United States. Shigella, Salmonella, and E. coli are bacterial pathogens.

The most common clinical manifestation(s) of brain tumors in children is which of the following? a. Irritability b. Seizures c. Headaches and vomiting d. Fever and poor fine motor control

ANS: C Headaches, especially on awakening, and vomiting that is not related to feeding are the most common clinical manifestation(s) of brain tumors in children.

29. Which statement best characterizes hepatitis A? a. Incubation period is 6 weeks to 6 months. b. Principal mode of transmission is through the parenteral route. c. Onset is usually rapid and acute. d. There is a persistent carrier state.

ANS: C Hepatitis A is the most common form of acute hepatitis in most parts of the world. It is characterized by a rapid and acute onset. The incubation period is approximately 3 weeks for hepatitis A and the principal mode of transmission for it is the fecal-oral route. Hepatitis A does not have a carrier state.

A school nurse is conducting a staff in-service to other school nurses on idiopathic scoliosis. During which period of child development does idiopathic scoliosis become most noticeable? a. Newborn period b. When child starts to walk c. Preadolescent growth spurt d. Adolescence

ANS: C Idiopathic scoliosis is most noticeable during the preadolescent growth spurt. Idiopathic scoliosis is seldom apparent before age 10 years. Diagnosis usually occurs during the preadolescent growth spurt.

Which of the following statements best describes hypopituitarism? a. Growth is normal during the first 3 years of life. b. Weight is usually more retarded than height. c. Skeletal proportions are normal for age. d. Most of these children have subnormal intelligence.

ANS: C In children with hypopituitarism, the skeletal proportions are normal.

The nurse is teaching a family how to care for their infant in a Pavlik harness to treat developmental dysplasia of the hip. Which should be included? a. Apply lotion or powder to minimize skin irritation. b. Remove harness several times a day to prevent contractures. c. Return to clinic every 1 to 2 weeks. d. Place diaper over harness, preferably using a superabsorbent disposable diaper that is relatively thin.

ANS: C Infants have a rapid growth pattern. The child needs to be assessed by the practitioner every 1 to 2 weeks for possible adjustments. Lotions and powders should not be used with the harness. The harness should not be removed, except as directed by the practitioner. A thin disposable diaper can be placed under the harness.

19. The nurse is explaining to a parent how to care for a school-age child with vomiting associated with a viral illness. Which action should the nurse include? a. Avoid carbohydrate-containing liquids. b. Give nothing by mouth for 24 hours. c. Brush teeth or rinse mouth after vomiting. d. Give plain water until vomiting ceases for at least 24 hours.

ANS: C It is important to emphasize the need for the child to brush the teeth or rinse the mouth after vomiting to dilute the hydrochloric acid that comes in contact with the teeth. Ad libitum administration of glucose-electrolyte solution to an alert child will help restore water and electrolytes satisfactorily. It is important to include carbohydrate to spare body protein and avoid ketosis.

Which of the following is the initial clinical manifestation of generalized seizures? a. Being confused b. Feeling frightened c. Losing consciousness d. Seeing flashing lights

ANS: C Loss of consciousness is a frequent occurrence in generalized seizures and is the initial clinical manifestation.

At what age is sexual development in boys and girls considered to be precocious? a. Boys, 11 years; girls, 9 years b. Boys, 12 years; girls, 10 years c. Boys, 9 years; girls, 8 years d. Boys, 10 years; girls, 9-1/2 years

ANS: C Manifestations of sexual development before age 9 in boys and age 8 in girls is considered precocious and should be investigated.

2. A nurse is conducting an in-service on gastrointestinal disorders. The nurse includes that melena, the passage of black, tarry stools, suggests bleeding from which area? a. Perianal or rectal area b. Hemorrhoids or anal fissures c. Upper gastrointestinal (GI) tract d. Lower GI tract

ANS: C Melena is denatured blood from the upper GI tract or bleeding from the right colon. Blood from the perianal or rectal area, hemorrhoids, or lower GI tract would be bright red

Which snack should the nurse recommend parents offer to their slightly overweight preschool child? a. Carbonated beverage b. 10% fruit juice c. Low fat chocolate milk d. Whole milk

ANS: C Milk and dairy products are excellent sources of calcium and vitamin D (fortified). Low-fat milk may be substituted, so the quantity of milk may remain the same while limiting fat intake overall. Parents should be educated regarding non-nutritious fruit drinks, which usually contain less than 10% fruit juice yet are often advertised as healthy and nutritious, sugar content is dramatically increased and often precludes an adequate intake of milk by the child. In young children, intake of carbonated beverages that are acidic or that contain high amounts of sugar is also known to contribute to dental caries. Low fat milk should be substituted for whole milk if the child is slightly overweight.

10. Which therapeutic management should the nurse prepare to initiate first for a child with acute diarrhea and moderate dehydration? a. Clear liquids b. Adsorbents, such as kaolin and pectin c. Oral rehydration solution (ORS) d. Antidiarrheal medications such as paregoric

ANS: C ORS is the first treatment for acute diarrhea. Clear liquids are not recommended because they contain too much sugar, which may contribute to diarrhea. Adsorbents are not recommended. Antidiarrheals are not recommended because they do not get rid of pathogens

Which of the following terms is used to describe a child's level of consciousness when the child is arousable with stimulation? a. Stupor b. Confusion c. Obtundation d. Disorientation

ANS: C Obtundation describes a level of consciousness in which the child is arousable with stimulation.

An appropriate nursing intervention when caring for an unconscious child would be which of the following? a. Change the child's position infrequently to minimize the chance of increased ICP. b. Avoid using narcotics or sedatives to provide comfort and pain relief. c. Monitor fluid intake and output carefully to avoid fluid overload and cerebral edema. d. Give tepid sponge baths to reduce fever, since antipyretics are contraindicated.

ANS: C Often comatose patients cannot cope with the quantity of fluids that they normally tolerate. Overhydration must be avoided to prevent fatal cerebral edema.

Imaginary playmates are beneficial to the preschool child because they: a. take the place of social interactions. b. take the place of pets and other toys. c. become friends in times of loneliness. d. accomplish what the child has already successfully accomplished.

ANS: C One purpose of an imaginary friend is to be a friend in time of loneliness. Imaginary friends do not take the place of social interaction, but may encourage conversation. Imaginary friends do not take the place of pets or toys. Imaginary friends accomplish what the child is still attempting.

The nurse is taking care of a 10-year-old child who has osteomyelitis. Which treatment plan is considered the primary method of treating osteomyelitis? a. Joint replacement b. Bracing and casting c. Intravenous antibiotic therapy d. Long-term corticosteroid therapy

ANS: C Osteomyelitis is an infection of the bone, most commonly caused by Staphylococcus aureus. The treatment of choice is antibiotics. Joint replacement, bracing and casting, and long-term corticosteroid therapy are not indicated for infectious processes.

Parents need further teaching about the use of car safety seats if they make which statement? a. "Even if our toddler helps buckle the straps, we will double-check the fastenings." b. "We won't start the car until everyone is properly restrained." c. "We won't need to use the car seat on short trips to the store." d. "We will anchor the car seat to the car's anchoring system."

ANS: C Parents need to be taught to always use the restraint even for short trips. Further teaching is needed if they make this statement. Parents have understood the teaching if they encourage the child to help attach buckles, straps, and shields but always double-check fastenings; do not start the car until everyone is properly restrained; and anchor the car safety seat securely to the car's anchoring system and apply the harness snugly to the child.

A 4-year-old child tells the nurse that she does not want another blood sample drawn because "I need all my insides, and I don't want anyone taking them out." Which is the nurse's best interpretation of this? a. Child is being overly dramatic. b. Child has a disturbed body image. c. Preschoolers have poorly defined body boundaries. d. Preschoolers normally have a good understanding of their bodies.

ANS: C Preschoolers have little understanding of body boundaries, which leads to fears of mutilation. The child is not capable of being dramatic at 4 years of age. She truly has fear. Body image is just developing in the school-age child. Preschoolers do not have good understanding of their bodies.

37. During the first few days after surgery for cleft lip, which intervention should the nurse do? a. Leave infant in crib at all times to prevent suture strain. b. Keep infant heavily sedated to prevent suture strain. c. Remove restraints periodically to cuddle infant. d. Alternate position from prone to side-lying to supine.

ANS: C Remove restraints periodically, while supervising the infant, to allow him or her to exercise arms and to provide cuddling and tactile stimulation. The infant should not be left in the crib, but should be removed for appropriate holding and stimulation. Analgesia and sedation are administered for pain. Heavy sedation is not indicated. The child should not be placed in the prone position.

7. Which pathogen is the viral pathogen that frequently causes acute diarrhea in young children? a. Giardia organisms b. Shigella organisms c. Rotavirus d. Salmonella organisms

ANS: C Rotavirus is the most frequent viral pathogen that causes diarrhea in young children. Giardia (parasite) and Salmonella are bacterial pathogens that cause diarrhea. Shigella is a bacterial pathogen that is uncommon in the United States.

Which type of traction uses skin traction on the lower leg and a padded sling under the knee? a. Dunlop b. Bryant c. Russell d. Buck extension

ANS: C Russell traction uses skin traction on the lower leg and a padded sling under the knee. The combination of longitudinal and perpendicular traction allows realignment of the lower extremity and immobilizes the hips and knees in a flexed position. Dunlop traction is an upper-extremity traction used for fractures of the humerus. Bryant traction is skin traction with the legs flexed at a 90-degree angle at the hip. Buck extension traction is a type of skin traction with the legs in an extended position. It is used primarily for short-term immobilization, preoperatively with dislocated hips, for correcting contractures, or for bone deformities such as Legg-Calvé-Perthes disease.

10. An 8-year-old girl asks the nurse how the blood pressure apparatus works. The most appropriate nursing action is to: a. ask her why she wants to know. b. determine why she is so anxious. c. explain in simple terms how it works. d. tell her she will see how it works as it is used.

ANS: C School-age children require explanations and reasons for everything. They are interested in the functional aspect of all procedures, objects, and activities. It is appropriate for the nurse to explain how equipment works and what will happen to the child so that the child can then observe during the procedure. A nurse should respond positively for requests for information about procedures and health information. By not responding, the nurse may be limiting communication with the child. The child is not exhibiting anxiety in asking how the blood pressure apparatus works, just requesting clarification of what will occur.

A child is brought to the emergency department after experiencing a seizure at school. There is no previous history of seizures. The father tells the nurse that he cannot believe the child has epilepsy. The nurse's best response is which of the following? a. "Epilepsy is easily treated." b. "Very few children have actual epilepsy." c. "The seizure may or may not mean that your child has epilepsy." d. "Your child has had only one convulsion; it probably won't happen again."

ANS: C Seizures are the indispensable characteristic of epilepsy; however, not every seizure is epileptic. Epilepsy is a chronic seizure disorder with recurrent and unprovoked seizures.

A neonate is born with bilateral mild talipes equinovarus (clubfoot). When the parents ask the nurse how this will be corrected, the nurse should give which explanation? a. Traction is tried first. b. Surgical intervention is needed. c. Frequent, serial casting is tried first. d. Children outgrow this condition when they learn to walk.

ANS: C Serial casting is begun shortly after birth before discharge from nursery. Successive casts allow for gradual stretching of skin and tight structures on the medial side of the foot. Manipulation and casting of the leg are repeated frequently (every week) to accommodate the rapid growth of early infancy. Serial casting is the preferred treatment. Surgical intervention is done only if serial casting is not successful. Children do not improve without intervention.

23. When caring for a child with probable appendicitis, the nurse should be alert to recognize that which condition or symptom is a sign of perforation? a. Bradycardia b. Anorexia c. Sudden relief from pain d. Decreased abdominal distention

ANS: C Signs of peritonitis, in addition to fever, include sudden relief from pain after perforation. Tachycardia, not bradycardia, is a manifestation of peritonitis. Anorexia is already a clinical manifestation of appendicitis. Abdominal distention usually increases.

18. A 3-year-old child with Hirschsprung disease is hospitalized for surgery. A temporary colostomy will be necessary. The nurse should recognize that preparing this child psychologically is: a. not necessary because of child's age. b. not necessary because colostomy is temporary. c. necessary because it will be an adjustment. d. necessary because the child must deal with a negative body image.

ANS: C The child's age dictates the type and extent of psychological preparation. When a colostomy is performed, the child who is at least preschool age is told about the procedure and what to expect in concrete terms, with the use of visual aids. It is necessary to prepare a 3-year-old child for procedures. The preschooler is not yet concerned with body image.

41. Which observation made of the exposed abdomen is most indicative of pyloric stenosis? a. Abdominal rigidity b. Substernal retraction c. Palpable olive-like mass d. Marked distention of lower abdomen

ANS: C The diagnosis of pyloric stenosis is often made after the history and physical examination. The olive-like mass is easily palpated when the stomach is empty, the infant is quiet, and the abdominal muscles are relaxed. Abdominal rigidity and substernal retraction are usually not present. The upper abdomen, not lower abdomen, is distended.

Which of the following is characteristic of the immune-mediated type 1 diabetes mellitus? a. Ketoacidosis is infrequent. b. Onset is gradual. c. Age at onset is usually younger than 20 years. d. Oral agents are often effective for treatment.

ANS: C The immune-mediated type 1 diabetes mellitus typically has its onset in children or young adults.

The nurse is caring for a school-age child diagnosed with juvenile idiopathic arthritis (JIA). Which intervention should be a priority? a. Apply ice packs to relieve stiffness and pain. b. Administer acetaminophen to reduce inflammation. c. Teach the child and family correct administration of medications. d. Encourage range-of-motion exercises during periods of inflammation.

ANS: C The management of JIA is primarily pharmacologic. The family should be instructed regarding administration of medications and the value of regular schedule of administration to maintain a satisfactory blood level in the body. They need to know that NSAIDs should not be given on an empty stomach and to be alert for signs of toxicity. Warm moist heat is best for relieving stiffness and pain. Acetaminophen does not have antiinflammatory effects. Range-of-motion exercises should not be done during periods of inflammation.

21. A histamine-receptor antagonist such as cimetidine (Tagamet) or ranitidine (Zantac) is ordered for an infant with GER. The purpose of this is to: a. prevent reflux. b. prevent hematemesis. c. reduce gastric acid production. d. increase gastric acid production.

ANS: C The mechanism of action of histamine-receptor antagonists is to reduce the amount of acid present in gastric contents and perhaps prevent esophagitis. Preventing reflux or hematemesis and increasing gastric acid production are not the modes of action of histamine-receptor antagonists

A toddler's parent asks the nurse for suggestions on dealing with temper tantrums. Which is the most appropriate recommendation? a. Punish the child. b. Leave the child alone until the tantrum is over. c. Remain close by the child but without eye contact. d. Explain to child that this is wrong.

ANS: C The parent should be told that the best way to deal with temper tantrums is to ignore the behaviors, provided that the actions are not dangerous to the child. Tantrums are common in toddlers as the child becomes more independent and overwhelmed by increasingly complex tasks. The parents and caregivers need to have consistent and developmentally appropriate expectations. Punishment and explanations will not be beneficial. The parent's presence is necessary both for safety and to provide a feeling of control and security to the child when the tantrum is over.

A 5-year-old girl sustained a concussion when she fell out of a tree. In preparation for discharge, the nurse is discussing home care with her mother. Which of the following statements made by the mother indicates a correct understanding of the teaching? a. "I should expect my child to have a few episodes of vomiting." b. "If I notice sleep disturbances, I should contact the physician immediately." c. "I should expect my child to have some behavioral changes after the accident." d. "If I notice diplopia, I will have my child rest for 1 hour."

ANS: C The parents are advised of probable posttraumatic symptoms that may be expected. These include behavioral changes and sleep disturbances.

A nurse is conducting discharge teaching for parents of an infant with osteogenesis imperfecta (OI). Further teaching is indicated if the parents make which statement? a. "We will be very careful handling the baby." b. "We will lift the baby by the buttocks when diapering." c. "We're glad there is a cure for this disorder." d. "We will schedule follow-up appointments as instructed."

ANS: C The treatment for OI is primarily supportive. Although patients and families are optimistic about new research advances, there is no cure. The use of bisphosphonate therapy with IV pamidronate to promote increased bone density and prevent fractures has become standard therapy for many children with OI; however, long bones are weakened by prolonged treatment. Infants and children with this disorder require careful handling to prevent fractures. They must be supported when they are being turned, positioned, moved, and held. Even changing a diaper may cause a fracture in severely affected infants. These children should never be held by the ankles when being diapered but should be gently lifted by the buttocks or supported with pillows. Follow-up appointments for treatment with bisphosphonate can be expected.

Glucocorticoids, mineralocorticoids, and sex steroids are secreted by the: a. thyroid gland. b. parathyroid glands. c. adrenal cortex. d. anterior pituitary.

ANS: C These hormones are secreted by the adrenal cortex.

The nurse is caring for a 12-year-old child with a left leg below the knee amputation (BKA). The child had the surgery 1 week ago. Which intervention should the nurse plan to implement for this child? a. Elevate the left stump on a pillow. b. Place ice pack on the stump. c. Encourage the child to use an overhead bed trapeze when repositioning. d. Replace the ace wrap covering the stump with a gauze dressing.

ANS: C Use of the overhead bed trapeze should be encouraged to begin to build up the arm muscles necessary for walking with crutches. Stump elevation may be used during the first 24 hours, but after this time, the extremity should not be left in this position because contractures in the proximal joint will develop and seriously hamper ambulation. Ice would not be an appropriate intervention and would decrease circulation to the stump. Stump shaping is done postoperatively with special elastic bandaging using a figure-eight bandage, which applies pressure in a cone-shaped fashion. This technique decreases stump edema, controls hemorrhage, and aids in developing desired contours so the child will bear weight on the posterior aspect of the skin flap rather than on the end of the stump. This wrap should not be replaced with a gauze dressing.

42. The nurse is caring for an infant with suspected pyloric stenosis. Which clinical manifestation would indicate pyloric stenosis? a. Abdominal rigidity and pain on palpation b. Rounded abdomen and hypoactive bowel sounds c. Visible peristalsis and weight loss d. Distention of lower abdomen and constipation

ANS: C Visible gastric peristaltic waves that move from left to right across the epigastrium and weight loss are observed in pyloric stenosis. Abdominal rigidity and pain on palpation or a rounded abdomen and hypoactive bowel sounds are usually not present. The upper abdomen, not lower abdomen, is distended.

A child with hypoparathyroidism is receiving vitamin D therapy. The parents should be advised to watch for which of the following signs of vitamin D toxicity? a. Headache and seizures b. Physical restlessness and voracious appetite without weight gain c. Weakness and lassitude d. Anorexia and insomnia

ANS: C Vitamin D toxicity can be a serious consequence of therapy. Parents are advised to watch for signs, including weakness, fatigue, lassitude, headache, nausea, vomiting, and diarrhea. Renal impairment is manifested through polyuria, polydipsia, and nocturia.

Which statement is the most accurate about moral development in the 9-year-old school-age child? a. Right and wrong are based on physical consequences of behavior. b. The child obeys parents because of fear of punishment. c. The school-age child conforms to rules to please others. d. Parents are the determiners of right and wrong for the school-age child.

ANS: C Feedback A Children 4 to 7 years of age base right and wrong on consequences. B Consequences are the most important consideration for the child between 4 and 7 years of age. C The 7- to 12-year-old child bases right and wrong on a good-boy or good-girl orientation in which the child conforms to rules to please others and avoid disapproval. D Parents determine right and wrong for the child younger than 4 years of age.

Why do peer relationships become more important during adolescence? a. Adolescents dislike their parents. b. Adolescents no longer need parental control. c. They provide adolescents with a feeling of belonging. d. They promote a sense of individuality in adolescents.

ANS: C Feedback A During adolescence, the parent/child relationship changes from one of protection-dependency to one of mutual affection and quality. B Parents continue to play an important role in the personal and health-related decisions. C The peer group serves as a strong support to teenagers, providing them with a sense of belonging and a sense of strength and power. D The peer group forms the transitional world between dependence and autonomy.

In general, the earliest age at which puberty begins is ____ years in girls and _____ years in boys. a. 13; 13 b. 11; 11 c. 10; 12 d. 12; 10

ANS: C Feedback A Girls and boys do not usually begin puberty at the same age; girls usually begin earlier than boys. B Girls and boys do not usually begin puberty at the same age; girls usually begin earlier than boys. C Puberty signals the beginning of the development of secondary sex characteristics. This begins in girls earlier than in boys. Usually, there is a 2-year difference in the age at onset. D Girls and boys do not usually begin puberty at the same age; girls usually begin earlier than boys.

Which statement by the nurse is most appropriate to a 15-year-old whose friend has mentioned suicide? a. "Tell your friend to come to the clinic immediately." b. "You need to gather details about your friend's suicide plan." c. "Your friend's threat needs to be taken seriously, and immediate help for your friend is important." d. "If your friend mentions suicide a second time, you will want to get your friend some help."

ANS: C Feedback A Instructing a 15-year-old to tell a friend to come to the clinic immediately provides the teen with limited information and does not address the concern. B It is important to determine whether a person threatening suicide has a plan of action; however, the best information for the 15-year-old to have is that all threats of suicide should be taken seriously and immediate help is important. C Suicide is the third most common cause of death among American adolescents. A suicide threat from an adolescent serves as a dramatic message to others and should be taken seriously. Adolescents at risk should be targeted for supportive guidance and counseling before a crisis occurs. D It is imperative that help is provided immediately for a teenager who is talking about suicide. Waiting until the teen discusses it a second time may be too late.

A group of boys ages 9 and 10 years have formed a "boys-only" club that is open to neighborhood and school friends who have skateboards. This should be interpreted as a. Behavior that encourages bullying and sexism b. Behavior that reinforces poor peer relationships c. Characteristic of social development of this age d. Characteristic of children who later are at risk for membership in gangs

ANS: C Feedback A Peer-group identification and association are essential to a child's socialization. Poor relationships with peers and a lack of group identification can contribute to bullying. B Peer-group identification and association are essential to a child's socialization. Poor relationships with peers and a lack of group identification can contribute to bullying. C One of the outstanding characteristics of middle childhood is the creation of formalized groups or clubs. D A boys-only club does not have a direct correlation with later gang activity.

The nurse is explaining Tanner staging to an adolescent and her mother. Which statement best describes Tanner staging? a. Predictable stages of puberty that are based on chronologic age b. Staging of puberty based on the initiation of menarche and nocturnal emissions c. Predictable stages of puberty that are based on primary and secondary sexual characteristics d. Staging of puberty based on the initiation of primary sexual characteristics

ANS: C Feedback A Tanner stages are not based on chronologic age. The age at which an adolescent enters puberty is variable. B The puberty stage in girls begins with breast development. Puberty stage in boys begins with genital enlargement. C Tanner sexual-maturing ratings are based on the development of stages of primary and secondary sexual characteristics. D Primary sexual characteristics are not the basis of Tanner staging.

25. A nurse is counseling parents of a child beginning to show signs of being overweight. The nurse accurately relates which body mass index (BMI)-for-age percentile indicates a risk for being overweight? a. 10th percentile b. 9th percentile c. 85th percentile d. 95th percentile

ANS: C Children who have BMI-for-age greater than or equal to the 85th percentile and less than the 95th percentile are at risk for being overweight. Children in the 9th and 10th percentiles are within normal limits. Children who are greater than or equal to the 95th percentile are considered overweight.

19. When interviewing the mother of a 3-year-old child, the nurse asks about developmental milestones such as the age of walking without assistance. This should be considered: a. unnecessary information because child is age 3 years. b. an important part of the family history. c. an important part of the child's past history . d. an important part of the child's review of systems.

ANS: C Information about the attainment of developmental milestones is important to obtain. It provides data about the child's growth and development that should be included in the past history. Developmental milestones provide important information about the child's physical, social, and neurologic health and should be included in the history for a 3-year-old child. If pertinent, attainment of milestones by siblings would be included in the family history. The review of systems does not include the developmental milestones

A nurse is preparing to perform a physical assessment on a toddler. Which approach should the nurse use for this child? a. Always proceed in a head-to-toe direction. b. Perform traumatic procedures first. c. Use minimal physical contact initially . d. Demonstrate use of equipment.

ANS: C Parents can remove clothing, and the child can remain on the parent's lap. The nurse should use minimal physical contact initially to gain the child's cooperation. The head-to-toe assessment can be done in older children but usually must be adapted in younger children. Traumatic procedures should always be performed last. These will most likely upset the child and inhibit cooperation. The nurse should introduce the equipment slowly. The child can inspect the equipment, but demonstrations are usually too complex for toddlers.

33. Where is the best place to observe for the presence of petechiae in dark-skinned individuals? a. Face b. Buttocks c. Oral mucosa d. Palms and soles

ANS: C Petechiae, small distinct pinpoint hemorrhages, are difficult to see in dark skin unless they are in the mouth or conjunctiva

When doing a nutritional assessment on a Hispanic family, the nurse learns that their diet consists mainly of vegetables, legumes, and starches. The nurse should recognize that this diet: a. indicates they live in poverty. b. is lacking in protein. c. may provide sufficient amino acids. d. should be enriched with meat and milk.

ANS: C The diet that contains vegetable, legumes, and starches may provide sufficient essential amino acids, even though the actual amount of meat or dairy protein is low. Many cultures use diets that contain this combination of foods. It is not indicative of poverty. Combinations of foods contain the essential amino acids necessary for growth. A dietary assessment should be done, but many vegetarian diets are sufficient for growth

4. . What is the single most important factor to consider when communicating with children? a. The child's physical condition b. Presence or absence of the child's parent c. The child's developmental level d. The child's nonverbal behavior

ANS: C The nurse must be aware of the child's developmental stage to engage in effective communication. The use of both verbal and nonverbal communication should be appropriate to the developmental level. Although the child's physical condition is a consideration, developmental level is much more important. The parents' presence is important when communicating with young children but may be detrimental when speaking with adolescents. Nonverbal behaviors will vary in importance, based on the child's developmental level

29. A nurse notes that a 10-month-old infant has a larger head circumference than chest. The nurse interprets this as a normal finding because the head and chest circumference become equal at which age? a. 1 month b. 6 to 9 months c. 1 to 2 years d. 2 1/2 to 3 years

ANS: C Head circumference begins larger than chest circumference. Between ages 1 and 2 years, they become approximately equal. Head circumference is larger than chest circumference before age 1. Chest circumference is larger than head circumference at 2 1/2 to 3 years.

9. A nurse is assigned to four children of different ages. In which age group should the nurse understand that body integrity is a concern? a. Toddler b. Preschooler c. School-age child d. Adolescent

ANS: C School-age children have a heightened concern about body integrity. They place importance and value on their bodies and are oversensitive to anything that constitutes a threat or suggestion of injury. Body integrity is not as important a concern to toddlers, preschoolers, or adolescents

36. During a routine health assessment, the nurse notes that an 8-month-old infant has significant head lag. Which is the nurse's most appropriate action? a. Teach parents appropriate exercises. b. Recheck head control at next visit . c. Refer child for further evaluation. d. Refer child for further evaluation if anterior fontanel is still open

ANS: C Significant head lag after age 6 months strongly indicates cerebral injury and is referred for further evaluation. Reduction of head lag is part of normal development. Exercises will not be effective. The lack of achievement of this developmental milestone must be evaluated

Which of the following is a clinical manifestation of increased intracranial pressure (ICP) in infants? (Select all that apply.) a. Low-pitched cry b. Sunken fontanel c. Diplopia and blurred vision d. Irritability e. Distended scalp veins f. Increased blood pressure

ANS: C, D, E Diplopia and blurred vision, irritability, and distended scalp veins are signs of increased ICP in infants.

5. A nurse is conducting dietary teaching on high-fiber foods for parents of a child with constipation. Which foods should the nurse include as being high in fiber? (Select all that apply.) a. White rice b. Avocados c. Whole grain breads d. Bran pancakes e. Raw carrots

ANS: C, D, E High-fiber foods include whole grain breads, bran pancakes, and raw carrots. Unrefined (brown) rice is high in fiber but not white rice. Raw fruits, especially those with skins or seeds, other than ripe banana or avocado are high in fiber.

The nurse is caring for a preschool child with a cast applied recently for a fractured tibia. Which assessment findings indicate possible compartment syndrome? (Select all that apply.) a. Palpable distal pulse b. Capillary refill to extremity less than 3 seconds c. Severe pain not relieved by analgesics d. Tingling of extremity e. Inability to move extremity

ANS: C, D, E Indications of compartment syndrome are severe pain not relieved by analgesics, tingling of extremity, and inability to move extremity. A palpable distal pulse and capillary refill to the extremity less than 3 seconds are expected findings.

39. Which type of hernia has an impaired blood supply to the herniated organ? a. Hiatal hernia b. Incarcerated hernia c. Omphalocele d. Strangulated hernia

ANS: D A strangulated hernia is one in which the blood supply to the herniated organ is impaired. Hiatal hernia is the intrusion of an abdominal structure, usually the stomach, through the esophageal hiatus. Incarcerated hernia is a hernia that cannot be reduced easily. Omphalocele is the protrusion of intra-abdominal viscera into the base of the umbilical cord. The sac is covered with peritoneum, not skin.

The nurse is guiding parents in selecting a daycare facility for their child. Which is especially important to consider when making the selection? a. Structured learning environment b. Socioeconomic status of children c. Cultural similarities of children d. Teachers knowledgeable about development

ANS: D A teacher knowledgeable about development will structure activities for learning. A structured learning environment is not necessary at this age. Socioeconomic status is not the most important factor in selecting a preschool. Preschool is about expanding experiences with others, so cultural similarities are not necessary

Which of the following may be beneficial in reducing the risk of Reye syndrome? a. Immunization against the disease b. Medical attention for all head injuries c. Prompt treatment of bacterial meningitis d. Avoidance of aspirin and ibuprofen for children with varicella or those suspected of having influenza

ANS: D Although the etiology of Reye syndrome is obscure, most cases follow a common viral illness, either varicella or influenza. A potential association exists between aspirin therapy and the development of Reye syndrome, so use of aspirin is avoided.

At what age should the nurse expect a child to give both first and last names when asked? a. 15 months b. 18 months c. 24 months d. 30 months

ANS: D At 30 months, the child is able to give both first and last names and refer to self with an appropriate pronoun. At 15 and 18 months, the child is too young to give his or her own name. At 24 months, the child is able to give first name and refer to self by that name.

Which is a type of skin traction with legs in an extended position? a. Dunlop b. Bryant c. Russell d. Buck extension

ANS: D Buck extension traction is a type of skin traction with the legs in an extended position. It is used primarily for short-term immobilization, preoperatively with dislocated hips, for correcting contractures, or for bone deformities such as Legg-Calvé-Perthes disease. Dunlop traction is an upper-extremity traction used for fractures of the humerus. Bryant traction is skin traction with the legs flexed at a 90-degree angle at the hip. Russell traction uses skin traction on the lower leg and a padded sling under the knee. The combination of longitudinal and perpendicular traction allows realignment of the lower extremity and immobilizes the hips and knees in a flexed position.

Children taking phenobarbital and/or phenytoin may experience a deficiency of: a. calcium. b. vitamin C. c. fat-soluble vitamins. d. vitamin D and folic acid.

ANS: D Deficiencies of vitamin D and folic acid have been reported in children taking phenobarbital and phenytoin.

11. A school-age child with diarrhea has been rehydrated. The nurse is discussing the child's diet with the family. Which statement by the parent would indicate a correct understanding of the teaching? a. "I will keep my child on a clear liquid diet for the next 24 hours." b. "I should encourage my child to drink carbonated drinks but avoid food for the next 24 hours." c. "I will offer my child bananas, rice, applesauce, and toast for the next 48 hours." d. "I should have my child eat a normal diet with easily digested foods for the next 48 hours."

ANS: D Easily digested foods such as cereals, cooked vegetables, and meats should be provided for the child. Early reintroduction of nutrients is desirable. Continued feeding or reintroduction of a regular diet has no adverse effects and actually lessens the severity and duration of the illness. Clear liquids and carbonated drinks have high carbohydrate content and few electrolytes. Caffeinated beverages should be avoided because caffeine is a mild diuretic. The BRAT diet has little nutritional value and is high in carbohydrates.

The nurse recommends to parents that peanuts are not a good snack food for toddlers. The nurse's rationale for this action is that they: a. are low in nutritive value. b. are high in sodium. c. cannot be entirely digested. d. can be easily aspirated.

ANS: D Foreign-body aspiration is common during the second year of life. Although they chew well, this age child may have difficulty with large pieces of food, such as meat and whole hot dogs, and with hard foods, such as nuts or dried beans. Peanuts have many beneficial nutrients, but should be avoided because of the risk of aspiration in this age group. The sodium level may be a concern, but the risk of aspiration is more important. Many foods pass through the gastrointestinal tract incompletely undigested. This is not necessarily detrimental to the child.

The mother of a 1-month-old infant tells the nurse she worries that her baby will get meningitis like her oldest son did when he was an infant. The nurse should base her response on which of the following? a. Meningitis rarely occurs during infancy. b. Often a genetic predisposition to meningitis is found. c. Vaccination to prevent all types of meningitis is now available. d. Vaccination to prevent Haemophilus influenzae type b meningitis has decreased the frequency of this disease in children.

ANS: D H. influenzae type b meningitis has been virtually eradicated in areas of the world where the vaccine is administered routinely.

Which of the following is considered a cardinal sign of diabetes mellitus? a. Nausea b. Seizures c. Impaired vision d. Frequent urination

ANS: D Hallmarks of diabetes mellitus are glycosuria, polyuria, and polydipsia.

4. A nurse is admitting an infant with dehydration caused from water loss in excess of electrolyte loss. Which type of dehydration is this infant experiencing? a. Isotonic b. Isosmotic c. Hypotonic d. Hypertonic

ANS: D Hypertonic dehydration results from water loss in excess of electrolyte loss. This is the most dangerous type of dehydration. It is caused by feeding children fluids with high amounts of solute. Isotonic dehydration occurs in conditions in which electrolyte and water deficits are present in balanced proportion and is another term for isotonic dehydration. Hypotonic dehydration occurs when the electrolyte deficit exceeds the water deficit, leaving the serum hypotonic.

Which should cause a nurse to suspect that an infection has developed under a cast? a. Complaint of paresthesia b. Cold toes c. Increased respirations d. "Hot spots" felt on cast surface

ANS: D If hot spots are felt on the cast surface, they usually indicate infection beneath the area. This should be reported so that a window can be made in the cast to observe the site. The five Ps of ischemia from a vascular injury are pain, pallor, pulselessness, paresthesia, and paralysis. Paresthesia is an indication of vascular injury, not infection. Cold toes may be indicative of too tight a cast and need further evaluation. Increased respirations may be indicative of a respiratory tract infection or pulmonary emboli. This should be reported, and child should be evaluated.

The psychosocial developmental tasks of toddlerhood include which characteristic? a. Development of a conscience b. Recognition of sex differences c. Ability to get along with age-mates d. Ability to delay gratification

ANS: D If the need for basic trust has been satisfied, then toddlers can give up dependence for control, independence, and autonomy. One of the tasks that the toddler is concerned with is the ability to delay gratification. Development of a conscience occurs during the preschool years. The recognition of sex differences occurs during the preschool years. The ability to get along with age-mates develops during the preschool and school-age years.

44. Invagination of one segment of bowel within another is called: a. atresia. b. stenosis. c. herniation. d. intussusception

ANS: D Intussusception occurs when a proximal section of the bowel telescopes into a more distal segment, pulling the mesentery with it. The mesentery is compressed and angled, resulting in lymphatic and venous obstruction. Invagination of one segment of bowel within another is the definition of intussusception, not atresia, stenosis, or herniation.

The nurse is taking care of an adolescent diagnosed with kyphosis. Which describes this condition? a. Lateral curvature of the spine b. Immobility of the shoulder joint c. Exaggerated concave lumbar curvature of the spine d. Increased convex angulation in the curve of the thoracic spine

ANS: D Kyphosis is an abnormally increased convex angulation in the curve of the thoracic spine. Scoliosis is a complex spinal deformity usually involving lateral curvature, spinal rotation causing rib asymmetry, and thoracic hypokyphosis. Ankylosis is the immobility of a joint. Lordosis is an exaggerated concave lumbar curvature of the spine.

The nurse should recommend medical attention if a child with a slight head injury experiences which of the following? a. Sleepiness b. Vomiting, even once c. Headache, even if slight d. Confusion or abnormal behavior

ANS: D Medical attention should be sought if the child exhibits confusion or abnormal behavior, loses consciousness, has amnesia, has fluid leaking from the nose or ears, complains of blurred vision, or has an unsteady gait.

16. Which therapeutic management treatment is implemented for children with Hirschsprung disease? a. Daily enemas b. Low-fiber diet c. Permanent colostomy d. Surgical removal of affected section of bowel

ANS: D Most children with Hirschsprung disease require surgical rather than medical management. Surgery is done to remove the aganglionic portion of the bowel, relieve obstruction, and restore normal bowel motility and function of the internal anal sphincter. Preoperative management may include enemas and low-fiber, high-calorie, high-protein diet, until the child is physically ready for surgery. The colostomy that is created in Hirschsprung disease is usually temporary.

Which medication is usually tried first when a child is diagnosed with juvenile idiopathic arthritis (JIA)? a. Aspirin b. Corticosteroids c. Cytotoxic drugs such as methotrexate d. Nonsteroidal anti-inflammatory drugs (NSAIDs)

ANS: D NSAIDs are the first drugs used in JIA. Naproxen, ibuprofen, and tolmetin are approved for use in children. Aspirin, once the drug of choice, has been replaced by the NSAIDs because they have fewer side effects and easier administration schedules. Corticosteroids are used for life-threatening complications, incapacitating arthritis, and uveitis. Methotrexate is a second-line therapy for JIA.

9. A child is admitted with bacterial gastroenteritis. Which lab results of a stool specimen confirm this diagnosis? a. Eosinophils b. Occult blood c. pH less than 6 d. Neutrophils and red blood cells

ANS: D Neutrophils and red blood cells in stool indicate bacterial gastroenteritis. Protein intolerance and parasitic infections are suspected in the presence of eosinophils. Occult blood may indicate pathogens such as Shigella, Campylobacter, or hemorrhagic Escherichia coli strains. A pH of less than 6 may indicate carbohydrate malabsorption or secondary lactase insufficiency.

22. Which clinical manifestation would be the most suggestive of acute appendicitis? a. Rebound tenderness b. Bright red or dark red rectal bleeding c. Abdominal pain that is relieved by eating d. Abdominal pain that is most intense at McBurney point

ANS: D Pain is the cardinal feature. It is initially generalized, usually periumbilical. The pain localizes to the right lower quadrant at McBurney point. Rebound tenderness is not a reliable sign and is extremely painful to the child. Bright red or dark red rectal bleeding and abdominal pain that is relieved by eating are not signs of acute appendicitis.

Which of the following clinical manifestations may occur in the child who is receiving too much propylthiouracil for the treatment of hyperthyroidism (Graves disease)? a. Seizures b. Enlargement of all lymph glands c. Pancreatitis or cholecystitis d. Lethargy and somnolence

ANS: D Parents should be aware of the signs of hypothyroidism that can occur from overdosage of the drug. The most common manifestations are lethargy and somnolence.

Type 1 diabetes mellitus is suspected in an adolescent. Which of the following clinical manifestations may be present? a. Moist skin b. Weight gain c. Fluid overload d. Poor wound healing

ANS: D Poor wound healing may be present in an individual with type 1 diabetes mellitus.

Manifestations of hypoglycemia include which of the following? a. Lethargy b. Thirst c. Nausea and vomiting d. Shaky feeling and dizziness

ANS: D Some of the clinical manifestations of hypoglycemia include shaky feelings; dizziness; difficulty concentrating, speaking, focusing, or coordinating; sweating; and pallor.

A nurse is planning care for a 17-month-old child. According to Piaget, which stage should the nurse expect the child to be in cognitively? a. Trust b. Preoperational c. Secondary circular reaction d. Tertiary circular reaction

ANS: D The 17-month-old child is in the fifth stage of the sensorimotor phase, tertiary circular reactions. The child uses active experimentation to achieve previously unattainable goals. Trust is Erikson's first stage. Preoperational is the stage of cognitive development usually present in older toddlers and preschoolers. Secondary circular reactions last from about ages 4 to 8 months.

Which characteristic best describes the gross motor skills of a 24-month-old child? a. Skips and can hop in place on one foot b. Rides tricycle and broad jumps c. Jumps with both feet and stands on one foot momentarily d. Walks up and down stairs and runs with a wide stance

ANS: D The 24-month-old child can go up and down stairs alone with two feet on each step and runs with a wide stance. Skipping and hopping on one foot are achieved by 4-year-old children. Jumping with both feet and standing on one foot momentarily are achieved by 30-month-old children. Tricycle riding and broad jumping are achieved at age 3.

The parents of a newborn say that their toddler "hates the baby; he suggested that we put him in the trash can so the trash truck could take him away." Which is the nurse's best reply? a. "Let's see if we can figure out why he hates the new baby." b. "That's a strong statement to come from such a small boy." c. "Let's refer him to counseling to work this hatred out. It's not a normal response." d. "That is a normal response to the birth of a sibling. Let's look at ways to deal with this."

ANS: D The arrival of a new infant represents a crisis for even the best-prepared toddler. Toddlers have their entire schedule and routines disrupted because of the new family member. The nurse should work with parents on ways to involve the toddler in the newborn's care and to help focus attention on the toddler. The toddler does not hate the infant. This is an expected response to the changes in routines and attention that affect the toddler. The toddler can be provided with a doll to tend to the doll's needs at the same time the parent is performing similar care for the newborn.

A father tells the nurse that his daughter wants the same plate and cup used at every meal, even if they go to a restaurant. The nurse should explain that this is: a. a sign the child is spoiled. b. a way to exert unhealthy control. c. regression, common at this age. d. ritualism, common at this age.

ANS: D The child is exhibiting the ritualism that is characteristic at this age. Ritualism is the need to maintain sameness and reliability. It provides a sense of comfort to the toddler. It will dictate certain principles in feeding practices, including rejecting a favorite food because it is served in a different container. Ritualism is not indicative of a child who has unreasonable expectations, but rather normal development. Toddlers use ritualistic behaviors to maintain necessary structure in their lives. This is not regression, which is a retreat from a present pattern of functioning.

A 4-year-old child is newly diagnosed with Legg-Calvé-Perthes disease. Nursing considerations should include which action? a. Encouraging normal activity for as long as is possible b. Explaining the cause of the disease to the child and family c. Preparing the child and family for long-term, permanent disabilities d. Teaching the family the care and management of the corrective appliance

ANS: D The family needs to learn the purpose, function, application, and care of the corrective device and the importance of compliance to achieve the desired outcome. The initial therapy is rest and non-weight bearing, which helps reduce inflammation and restore motion. Legg-Calvé-Perthes is a disease with an unknown etiology. A disturbance of circulation to the femoral capital epiphysis produces an ischemic aseptic necrosis of the femoral head. The disease is self-limiting, but the ultimate outcome of therapy depends on early and efficient therapy and the child's age at onset.

The nurse is conducting teaching to parents of a 7-year-old child who fractured an arm and is being discharged with a cast. Which instruction should be included in the teaching? a. Swelling of the fingers is to be expected for the next 48 hours. b. Immobilize the shoulder to decrease pain in the arm. c. Allow the affected limb to hang down for 1 hour each day. d. Elevate casted arm when resting and when sitting up.

ANS: D The injured extremity should be kept elevated while resting and in a sling when upright. This will increase venous return. Swelling of the fingers may indicate neurovascular damage and should be reported immediately. Permanent damage can occur within 6 to 8 hours. Joints above and below the cast on the affected extremity should be moved. The affected limb should not hang down for any length of time.

The nurse is assessing a child who was just admitted to the hospital for observation after a head injury. Which of the following is the most essential part of the nursing assessment to detect early signs of a worsening condition? a. Posturing b. Vital signs c. Focal neurologic signs d. Level of consciousness

ANS: D The most important nursing observation is assessment of the child's level of consciousness. Alterations in consciousness appear earlier in the progression of an injury than do alterations of vital signs or focal neurologic signs.

A parent asks the nurse about negativism in toddlers. Which is the most appropriate recommendation? a. Punish the child. b. Provide more attention. c. Ask child not always to say "no." d. Reduce the opportunities for a "no" answer.

ANS: D The nurse should suggest to the parent that questions be phrased with realistic choices rather than yes or no answers. This provides the toddler with a sense of control and reduces the opportunity for negativism. Negativism is not an indication of stubbornness or insolence and should not be punished. The negativism is not a function of attention; the child is testing limits to gain an understanding of the world. The toddler is too young to be asked to not always say "no."

The nurse is caring for a 4-year-old child immobilized by a fractured hip. Which complication should the nurse monitor related to the child's immobilization status? a. Metabolic rate increases b. Increased joint mobility leading to contractures c. Bone calcium increases, releasing excess calcium into the body (hypercalcemia) d. Venous stasis leading to thrombi or emboli formation

ANS: D The physiologic effects of immobilization, as a result of decreased muscle contraction, include venous stasis. This can lead to pulmonary emboli or thrombi. The metabolic rate decreases with immobilization. Loss of joint mobility leads to contractures. Bone demineralization with osteoporosis and hypercalcemia occur with immobilization.

In terms of fine motor development, which should the 3-year-old child be expected to do? a. Lace shoes and tie shoelaces with a bow. b. Use scissors to cut pictures, and print a few numbers. c. Draw a person with seven parts and correctly identify the parts. d. Draw a circle and name what has been drawn.

ANS: D Three-year-olds are able to accomplish this fine motor skill. Being able to lace shoes and tie shoelaces with a bow, use scissors to cut pictures, and print a few numbers, or draw a person with seven parts and correctly identify the parts are fine motor skills of 4- or 5-year-olds

Which should the nurse expect of a healthy 3-year-old child? a. Jump rope. b. Ride a two-wheel bicycle. c. Skip on alternate feet. d. Balance on one foot for a few seconds.

ANS: D Three-year-olds are able to accomplish this gross motor skill. Jumping rope, riding a two-wheel bicycle, and skipping on alternate feet are gross motor skills of 5-year-olds.

Which factor is most important in predisposing toddlers to frequent infections? a. Respirations are abdominal. b. Pulse and respiratory rates are slower than those in infancy. c. Defense mechanisms are less efficient than those during infancy. d. Toddlers have a short, straight internal ear canal and large lymph tissue.

ANS: D Toddlers continue to have the short, straight internal ear canal of infants. The lymphoid tissue of the tonsils and adenoids continues to be relatively large. These two anatomic conditions combine to predispose the toddler to frequent infections. The abdominal respirations and lowered pulse and respiratory rate of toddlers do not affect their susceptibility to infection. The defense mechanisms are more efficient compared with those of infancy

The parent of a child with diabetes mellitus asks the nurse when urine testing will be necessary. The nurse should explain that urine testing for: a. glucose is needed before administration of insulin. b. glucose is needed four times a day. c. glycosylated hemoglobin is required. d. ketonuria should be done when it is suspected.

ANS: D Urine testing is still performed to detect evidence of ketonuria.

34. Caring for the newborn with a cleft lip and palate before surgical repair includes: a. gastrostomy feedings. b. keeping infant in near-horizontal position during feedings. c. allowing little or no sucking. d. providing satisfaction of sucking needs.

ANS: D Using special or modified nipples for feeding techniques helps meet the infant's sucking needs. Gastrostomy feedings are usually not indicated. Feeding is best accomplished with the infant's head in an upright position. The child requires both nutritive and nonnutritive sucking.

A child is unconscious after a motor vehicle accident. The watery discharge from the nose tests positive for glucose. The nurse should recognize that this suggests: a. diabetic coma. b. brainstem injury. c. upper respiratory tract infection. d. leaking of cerebrospinal fluid (CSF).

ANS: D Watery discharge from the nose that is positive for glucose suggests leaking of CSF from a skull fracture.

The nurse uses the palms of the hands when handling a wet cast for which reason? a. To assess dryness of the cast b. To facilitate easy turning c. To keep the patient's limb balanced d. To avoid indenting the cast

ANS: D Wet casts should be handled by the palms of the hands, not the fingers, to avoid creating pressure points. Assessing dryness, facilitating easy turning, and keeping the patient's limb balanced are not reasons for using the palms of the hand rather than the fingers when handling a wet cast.

A school nurse is conducting a class on safety for a group of school-age children. Which statement indicates that the children may need further teaching? a. "My sister and I know two different ways to get out of the house." b. "I can dial 911 if there is a fire or a burglar in the house." c. "My mother has told us that if we have a fire, we have to meet at the neighbor's house." d. "If there is a fire I will have to go back in for my cat Fluffy because she will be scared."

ANS: D Feedback A All children should know two different escape routes from the house, in case one is blocked. B It is important for children to be taught how to call 911 in an emergency. C All families should have a predetermined meeting place away from the house. D Children should be taught never to return to a burning house, not even for a pet.

What is the best response a nurse can make to a 15-year-old girl who has verbalized a desire to have a baby? a. "Have you talked with your parents about this?" b. "Do you have plans to continue school?" c. "Will you be able to support the baby?" d. "Can you tell me how your life will be if you have an infant?"

ANS: D Feedback A Asking the teenager whether she has talked to her parents is not particularly helpful to the teen or the nurse and may terminate the communication. B A direct question about continuing school will not facilitate communication. Open-ended questions encourage communication. C Asking the teenager about how she will support the child will not facilitate communication. Open-ended questions encourage communication. D Having the teenager describe how the infant will affect her life will allow the teen to think more realistically. Her description will allow the nurse to assess the teen's perception and reality orientation.

The most common cause of death in the adolescent age-group involves a. Drownings b. Firearms c. Drug overdoses d. Motor vehicles

ANS: D Feedback A Drownings are major concerns in adolescence but do not cause the majority of deaths. B Firearms are major concerns in adolescence but do not cause the majority of deaths. C Drug overdoses are major concerns in adolescence but do not cause the majority of deaths. D Risk taking behaviors play a major role in the high incidence of motor vehicle injuries and death among teenagers i.e. alcohol use, failure to wear a seatbelt, and inexperience.

Which comment is most developmentally typical of a 7-year-old boy? a. "I am a Power Ranger, so don't make me angry." b. "I don't know whether I like Mary or Joan better." c. "My mom is my favorite person in the world." d. "Jimmy is my best friend."

ANS: D Feedback A Magical thinking is developmentally appropriate for the preschooler. B Opposite-sex friendships are not typical for the 7-year-old child. C Seven-year-old children socialize with their peers, not their parents. D School-age children form friendships with peers of the same sex, those who live nearby, and other children who have toys that they enjoy.

Which behavior is not demonstrated in the 8-year-old child? a. Understands that his or her point of view is not the only one b. Enjoys telling riddles and silly jokes c. Understands that pouring liquid from a small to large container does not change the amount d. Engages in fantasy and magical thinking

ANS: D Feedback A School-age children enter the stage of concrete operations. They learn that their point of view is not the only one. B The school-age child has a sense of humor. The child's increased language mastery and increased logic allow for appreciation of plays on words, jokes, and incongruities. C The school-age child understands that properties of objects do not change when their order, form, or appearance does. D The preschool-age child engages in fantasy and magical thinking. The school-age child moves away from this type of thinking and becomes more skeptical and logical. Belief in Santa Claus or the Easter Bunny ends in this period of development.

Which statement made by a mother of a school-age boy indicates a need for further teaching? a. "My child is playing soccer this year." b. "He is always busy with his friends playing games. He is very active." c. "I limit his television watching to about 2 hours a day." d. "I am glad his coach is a good role model. He emphasizes the importance of winning in today's society. The kids really are disciplined."

ANS: D Feedback A Team sports such as soccer are appropriate for exercise and refinement of motor skills. B School-age children need to participate in physical activities, which contribute to their physical fitness skills and well-being. C Limiting television to 2 hours a day is an appropriate restriction. School-age children should be encouraged to participate in physical activities. D Team sports are important for the development of sportsmanship and teamwork and for exercise and refinement of motor skills. A coach who emphasizes winning and strict discipline is not appropriate for children in this age-group.

When planning care for adolescents, the nurse should a. Teach parents first, and they, in turn, will teach the teenager. b. Provide information for their long-term health needs because teenagers respond best to long-range planning. c. Maintain the parents' role by providing explanations for treatment and procedures to the parents only. d. Give information privately to adolescents about how they can manage the specific problems that they identify.

ANS: D Feedback A Teenagers are socially and cognitively at the developmental stage where the health care provider can teach them. B Teenagers are more interested in immediate health care needs than in long-term needs. C Teenagers are at the developmental level that allows them to receive explanations about health care directly from the nurse. D Problems that teenagers identify and are interested in are typically the problems that they are the most willing to address. Confidentiality is important to adolescents. Adolescents prefer to confer privately (without parents) with the nurse and health care provider.

In providing anticipatory guidance to parents, which parental behavior is the most important in fostering moral development? a. Telling the child what is right and wrong b. Vigilantly monitoring the child and her peers c. Weekly family meetings to discuss behavior d. Living as the parents say they believe

ANS: D Feedback A Telling the child what is right and wrong is not effective unless the child has experienced what she hears. Parents need to live according to the values they are teaching to their children. B Vigilant monitoring of the child and her peers is an inappropriate action for the parent to initiate. It does not foster moral development and reasoning in the child. C Weekly family meetings to discuss behaviors may or may not be helpful in the development of moral reasoning. D Parents living what they believe gives nonambivalent messages and fosters the child's moral development and reasoning.

In girls, the initial indication of puberty is a. Menarche b. Growth spurt c. Growth of pubic hair d. Breast development

ANS: D Feedback A The usual sequence of secondary sexual characteristic development in girls is breast changes, rapid increase in height and weight, growth of pubic hair, appearance of axillary hair, menstruation, and abrupt deceleration of linear growth. B The usual sequence of secondary sexual characteristic development in girls is breast changes, rapid increase in height and weight, growth of pubic hair, appearance of axillary hair, menstruation, and abrupt deceleration of linear growth. C The usual sequence of secondary sexual characteristic development in girls is breast changes, rapid increase in height and weight, growth of pubic hair, appearance of axillary hair, menstruation, and abrupt deceleration of linear growth. D In most girls, the initial indication of puberty is the appearance of breast buds, an event known as thelarche.

26. The nurse is using the NCHS growth chart for an African-American child. Which statement should the nurse consider? a. This growth chart should not be used. b. Growth patterns of African-American children are the same as for all other ethnic groups. c. A correction factor is necessary when the NCHS growth chart is used for non-Caucasian ethnic groups. d. The NCHS charts are accurate for U.S. African-American children.

ANS: D The NCHS growth charts can serve as reference guides for all racial or ethnic groups. U.S. African-American children were included in the sample population. The growth chart can be used with the perspective that different groups of children have varying normal distributions on the growth curves. No correction factor exists

The number of hours spent sleeping decreases as the child grows older. Children ages 6 and 7 years require approximately 9 or 10 hours of sleep per night. Is this statement true or false?

ANS: F Children ages 6 and 7 actually need approximately 12 hours of sleep per night. Some children also continue to need an afternoon nap or quiet time to restore energy levels. The 12-year-old needs approximately 9 to 10 hours of sleep at night. Adequate sleep is important for school performance and physical growth. Inadequate sleep can cause irritability, inability to concentrate, and poor school performance.

The use of electronic or digital media for communication has had a negative effect on the language development of adolescents. Is this statement true or false?

ANS: T Text messaging, instant messaging, blogs, and Twitter all contribute to abbreviated communication techniques, which eliminate not only grammar and sentence construction, but also word development (e.g., using ur, for you are).

An appropriate nursing intervention when caring for a child with pneumonia is to:

Encourage rest. Lying on the affected side may promote comfort by splinting the chest and reducing pleural rubbing.

A child has a chronic, nonproductive cough and diffuse wheezing during the expiratory phase of respiration. This suggests:

Asthma.

Which consideration is the most important in managing tuberculosis (TB) in children?

Pharmacotherapy

The nurse recognizes that these symptoms are characteristic of which respiratory condition?

Sinusitis

It is now recommended that children with asthma who are taking long-term inhaled steroids should be assessed frequently because they may develop:

Slowed growth.

An infant has developed staphylococcal pneumonia. Nursing care of the child with pneumonia includes which of the following? Cluster care to conserve energy Administration of antibiotics

The nurse is caring for a 10-month-old infant with respiratory syncytial virus (RSV) bronchiolitis. Which interventions should be included in the child's care

A nurse is conducting an in-service on asthma. Which statement is the most descriptive of bronchial asthma? There is heightened airway reactivity.

There is heightened airway reactivity.

When both parents have sickle cell trait, which is the chance their children will have sickle cell anemia? a. 25% b. 50% c. 75% d. 100%

a. 25% Sickle cell anemia is inherited in an autosomal recessive pattern. If both parents have sickle cell trait (one copy of the sickle cell gene), then for each pregnancy, a 25% chance exists that their child will be affected with sickle cell disease. With each pregnancy, a 50% chance exists that the child will have sickle cell trait. Percentages of 75% and 100% are too high for the children of parents who have sickle cell trait.

The nurse is admitting a child with a Wilms tumor. Which is the initial assessment finding associated with this tumor? a. Abdominal swelling b. Weight gain c. Hypotension d. Increased urinary output

a. Abdominal swelling The initial assessment finding with a Wilms (kidney) tumor is abdominal swelling. Weight loss, not weight gain, may be a finding. Hypertension occasionally occurs with a Wilms tumor. Urinary output is not increased, but hematuria may be noted.

In which of the conditions are all the formed elements of the blood simultaneously depressed? a. Aplastic anemia b. Sickle cell anemia c. Thalassemia major d. Iron deficiency anemia

a. Aplastic anemia Aplastic anemia refers to a bone marrow-failure condition in which the formed elements of the blood are simultaneously depressed. Sickle cell anemia is a hemoglobinopathy in which normal adult hemoglobin is partly or completely replaced by abnormal sickle hemoglobin. Thalassemia major is a group of blood disorders characterized by deficiency in the production rate of specific hemoglobin globin chains. Iron deficiency anemia results in a decreased amount of circulating red cells.

The school nurse is informed that a child with human immunodeficiency virus (HIV) will be attending school soon. Which is an important nursing intervention? a. Carefully follow universal precautions. b. Determine how the child became infected. c. Inform the parents of the other children. d. Reassure other children that they will not become infected.

a. Carefully follow universal precautions. Universal precautions are necessary to prevent further transmission of the disease. It is not the role of the nurse to determine how the child became infected. Informing the parents of other children and reassuring children that they will not become infected is a violation of the child's right to privacy.

The head-to-tail direction of growth is referred to as: a. Cephalocaudal. c. Mass to specific. b. Proximodistal. d. Sequential.

a. Cephalocaudal.

The nurse is conducting a staff in-service on renal ultrasounds. Which statement describes this diagnostic test? a. Computed tomography uses external radiation to visualize the renal system. b. Visualization of the renal system is accomplished without exposure to radiation or radioactive isotopes. c. Contrast medium and x-rays allow for visualization of the renal system. d. External radiation for x-ray films is used to visualize the renal system, before, during, and after voiding.

a. Computed tomography uses external radiation to visualize the renal system. A renal ultrasound transmits ultrasonic waves through the renal parenchyma allowing for visualization of the renal system without exposure to external beam radiation or radioactive isotopes. Computed tomography uses external radiation and sometimes contrast media to visualize the renal system. An intravenous pyelogram uses contrast medium and external radiation for x-ray films. The voiding cystourethrogram visualizes the renal system with injection of a contrast media into the bladder through the urethral opening and use of x-ray before, during, and after voiding.

Which is instituted for the therapeutic management of minimal change nephrotic syndrome? a. Corticosteroids b. Antihypertensive agents c. Long-term diuretics d. Increased fluids to promote diuresis

a. Corticosteroids Corticosteroids are the first line of therapy for minimal change nephrotic syndrome. Response is usually seen within 7 to 21 days. Antihypertensive agents and long-term diuretic therapy are usually not necessary. A diet that has fluid and salt restrictions may be indicated.

The nurse closely monitors the temperature of a child with minimal change nephrotic syndrome. The purpose of this assessment is to detect an early sign of which possible complication? a. Infection b. Hypertension c. Encephalopathy d. Edema

a. Infection Infection is a constant source of danger to edematous children and those receiving corticosteroid therapy. An increased temperature could be an indication of an infection. Temperature is not an indication of hypertension or edema. Encephalopathy is not a complication usually associated with minimal change nephrotic syndrome. The child will most likely have neurologic signs and symptoms.

Parents of a child with sickle cell anemia ask the nurse, "What happens to the hemoglobin in sickle cell anemia?" Which statement by the nurse explains the disease process?" a. Normal adult hemoglobin is replaced by abnormal hemoglobin. b. There is a lack of cellular hemoglobin being produced. c. There is a deficiency in the production of globulin chains. d. The size and depth of the hemoglobin are affected.

a. Normal adult hemoglobin is replaced by abnormal hemoglobin. Sickle cell anemia is one of a group of diseases collectively called hemoglobinopathies, in which normal adult hemoglobin is replaced by abnormal hemoglobin. Aplastic anemia is a lack of cellular elements being produced. Thalassemia major refers to a variety of inherited disorders characterized by deficiencies in production of certain globulin chains. Iron deficiency anemia affects the size and depth of the color.

A hospitalized child with minimal change nephrotic syndrome is receiving high doses of prednisone. Which is an appropriate nursing goal related to this? a. Prevent infection. b. Stimulate appetite. c. Detect evidence of edema. d. Ensure compliance with prophylactic antibiotic therapy.

a. Prevent infection High-dose steroid therapy has an immunosuppressant effect. These children are particularly vulnerable to upper respiratory tract infections. A priority nursing goal is to minimize the risk of infection by protecting the child from contact with infectious individuals. Appetite is increased with prednisone therapy. The amount of edema should be monitored as part of the disease process, not necessarily related to the administration of prednisone. Antibiotics would not be used as prophylaxis.

A young child with leukemia has anorexia and severe stomatitis. The nurse should suggest that the parents try which intervention? a. Relax any eating pressures. b. Firmly insist that child eat normally. c. Begin gavage feedings to supplement diet. d. Serve foods that are either hot or cold.

a. Relax any eating pressures. A multifaceted approach is necessary for children with severe stomatitis and anorexia. First, the parents should relax eating pressures. The nurse should suggest that the parents try soft, bland foods; normal saline or bicarbonate mouthwashes; and local anesthetics. The stomatitis is a temporary condition. The child can resume good food habits as soon as the condition resolves.

The nurse is observing parents playing with their 10-month-old daughter. What should the nurse recognize as evidence that the child is developing object permanence? a. She looks for the toy the parents hide under the blanket. b. She returns the blocks to the same spot on the table. c. She recognizes that a ball of clay is the same when flattened out. d. She bangs two cubes held in her hands.

a. She looks for the toy the parents hide under the blanket.

suddenly begins to wheeze and have severe urticaria. Which is the most appropriate nursing action? a. Stop drug infusion immediately. b. Recheck rate of drug infusion. c. Observe child closely for next 10 minutes. d. Explain to child that this is an expected side effect.

a. Stop drug infusion immediately. If an allergic reaction is suspected, the drug should be immediately discontinued. Any drug in the line should be withdrawn, and a normal saline infusion begun to keep the line open. Rechecking the rate of drug infusion, observing the child closely for next 10 minutes, and explaining to the child that this is an expected side effect can all be done after the drug infusion is stopped and the child is evaluated.

At what age do children tend to imitate the religious gestures and behaviors of others without understanding their significance? a. Toddlerhood c. Older school-age period b. Young school-age period d. Adolescence

a. Toddlerhood

When a child has chronic renal failure, the progressive deterioration produces a variety of clinical and biochemical disturbances that eventually are manifested in the clinical syndrome known as: a. uremia. b. oliguria. c. proteinuria. d. pyelonephritis.

a. Uremia Uremia is the retention of nitrogenous products, producing toxic symptoms. Oliguria is diminished urinary output. Proteinuria is the presence of protein, usually albumin, in the urine. Pyelonephritis is an inflammation of the kidney and renal pelvis.

A possible cause of acquired aplastic anemia in children is: a. drugs. b. injury. c. deficient diet. d. congenital defect.

a. drugs. Drugs, such as chemotherapeutic agents and several antibiotics (e.g., chloramphenicol), can cause aplastic anemia. Injury, deficient diet, and congenital defect are not causative agents in acquired aplastic anemia.

Meperidine (Demerol) is not recommended for children in sickle cell crisis because it: a. may induce seizures. b. is easily addictive. c. is not adequate for pain relief. d. is given by intramuscular injection. ANS: A A metabolite of meperidine, normeperidine, is a central nervous system stimulant that produces anxiety, tremors, myoclonus, and generalized seizures when it accumulates with repetitive dosing. Patients with sickle cell disease are particularly at risk for normeperidine-induced seizures. Meperidine is no more addictive than other narcotic agents. Meperidine is adequate for pain relief. It is available for IV infusion.

a. may induce seizures. A metabolite of meperidine, normeperidine, is a central nervous system stimulant that produces anxiety, tremors, myoclonus, and generalized seizures when it accumulates with repetitive dosing. Patients with sickle cell disease are particularly at risk for normeperidine-induced seizures. Meperidine is no more addictive than other narcotic agents. Meperidine is adequate for pain relief. It is available for IV infusion.

The nurse is planning care for a school-age child admitted to the hospital with hemophilia. Which interventions should the nurse plan to implement for this child? (Select all that apply.) a. Finger sticks for blood work instead of venipunctures b. Avoidance of IM injections c. Acetaminophen (Tylenol) for mild pain control d. Soft tooth brush for dental hygiene e. Administration of packed red blood cells

b. Avoidance of IM injections c. Acetaminophen (Tylenol) for mild pain control d. Soft tooth brush for dental hygiene ANS: B, C, D Nurses should take special precautions when caring for a child with hemophilia to prevent the use of procedures that may cause bleeding, such as IM injections. The subcutaneous route is substituted for IM injections whenever possible. Venipunctures for blood samples are usually preferred for these children. There is usually less bleeding after the venipuncture than after finger or heel punctures. Neither aspirin nor any aspirin-containing compound should be used. Acetaminophen is a suitable aspirin substitute, especially for controlling mild pain. A soft toothbrush is recommended for dental hygiene to prevent bleeding from the gums. Packed red blood cells are not administered. The primary therapy for hemophilia is replacement of the missing clotting factor. The products available are factor VIII concentrates.

Calcium carbonate is given with meals to a child with chronic renal disease. The purpose of this is to: a. prevent vomiting. b. bind phosphorus. c. stimulate appetite. d. increase absorption of fat-soluble vitamins.

b. Bind phosphorus Oral calcium carbonate preparations combine with phosphorus to decrease gastrointestinal absorption and the serum levels of phosphate. Serum calcium levels are increased by the calcium carbonate, and vitamin D administration is necessary to increase calcium absorption. Calcium carbonate does not prevent vomiting, stimulate appetite, or increase the absorption of fat-soluble vitamins.

The nurse observes some children in the playroom. Which play situation exhibits the characteristics of parallel play? a. Kimberly and Amanda sharing clay to each make things b. Brian playing with his truck next to Kristina playing with her truck c. Adam playing a board game with Kyle, Steven, and Erich d. Danielle playing with a music box on her mother's lap

b. Brian playing with his truck next to Kristina playing with her truck

One of the clinical manifestations of chronic renal failure is uremic frost. Which best describes this term? a. Deposits of urea crystals in urine b. Deposits of urea crystals on skin c. Overexcretion of blood urea nitrogen d. Inability of body to tolerate cold temperatures

b. Deposits of urea crystals on skin Uremic frost is the deposition of urea crystals on the skin. The urea crystals are present on the skin, not in the urine. The kidneys are unable to excrete blood urea nitrogen, leading to elevated levels. There is no relation between cold temperatures and uremic frost.

By the time children reach their twelfth birthday, they should have learned to trust others and should have developed a sense of: a. Identity. c. Integrity. b. Industry. d. Intimacy.

b. Industry.

Which statement is descriptive of renal transplantation in children? a. It is an acceptable means of treatment after age 10 years. b. It is the preferred means of renal replacement therapy in children. c. Children can receive kidneys only from other children. d. The decision for transplantation is difficult because a relatively normal lifestyle is not possible.

b. It is the preferred means of renal replacement therapy in children. Renal transplant offers the opportunity for a relatively normal life and is the preferred means of renal replacement therapy in end-stage renal disease. Renal transplantation can be done in children as young as age 6 months. Both children and adults can serve as donors for renal transplant purposes. Renal transplantation affords the child a more normal lifestyle than dependence on dialysis.

How does the onset of the pubertal growth spurt compare in girls and boys? a. It occurs earlier in boys. b. It occurs earlier in girls. c. It is about the same in both boys and girls. d. In both boys and girls it depends on their growth in infancy.

b. It occurs earlier in girls.

The karyotype of a person is 47, XY, +21. This person is a: a. Normal male. c. Normal female. b. Male with Down syndrome. d. Female with Turner syndrome.

b. Male with Down syndrome.

A school-age child with chronic renal failure is admitted to the hospital with a serum potassium level of 5.2 mEq/L. Which prescribed medication should the nurse plan to administer? a. Spironolactone (Aldactone) b. Sodium polystyrene sulfonate (Kayexalate) c. Lactulose (Cephulac) d. Calcium carbonate (Calcitab)

b. Sodium polystyrene sulfonate (Kayexalate) Normal serum potassium levels in a school-age child are 3.5 to 5 mEq/L. Sodium polystyrene sulfonate is administered to reduce serum potassium levels. Spironolactone is a potassium sparing diuretic and should not be used if the serum potassium is elevated. Lactulose is administered to reduce ammonia levels in patients with liver disease. Calcium carbonate may be prescribed as a calcium supplement, but it will not reduce serum potassium levels.

The nurse has initiated a blood transfusion on a preschool child. The child begins to exhibit signs of a transfusion reaction. Place in order the interventions the nurse should implement sequencing from the highest priority to the lowest. Provide answer using lowercase letters separated by commas (e.g., a, b, c, d). a. Take the vital signs. b. Stop the transfusion. c. Notify the practitioner. d. Maintain a patent IV line with normal saline. ANS: b, a, d, c If a blood transfusion reaction of any type is suspected, stop the transfusion, take vital signs, maintain a patent IV line with normal saline and new tubing, notify the practitioner, and do not restart the transfusion until the child's condition has been medically evaluated.

b. Stop the transfusion. a. Take the vital signs. d. Maintain a patent IV line with normal saline. c. Notify the practitioner. ANS: b, a, d, c If a blood transfusion reaction of any type is suspected, stop the transfusion, take vital signs, maintain a patent IV line with normal saline and new tubing, notify the practitioner, and do not restart the transfusion until the child's condition has been medically evaluated.

A school-age child is admitted in vasoocclusive sickle cell crisis. The child's care should include: a. correction of acidosis. b. adequate hydration and pain management. c. pain management and administration of heparin. d. adequate oxygenation and replacement of factor VIII.

b. adequate hydration and pain management. The management of crises includes adequate hydration, minimization of energy expenditures, pain management, electrolyte replacement, and blood component therapy if indicated. Hydration and pain control are two of the major goals of therapy. The acidosis will be corrected as the crisis is treated. Heparin and factor VIII is not indicated in the treatment of vasoocclusive sickle cell crisis. Oxygen may prevent further sickling, but it is not effective in reversing sickling because it cannot reach the clogged blood vessels.

A young child with human immunodeficiency virus (HIV) is receiving several antiretroviral drugs. The purpose of these drugs is to: a. cure the disease. b. delay disease progression. c. prevent spread of disease. d. treat Pneumocystis carinii pneumonia.

b. delay disease progression. Although not a cure, these antiviral drugs can suppress viral replication, preventing further deterioration of the immune system and delaying disease progression. At this time, cure is not possible. These drugs do not prevent the spread of the disease. P. carinii prophylaxis is accomplished with antibiotics.

Chelation therapy is begun on a child with -thalassemia major. The purpose of this therapy is to: a. treat the disease. b. eliminate excess iron. c. decrease risk of hypoxia. d. manage nausea and vomiting.

b. eliminate excess iron. A complication of the frequent blood transfusions in thalassemia is iron overload. Chelation therapy with deferoxamine (an iron-chelating agent) is given with oral supplements of vitamin C to increase iron excretion. Chelation therapy treats the side effect of the disease management. Decreasing the risk of hypoxia and managing nausea and vomiting are not the purposes of chelation therapy.

A child is admitted with acute glomerulonephritis. The nurse should expect the urinalysis during this acute phase to show: a. bacteriuria, hematuria. b. hematuria, proteinuria. c. bacteriuria, increased specific gravity. d. proteinuria, decreased specific gravity.

b. hematuria, proteinuria Urinalysis during the acute phase characteristically shows hematuria and proteinuria. Bacteriuria and changes in specific gravity are not usually present during the acute phase.

The nurse is teaching parents of a child with chronic renal failure (CRF) about the use of recombinant human erythropoietin (rHuEPO) subcutaneous injections. Which statement indicates the parents have understood the teaching? a. "These injections will help with the hypertension." b. "We're glad the injections only need to be given once a month." c. "The red blood cell count should begin to improve with these injections." d. "Urine output should begin to improve with these injections."

c. "The red blood cell count should begin to improve with these injections." Anemia in children with CRF is related to decreased production of erythropoietin. Recombinant human erythropoietin (rHuEPO) is being offered to these children as thrice-weekly or weekly subcutaneous injections and is replacing the need for frequent blood transfusions. The parents understand the teaching if they say that the red blood cell count will begin to improve with these injections.

By what age does birth length usually double? a. 1 year c. 4 years b. 2 years d. 6 years

c. 4 years

Which should the nurse include when teaching the mother of a 9-month-old infant about administering liquid iron preparations? a. They should be given with meals. b. They should be stopped immediately if nausea and vomiting occur. c. Adequate dosage will turn the stools a tarry green color. d. Allow preparation to mix with saliva and bathe the teeth before swallowing.

c. Adequate dosage will turn the stools a tarry green color. The nurse should prepare the mother for the anticipated change in the child's stools. If the iron dose is adequate, the stools will become a tarry green color. The lack of the color change may indicate insufficient iron. The iron should be given in two divided doses between meals when the presence of free hydrochloric acid is greatest. Iron is absorbed best in an acidic environment. Vomiting and diarrhea may occur with iron administration. If these occur, the iron should be given with meals, and the dosage reduced, then gradually increased as the child develops tolerance. Liquid preparations of iron stain the teeth. They should be administered through a straw and the mouth rinsed after administration.

A school-age child with leukemia experienced severe nausea and vomiting when receiving chemotherapy for the first time. Which is the most appropriate nursing action to prevent or minimize these reactions with subsequent treatments? a. Encourage drinking large amounts of favorite fluids. b. Encourage child to take nothing by mouth (remain NPO) until nausea and vomiting subside. c. Administer an antiemetic before chemotherapy begins. d. Administer an antiemetic as soon as child has nausea.

c. Administer an antiemetic before chemotherapy begins. The most beneficial regimen to minimize nausea and vomiting associated with chemotherapy is to administer the antiemetic before the chemotherapy is begun. The goal is to prevent anticipatory symptoms. Drinking fluids will add to the discomfort of the nausea and vomiting. Remaining until nausea and vomiting subside will help with this episode, but the child will have the discomfort and be at risk for dehydration. Administering an antiemetic as soon as the child has nausea does not prevent anticipatory nausea.

In what type of play are children engaged in similar or identical activity without organization, division of labor, or mutual goal? a. Solitary c. Associative b. Parallel d. Cooperative

c. Associative

Which strategy would be the least appropriate for a child to use to cope? a. Learning problem solving c. Having parents solve problems b. Listening to music d. Using relaxation techniques

c. Having parents solve problems

Which behavior is most characteristic of the concrete operations stage of cognitive development? a. Progression from reflex activity to imitative behavior b. Inability to put oneself in another's place c. Increasingly logical and coherent thought processes d. Ability to think in abstract terms and draw logical conclusions

c. Increasingly logical and coherent thought processes

Which is most descriptive of the pathophysiology of leukemia? a. Increased blood viscosity occurs. b. Thrombocytopenia (excessive destruction of platelets) occurs. c. Unrestricted proliferation of immature white blood cells (WBCs) occurs. d. First stage of coagulation process is abnormally stimulated.

c. Unrestricted proliferation of immature white blood cells (WBCs) occurs. Leukemia is a group of malignant disorders of the bone marrow and lymphatic system. It is defined as an unrestricted proliferation of immature WBCs in the blood-forming tissues of the body. Increased blood viscosity may occur secondary to the increased number of WBCs. Thrombocytopenia may occur secondary to the overproduction of WBCs in the bone marrow. The coagulation process is unaffected by leukemia.

Which is a major complication in a child with chronic renal failure? a. Hypokalemia b. Metabolic alkalosis c. Water and sodium retention d. Excessive excretion of blood urea nitrogen

c. Water and sodium retention Chronic renal failure leads to water and sodium retention, which contributes to edema and vascular congestion. Hyperkalemia, metabolic acidosis, and retention of blood urea nitrogen are complications of chronic renal failure.

Parents of a hemophiliac child ask the nurse, "Can you describe hemophilia to us?" Which response by the nurse is descriptive of most cases of hemophilia? a. Autosomal dominant disorder causing deficiency in a factor involved in the blood-clotting reaction b. X-linked recessive inherited disorder causing deficiency of platelets and prolonged bleeding c. X-linked recessive inherited disorder in which a blood-clotting factor is deficient d. Y-linked recessive inherited disorder in which the red blood cells become moon-shaped

c. X-linked recessive inherited disorder in which a blood-clotting factor is deficient ANS: C The inheritance pattern in 80% of all of the cases of hemophilia is X-linked recessive. The two most common forms of the disorder are factor VIII deficiency, hemophilia A or classic hemophilia; and factor IX deficiency, hemophilia B or Christmas disease. The inheritance pattern is X-linked recessive. The disorder involves coagulation factors, not platelets, and does not involve red cells or the Y chromosomes.

Iron dextran is ordered for a young child with severe iron deficiency anemia. Nursing considerations include to: a. administer with meals. b. administer between meals. c. inject deeply into a large muscle. d. massage injection site for 5 minutes after administration of drug.

c. inject deeply into a large muscle. Iron dextran is a parenteral form of iron. When administered intramuscularly, it must be injected into a large muscle. Iron dextran is for intramuscular or intravenous (IV) administration. The site should not be massaged to prevent leakage, potential irritation, and staining of the skin.

The nurse notes that a child has lost 8 pounds after 4 days of hospitalization for acute glomerulonephritis. This is most likely the result of: a. poor appetite. b. increased potassium intake. c. reduction of edema. d. restriction to bed rest.

c. reduction of edema This amount of weight loss in this period is a result of the improvement of renal function and mobilization of edema fluid. Poor appetite and bed rest would not result in a weight loss of 8 pounds in 4 days. Foods with substantial amounts of potassium are avoided until renal function is normalized.

Several blood tests are ordered for a preschool child with severe anemia. The child is crying and upset because of memories of the venipuncture done at the clinic 2 days ago. The nurse should explain: a. venipuncture discomfort is very brief. b. only one venipuncture will be needed. c. topical application of local anesthetic can eliminate venipuncture pain. d. most blood tests on children require only a finger puncture because a small amount of blood is needed.

c. topical application of local anesthetic can eliminate venipuncture pain. Preschool children are concerned with both pain and the loss of blood. When preparing the child for venipuncture, the nurse will use a topical anesthetic to eliminate any pain. This is a traumatic experience for preschool children. They are concerned about their bodily integrity. A local anesthetic should be used, and a bandage should be applied to maintain bodily integrity. The nurse should not promise one attempt in case multiple attempts are required. Both finger punctures and venipunctures are traumatic for children. Both require preparation.

The nurse is reviewing first aid with a group of school nurses. Which statement made by a participant indicates a correct understanding of the information? a. "If a child loses a tooth due to injury, I should place the tooth in warm milk." b. "If a child has recurrent abdominal pain, I should send him or her back to class until the end of the day." c. "If a child has a chemical burn to the eye, I should irrigate the eye with normal saline." d. "If a child has a nosebleed, I should have the child sit up and lean forward."

d. "If a child has a nosebleed, I should have the child sit up and lean forward." If a child has a nosebleed, the child should lean forward, not lie down. A tooth should be placed in cold milk or saliva for transporting to a dentist. Recurrent abdominal pain is a physiologic problem and requires further evaluation. If a chemical burn occurs in the eye, the eye should be irrigated with water for 20 minutes.

A 13-year-old girl asks the nurse how much taller she will become. She has been growing about 2 inches per year but grew 4 inches this past year. Menarche recently occurred. The nurse should base her response on knowing that: a. Growth cannot be predicted. b. The pubertal growth spurt lasts about 1 year. c. Mature height is achieved when menarche occurs. d. Approximately 95% of mature height is achieved when menarche occurs.

d. Approximately 95% of mature height is achieved when menarche occurs.

The nurse is caring for a child with acute renal failure. Which clinical manifestation should the nurse recognize as a sign of hyperkalemia? a. Dyspnea b. Seizure c. Oliguria d. Cardiac arrhythmia

d. Cardiac arrhythmia Hyperkalemia is the most common threat to the life of the child. Signs of hyperkalemia include electrocardiograph anomalies such as prolonged QRS complex, depressed ST segments, peaked T waves, bradycardia, or heart block. Dyspnea, seizure, and oliguria are not manifestations of hyperkalemia.

Which is the most effective pain-management approach for a child who is having a bone marrow aspiration? a. Relaxation techniques b. Administration of an opioid c. EMLA cream applied over site d. Conscious or unconscious sedation

d. Conscious or unconscious sedation Effective pharmacologic and nonpharmacologic measures should be used to minimize pain associated with procedures. For bone marrow aspiration, conscious or unconscious sedation should be used. Relaxation, opioids, and EMLA can be used to augment the conscious or unconscious sedation.

Three children playing a board game would be an example of: a. Solitary play c. Associative play b. Parallel play d. Cooperative play

d. Cooperative play

Which is a common clinical manifestation of Hodgkin disease? a. Petechiae b. Bone and joint pain c. Painful, enlarged lymph nodes d. Enlarged, firm, nontender lymph nodes

d. Enlarged, firm, nontender lymph nodes Asymptomatic, enlarged, cervical or supraclavicular lymphadenopathy is the most common presentation of Hodgkin disease. Petechiae are usually associated with leukemia. Bone and joint pain are not likely in Hodgkin disease. The enlarged nodes are rarely painful.

Trauma to which site can result in a growth problem for children's long bones? a. Matrix c. Calcified cartilage b. Connective tissue d. Epiphyseal cartilage plate

d. Epiphyseal cartilage plate

The nurse is preparing a child for possible alopecia from chemotherapy. Which should be included? a. Explain to child that hair usually regrows in 1 year. b. Advise child to expose head to sunlight to minimize alopecia. c. Explain to child that wearing a hat or scarf is preferable to wearing a wig. d. Explain to child that when hair regrows, it may have a slightly different color or texture.

d. Explain to child that when hair regrows, it may have a slightly different color or texture. Alopecia is a side effect of certain chemotherapeutic agents. When the hair regrows, it may be a different color or texture. The hair usually grows back within 3 to 6 months after cessation of treatment. The head should be protected from sunlight to avoid sunburn. Children should choose the head covering they prefer.

Which clinical manifestation should the nurse expect when a child with sickle cell anemia experiences an acute vasoocclusive crisis? a. Circulatory collapse b. Cardiomegaly, systolic murmurs c. Hepatomegaly, intrahepatic cholestasis d. Painful swelling of hands and feet; painful joints

d. Painful swelling of hands and feet; painful joints A vasoocclusive crisis is characterized by severe pain in the area of involvement. If in the extremities, painful swelling of the hands and feet is seen; if in the abdomen, severe pain resembles that of acute surgical abdomen; and if in the head, stroke and visual disturbances occur. Circulatory collapse results from sequestration crises. Cardiomegaly, systolic murmurs, hepatomegaly, and intrahepatic cholestasis result from chronic vasoocclusive phenomena.

Which is an advantage of peritoneal dialysis? a. Treatments are done in hospitals. b. Protein loss is less extensive. c. Dietary limitations are not necessary. d. Parents and older children can perform treatments.

d. Parents and older children can perform treatments. Peritoneal dialysis is the preferred form of dialysis for parents, infants, and children who wish to remain independent. Parents and older children can perform the treatments themselves. Treatments can be done at home. Protein loss is not significantly different. The dietary limitations are necessary, but they are not as stringent as those for hemodialysis.

The nurse is admitting a newborn with hypospadias to the nursery. The nurse expects which finding in this newborn? a. Absence of a urethral opening is noted. b. Penis appears shorter than usual for age. c. The urethral opening is along the dorsal surface of the penis. d. The urethral opening is along the ventral surface of the penis.

d. The urethral opening is along the ventral surface of the penis. Hypospadias is a congenital condition in which the urethral opening is located anywhere along the ventral surface of the penis. The urethral opening is present, but not at the glans. Hypospadias refers to the urethral opening, not to the size of the penis. Urethral opening along ventral surface of penis is known as epispadias.

Lymphoid tissues such as lymph nodes are: a. Adult size by age 1 year. b. Adult size by age 13 years. c. Half their adult size by age 5 years. d. Twice their adult size by age 10 to 12 years.

d. Twice their adult size by age 10 to 12 years.

Which clinical manifestation would be seen in a child with chronic renal failure? a. Hypotension b. Massive hematuria c. Hypokalemia d. Unpleasant "uremic" breath odor

d. Unpleasant "uremic" breath odor Children with chronic renal failure have a characteristic breath odor resulting from the retention of waste products. Hypertension may be a complication of chronic renal failure. With chronic renal failure, little or no urinary output occurs. Hyperkalemia is a concern in chronic renal failure.

The nurse is caring for an adolescent who has just started dialysis. The child seems always angry, hostile, or depressed. The nurse should recognize that this is most likely related to: a. neurologic manifestations that occur with dialysis. b. physiologic manifestations of renal disease. c. adolescents having few coping mechanisms. d. adolescents often resenting the control and enforced dependence imposed by dialysis.

d. adolescents often resenting the control and enforced dependence imposed by dialysis. Older children and adolescents need control. The necessity of dialysis forces the adolescent into a dependent relationship, which results in these behaviors. These are a function of the child's age, not neurologic or physiologic manifestations of the dialysis. Feelings of anger, hostility, and depression are functions of the child's age, not neurologic or physiologic manifestations of the dialysis. Adolescents do have coping mechanisms, but they need to have some control over their disease management.

A child with leukemia is receiving triple intrathecal chemotherapy consisting of methotrexate, cytarabine, and hydrocortisone. The purpose of this is to prevent: a. infection. b. brain tumor. c. drug side effects. d. central nervous system (CNS) disease.

d. central nervous system (CNS) disease. For certain children, CNS prophylactic therapy is indicated. This drug regimen is used to prevent CNS leukemia and will not prevent infection or drug side effects. If the child has a brain tumor in addition to leukemia, additional therapy would be indicated.

An 8-year-old girl is receiving a blood transfusion when the nurse notes that she has developed precordial pain, dyspnea, distended neck veins, slight cyanosis, and a dry cough. These manifestations are most suggestive of: a. air emboli. b. allergic reaction. c. hemolytic reaction. d. circulatory overload.

d. circulatory overload. The signs of circulatory overload include distended neck veins, hypertension, crackles, dry cough, cyanosis, and precordial pain. Signs of air embolism are sudden difficulty breathing, sharp pain in the chest, and apprehension. Allergic reactions are manifested by urticaria, pruritus, flushing, asthmatic wheezing, and laryngeal edema. Hemolytic reactions are characterized by chills, shaking, fever, pain at infusion site, nausea, vomiting, tightness in chest, flank pain, red or black urine, and progressive signs of shock and renal failure.

Myelosuppression, associated with chemotherapeutic agents or some malignancies such as leukemia, can cause bleeding tendencies because of a(n): a. decrease in leukocytes. b. increase in lymphocytes. c. vitamin C deficiency. d. decrease in blood platelets.

d. decrease in blood platelets. The decrease in blood platelets secondary to the myelosuppression of chemotherapy can cause an increase in bleeding. The child and family should be alerted to avoid risk of injury. Decrease in leukocytes, increase in lymphocytes, and vitamin C deficiency would not affect bleeding tendencies.

A mother asks the nurse what would be the first indication that acute glomerulonephritis is improving. The nurse's best response should be that the: a. blood pressure will stabilize. b. the child will have more energy. c. urine will be free of protein. d. urinary output will increase.

d. urinary output will increase. An increase in urinary output may signal resolution of the acute glomerulonephritis. If blood pressure is elevated, stabilization usually occurs with the improvement in renal function. The child having more energy and the urine being free of protein are related to the improvement in urinary output.


Kaugnay na mga set ng pag-aaral

Chapter 14: Bonds and Long Term Notes

View Set

RG Exam Review: Dental Technology History & Theory

View Set

federal tax considerations for life insurance and annuities

View Set

Quiz chapter 10, MS1 Quiz 14: Chapter 9, Iggy: Chapter 21 Chapter 22 MedSurg Exam 4 Review (Evolve Questions)

View Set

Chapter 23 - A Changing Medieval World

View Set

ANT2000 FSU Midterm & Final (the final section is not finished)

View Set

Ch.27 assessment & management of hypertension

View Set

Test 2 Asthma/COPD/Chest Trauma/Respiratory Failure/ABG/TB.HIV

View Set